Practice Questions

अब Quizwiz के साथ अपने होमवर्क और परीक्षाओं को एस करें!

Psychodynamic Theories of Development

* Erikson

Trait theories of personality

* Gordon Allport * Raymond Cattell 16 Personality Factor Questionnaire * Eyseneck's Three Dimensions of Personality * Five-Factor Theory

Social Exchange Theory

* attraction is affected by the costs and benefits of being in a relationship

Erich Fromm

* experience of freedom frightens most people * "having" mode vs "being" mode

Continuous variable

* interval or ratio

Attachment - Harlow

* studied monkeys * "contact comfort"

Suppressor variable

* suppresses a correlation that exists

According to nigrescence models, the deracinated person is most likely to be in which of the following stages? • 1. Preencounter. • 2. Encounter. • 3. Immersion-emersion. • 4. Internalization.

1. Preencounter. (correct answer)

Of the following, the medication least implicated in causing anticholinergic side effects is: • 1. Sertraline. • 2. Amitriptyline. • 3. Imipramine. • 4. Doxepin.

1. Sertraline. (correct answer)

You are treating a patient who is diagnosed with schizoid personality disorder. According to Millon, your patient is most likely to resort to the defense of: • 1. intellectualization. • 2. rationalization. • 3. projection. • 4. regression.

1. intellectualization. (correct answer)

. A teaching assistant is asked to observe a hyperactive student's on-task behavior. She would most likely be using: • 1. interval recording. • 2. content sampling. • 3. event recording. • 4. purposive sampling.

1. interval recording. (correct answer)

According to Sandra Bem, persons with well-defined gender schemas: • 1. might manipulate, distort, or ignore information that does not conform to their gender expectations. • 2. tend to evidence flexibility in terms of gender expectations. • 3. tend to be better adjusted as compared with persons without well-defined gender schemas. • 4. do not clearly distinguish gender-inappropriate behaviors.

1. might manipulate, distort, or ignore information that does not conform to their gender expectations. (correct answer)

A researcher would use LISREL, a form of structural equation modeling, in an attempt to: • 1. test a causal model of relationships among variables. • 2. derive a causal model of relationships among variables. • 3. determine whether underlying variables are latent or manifest. • 4. determine the non-linear relationship between multiple IVs and multiple DVs.

1. test a causal model of relationships among variables. (correct answer)

You are working on developing a new test. To optimize your reliability you should: • 1. utilize a heterogeneous sample. • 2. utilize a homogeneous sample. • 3. measure subjects on the same test at two different points in time. • 4. measure subjects on two similar versions of the test at two different points in time.

1. utilize a heterogeneous sample. (correct answer, your response) There are several factors that specifically increase reliability: more items on a test, homogeneity of the items, an unrestricted range of scores that results from a more heterogeneous sample, and difficulty of guessing. Measuring subjects on the same test at two points in time simply describes test-retest reliability (Response 3). Measuring subjects on two similar versions of the test at two different points in time describes alternate-forms reliability (Response 4).

The best treatment for someone who has difficulty making friends, is shy, and can't maintain steady employment is: a. individual social skills training. b. paradoxical intention. c. token economy. d. group social skills training

19. D-- Treatment for very shy individuals who have trouble in interpersonal settings (as implied by the fact that the person cannot hold a job) typically involves social skills and assertiveness training. Although both of these treatments can be administered in individual therapy, the group format has a number of advantages: there are multiple models, and opportunities for feedback, support, and vicarious learning. These aren't available in individual training sessions. Thus, D is the best answer.

Which statement about McClelland's Acquired Needs theory is most accurate? • 1. People's work-related needs are generally stable throughout the life span. • 2. Exposure to different experiences can result in changes in people's work-related needs. • 3. Work-related needs develop early in life and are usually not modified. • 4. Research results have generally been inconsistent regarding whether work-related needs are stable or modifiable.

2. Exposure to different experiences can result in changes in people's work-related needs. (correct answer)

When aversive counterconditioning is used to reduce certain sexual behaviors, the fetish object is: • 1. an unconditioned stimulus. • 2. a conditioned stimulus. • 3. an unconditioned response. • 4. a conditioned response.

2. a conditioned stimulus. (correct answer)

Adolescents go through stages in order to form a mature identity. An adolescent who is at the identity status of moratorium would: • 1. be committed to a goal after having explored several alternatives. • 2. be in the process of exploration but not yet committed to a goal. • 3. have accepted a ready made identity that an authority figure may have chosen. • 4. not be committed to clear goals, nor be actively trying to reach them.

2. be in the process of exploration but not yet committed to a goal. (correct answer)

A teacher that works in a Montessori school seeks consultation because he is having problems managing his classroom that consists of children who are often disruptive. This illustrates: • 1. client-centered case consultation. • 2. consultee-centered case consultation. • 3. consultee-centered administrative consultation. • 4. program-centered administrative consultation.

2. consultee-centered case consultation. (correct answer)

A husband and wife have decided to use mediation to help end their marriage. At the end of the mediation process, both people are satisfied with the terms of the divorce settlement. This is an example of: • 1. informational justice. • 2. distributive justice. • 3. procedural justice. • 4. interpersonal justice.

2. distributive justice. (correct answer)

In attachment theory, early relationships are believed to influence later attachment patterns. The mechanism by which this influence is achieved is termed: • 1. internalized object world. • 2. internal working model. • 3. relational reciprocity. • 4. prototype activation.

2. internal working model. (correct answer)

A student acts in a disruptive manner and only stops when the teacher yells at her. The teacher's behavior of yelling is under the control of: • 1. positive reinforcement. • 2. negative reinforcement. • 3. positive punishment. • 4. negative punishment.

2. negative reinforcement. (correct answer)

The circadian clock is located in the hypothalamus and is responsible for sleep-wake patterns. The circadian clock is the: • 1. pineal gland • 2. suprachiasmic nucleus • 3. substantia nigra • 4. reticular activating system (RAS)

2. suprachiasmic nucleus (correct answer)

A man is presented with a list of nine words to remember. Five minutes after presentation, he is asked to recall the words. You would expect that: • 1. the man may remember all the items because a person can hold seven plus or minus two items in short-term memory. • 2. the man will remember items from the beginning of the list better than from the middle and end of the list. • 3. the man will remember items from the end of the list better than from the beginning and middle of the list. • 4. the man will remember items from the beginning of the list and from the end of the list best.

2. the man will remember items from the beginning of the list better than from the middle and end of the list. (correct answer)

For every five minutes that a hyperactive child does not speak out of turn, his teacher gives him a "green ticket." At the end of each hour, the child gets a minute of play time for every green ticket he has earned. This method of behavior modification is best described as: • 1. token economy. • 2. higher-order conditioning. • 3. DRO. • 4. shaping.

3. DRO. (correct answer)

The maximum value of the standard error of the measurement is: • 1. 1. • 2. rxx. • 3. SDx. • 4. SDy.

3. SDx. (correct answer, your response) You can approach this question by applying the formula for the standard error of measurement Smeas=SDxÖ1-rxx: . The lowest value would then be 0 (if rxx=1) and the highest value would SDx (if rxx=0). Note that SDy (Response 4) is the highest value for the standard error of the estimate

Handedness appears to be genetically determined. Preference for handedness first expresses itself at ____ and becomes firmly established by age ____: • 1. birth, four to five. • 2. age two, four to five. • 3. age two, seven to eight. • 4. age four, seven to eight.

3. age two, seven to eight. (correct answer, your response) While hand preference typically emerges around age two, it only becomes firmly established around age seven to eight. This age corresponds to the age associated with increased brain specialization and decreased brain plasticity.

A man is brought to the emergency room by the police after a fight at a local tavern. He has dilated pupils, seems agitated, and reports feeling nauseated. These symptoms are most likely the result of: • 1. alcohol intoxication. • 2. cocaine withdrawal. • 3. amphetamine intoxication. • 4. caffeine intoxication.

3. amphetamine intoxication. (correct answer)

After a car accident, a patient presents with complaints of significant memory deficits. This patient is most likely to be malingering if testing reveals: • 1. intact verbal memory, deficits in non-verbal memory. • 2. more significant problems with retrograde amnesia than anterograde amnesia. • 3. intact attention. • 4. more significant problems with recent memory than remote memory.

3. intact attention. (correct answer)

A person with Klinefelter's syndrome: • 1. is a woman with an extra X chromosome. • 2. is a woman missing an X chromosome. • 3. is a man with an extra X chromosome. • 4. is a man with an extra Y chromosome.

3. is a man with an extra X chromosome. (correct answer) Klinefelter's syndrome is a condition in which a mutation in the cell division process of one of the parents gametes results in a male infant who has an extra X chromosome (XXY). The most common symptom of Klinefelter's syndrome is infertility. There may also be abnormal development of secondary sex characteristics such as less body hair, less muscle development or partial breast development. Response 1 describes a woman with Triple X Syndrome (XXX), which may result in infertility. Response 2 describes a woman with Turner's syndrome (XO), a chromosomal abnormality that may result in abnormal development of secondary sexual characteristics (e.g., no menstruation, no ovulation) and infertility. Response 4 describes a male with XYY syndrome, a rare abnormality which results in men that are tall and thin and may have behavioral problems or learning disabilities.

In terms of the development of the brain, the stage following differentiation is: • 1. proliferation. • 2. migration. • 3. myelination. • 4. synaptogenesis.

3. myelination. (correct answer)

The halo effect can be controlled by all of the following except: • 1. training the raters. • 2. utilizing forced choice. • 3. utilizing relative methods. • 4. utilizing BARS.

3. utilizing relative methods. (correct answer)

Your supervisor assigns you a project in which you have very little interest. Cognitive Dissonance Theory predicts that you would be most likely to change your opinion of this project if: • 1. you are given a bonus in exchange for doing this project. • 2. the project greatly benefits your colleagues. • 3. you complete the project on your own time at home. • 4. you have expertise with this type of project.

3. you complete the project on your own time at home. (correct answer)

You are working with a college student who hopes to go to medical school. He expresses concern that he faints at the site of blood. Of the following, which would probably be the most effective treatment? • 1. Positive reinforcement. • 2. Cognitive therapy. • 3. Systematic desensitization. • 4. Exposure.

4. Exposure. (correct answer) The treatment of choice for specific phobias is flooding, also called exposure with response prevention. While systematic desensitization is still used in the treatment of specific phobias, exposure-based techniques are superior. Treatment approaches based on operant conditioning (e.g., positive reinforcement) (Response 1) are not used for specific phobias. Cognitive therapy (Response 2) is widely used for generalized anxiety disorder, as well as for depressive disorders.

Taylor-Russell tables demonstrate that to optimize your incremental validity you would want: • 1. both a base rate and a selection ratio of about .5. • 2. both a base rate and a selection ratio of about .1. • 3. a base rate of .1 and a selection ratio of .5. • 4. a base rate of .5 and a selection ratio of .1.

4. a base rate of .5 and a selection ratio of .1. (correct answer)

Which of the following personality variables is significantly correlated with job performance across the widest variety of occupations? a. agreeableness b. emotional stability c. extraversion d. conscientiousness

43. D-- Factor analytic studies have identified five personality dimensions -- agreeableness, emotional stability, extraversion, openness, and conscientiousness -- that appear to underlie all other personality traits. The dimensions are commonly referred to in the literature "The Big Five." Of these, conscientiousness has the highest correlation with job performance and training success across a wide range of settings.

Raising the cutoff score on a predictor test would have the effect of a. increasing true positives b. decreasing false positives c. decreasing true negatives d. decreasing false negatives.

49. B-- A simple way to answer this question is with reference to a chart such as the one displayed under the topic "Criterion-Related Validity" in the Test Construction section of your materials. If you look at this chart, you can see that increasing the predictor cutoff score (i.e., moving the vertical line to the right) decreases the number of false positives as well as true positives (you can also see that the number of both true and false negatives would be increased). You can also think about this question more abstractly by coming up with an example. Imagine, for instance, that a general knowledge test is used as a predictor of job success. If the cutoff score on this test is raised, fewer people will score above this cutoff and, therefore, fewer people will be predicted to be successful. Another way of saying this is that fewer people will come up "positive" on this predictor. This applies to both true positives and false positives.

The brand name for one of the medications used for ADHD is: a. Tegretol b. Depakote c. Dolophine d. Dexedrine

55. D-- Dexedrine is sometimes used in the treatment of ADHD. Tegretol and Depakote are both used in the treatment of bipolar disorder. Dolophine is used for the treatment of heroin addiction.

65. Huntington's Disease is most associated with decreased amounts of: a. dopamine b. epinephrine c. GABA d. norepinephrine

65. C-- Huntington's Disease is believed to begin when cells within the striatum (caudate and putamen) of the basal ganglia begin to be destroyed. The striatum is responsible for producing GABA, which regulates the levels of dopamine in the brain through an inhibitory process. The death of the striatum cells causes decreased amounts of GABA which leads to an overproduction of dopamine and results in chorea (uncontrollable and irregular muscle movements, especially of the arms, legs, and face).

84. Normal aging is least likely to negatively affect: a. free recall b. cued recall c. working memory d. picture recognition

84. D-- You may have recalled that free recall is more affected by age than cued recall; however, picture recognition is the least demanding cognitive process of all of the choices, and is, therefore, the least affected by aging. [See D.C. Park, Mediators of long-term memory performance across the lifespan, Psychology and Aging, 1996, 11(4), 621-637].

92. Walking through a dark park at night, you hear footsteps behind you. You think it may be a mugger so your breathing deepens and heart beats faster, at the same time you experience fear. This is an example of which theory? a. Cannon-Bard b. James-Lange c. Lazarus d. Schachter-Singer

92. C-- In contrast to the other three theories, Lazarus' theory proposes that a thought must precede any emotion or physiological arousal. Cannon-Bard (a.) states physiological and emotional arousal are experienced at the same time. According to the other two, an event causes physiological arousal first, then interpretation (b.) or reasoning (d.) and finally the experience of emotion. A fifth theory of emotion, facial feedback, argues that changes in facial muscles cue the brain and provide the basis of emotion.

A behavior therapist working with a child who bites her nails sets a timer so that it rings every 5 minutes. For every 5 minute period during which the child doesn't bite her nails, she is given a token. This treatment is an example of. a. D.R.O. b. response cost. c. overcorrection. d. Premack Principle.

a

A new client of yours, who is homosexual, tells you that he overheard his previous therapist make derogatory remarks about his sexual orientation to a co-worker. Your best course of action would be to. a. discuss his option of filing a complaint against the psychologist with the Ethics Committee. b. file a complaint against the psychologist with the Ethics Committee yourself. c. call the psychologist and tell him that your client overheard his remarks. d. provide the client with support and remind him that there are many prejudiced people in the world.

a

A psychologist would most likely administer the Vineland Scale when. a. she is assessing an individual for Mental Retardation. b. she is assessing an individual for Pervasive Developmental Disorder. c. she is checking an individual's recovery following brain injury. d. she wants to assess the cognitive skills of an individual who has limited English proficiency.

a

According to the path-goal theory of leadership, a primary task of a leader is to help subordinates find the best path for attaining goals. In terms of leadership style, this theory proposes that. a. the best style (directive, supportive, participative, or achievement-oriented) depends on certain characteristics of the task and the subordinates. b. the best style (directive, supportive, participative, or achievement-oriented) depends on the leader's experience, personality, and so on. c. the best style is a participative one that allows employees to help set their own goals. d. the best style is an achievement-oriented one that focuses on enhancing the skills and knowledge of subordinates so that they are better able to achieve their goals.

a

After a training program has been completed, an industrial psychologist wants to determine if the instruction was effective. What type of evaluation will he use. a. summative. b. formative. c. primary. d. secondary.

a

An achievement test is administered to one hundred people. The test's mean is 50 and its standard deviation is 5 and the test scores are normally distributed. If you want to use test scores to select the top 16 people, you would set the cutoff score at. a. 55. b. 60. c. 65. d. 70.

a

An adolescent's adoption of a "ready-made" identity that was formulated by a parent or other authority figure is referred to as. a. foreclosure. b. moratorium. c. fusion. d. emersion.

a

Delirium shares many symptoms with schizophrenia, schizophreniform disorder, and certain other psychotic disorders, such as disordered thinking, hallucinations, and delusions. Thus, the differential diagnosis of these disorders is often difficult. Generally speaking, however, delirium can be distinguished from the psychotic disorders because. a. the symptoms of delirium tend to be random and haphazard while the symptoms of the psychotic disorders are ordinarily systematized. b. the onset of symptoms in delirium is insidious while the onset of symptoms in psychotic disorders is usually rapid. c. clouding of consciousness is rare in delirium but common in the psychotic disorders. d. the psychotic disorders involve hallucinations and delusions but these symptoms are never present in delirium.

a

Elaine considers her friend, Tom, to be quiet and reserved. She says that when she is alone with Tom or when they are in a small group, Tom listens more than he talks and is never one to "make a scene." One day, however, Elaine observes Tom in a large crowd that is waiting for the arrival of a "special celebrity guest" at the opening of a new shopping mall. The guest is nearly 30 minutes late, and the crowd is beginning to show signs of impatience. Elaine is surprised when she sees Tom initiate loud "booing" and other derogatory comments while waving his clenched fist in the air in anger. Which of the following theories or models best explains Tom's unusual behavior. a. deindividuation. b. social facilitation. c. catharsis. d. paradoxical intention.

a

In group therapy, members establish a "taking turns"pattern in which each group session is devoted, sequentially, to each group member. In response to this situation, Yalom recommends: a. mass group interpretation. b. here-and-now activation. c. paradoxical prescription. d. reframing

a

Kohlberg argued that there is a monotonic relationship between moral judgment and moral action and proposed that, as one moves from a lower to a higher stage of moral development. a. the range of possible moral actions becomes narrower and the individual assumes greater responsibility for relating his or her judgments to actions. b. the range of possible moral actions becomes broader and the individual assumes greater responsibility for relating his or her judgments to actions. c. the range of possible moral actions remains the same but the individual assumes greater responsibility for relating his or her judgments to actions. d. the range of possible moral actions becomes narrower but the individual's sense of responsibility for relating his or her judgments to actions remains about the same.

a

Of the following, which is the best way to control the halo effect. a. train the raters. b. use peer ratings. c. make the results of assessments available to employees. d. involve raters in the development of the rating scale.

a

Research comparing heterogeneous to homogeneous work groups has found that heterogeneous groups. a. are more creative and better at decision-making. b. are more creative but less productive overall. c. make better decisions but are less productive overall. d. are more creative but worse at decision-making.

a

Stimulus control training is most effective when. a. the target behavior is consistently reinforced when it is performed in the presence of the discriminative stimulus. b. a continuous reinforcement schedule is replaced by an intermittent one once the target behavior has reached its desired level or intensity. c. the latency between presentation of the discriminative stimulus and the performance of the target behavior is maximized. d. the reinforcement is consistently withheld following the target behavior.

a

Subjects learn a task while under the influence of a CNS stimulant. Three days later, half of the subjects are tested while under the influence of the drug; the other half are tested while drug-free. Subjects who have been given the drug perform better on the task during testing. This result is predicted by which of the following? a. state dependent memory. b. proactive facilitation. c. drug-dependence effect. d. priming.

a

Test-retest reliability would be the most appropriate method of determining the reliability of a test if the test is designed to assess which of the following. a. general intelligence. b. mood. c. state anxiety. d. reading level.

a

The concept of triangulation is central to general systems theory and can be found in the work of Bowen, Minuchin, Satir, and others. Although these theorists differ somewhat in their definition of triangulation, in general, it is said to be occurring when. a. to reduce tension between them, a two-person system draws in a third party. b. to reduce tension between them, the members of a two-person system focus their attention on a third party. c. to avoid dealing with the conflict between them, a two-person system forms an alliance against a third person. d. to reduce conflict, two members of a three-person system assume a submissive role when interacting with the third dominant member.

a

The original purpose of the classic Hawthorne studies conducted at the Western Electric plant in Chicago in the 1920s and 1930s was to determine. a. the effects of certain environmental factors on work performance. b. the impact of information work group norms on worker productivity. c. the effects of innovative management styles on job satisfaction. d. the relationship between job responsibility and job satisfaction.

a

The presence of which of the following symptoms would suggest a diagnosis of Conduct Disorder rather than a diagnosis of Oppositional Defiant Disorder. a. frequent lying and running away from home. b. low frustration tolerance and temper outbursts. c. drug use. d. onset of symptoms after age 11.

a

The word HEART is projected onto a screen so that HE is projected to the subject's right visual field only while ART is projected to the subject's left visual field only. If the subject in the experiment is a "split brain"patient, you would expect him or her to report seeing. a. HE only. b. ART only. c. ART HE. d. HEART.

a

Undernutrition during the prenatal period and early infancy. a. affects both the size and structure of the brain's cells. b. primarily affects the peripheral (versus central) nervous system. c. increases the body's susceptibility to disease, especially during childhood. d. impairs later cognitive functioning but has little impact on social or emotional development.

a

Unilateral ECT to the right hemisphere would most likely produce. a. predominantly anterograde amnesia involving nonverbal material. b. predominantly anterograde amnesia involving verbal material. c. predominantly retrograde amnesia involving nonverbal material. d. predominantly retrograde amnesia involving verbal material.

a

Various predictors are used in industry for the purpose of personnel selection. When validating these predictors, which of the following is most commonly used as the criterion measure. a. supervisory ratings of job proficiency. b. peer ratings of job proficiency. c. objective measures of production quality and quantity. d. salary level

a

Which of the following is the most commonly used criterion-measure in organizational settings? a. supervisor ratings. b. standardized performance tests. c. quantitative measures of production. d. assessment centers.

a

You administer a test to a sample of young adults with PTSD and, from their scores, calculate a kappa coefficient of .95. This indicates that the test is. a. reliable. b. valid. c. reliable and valid. d. reliable but not valid.

a

"Assessment centers" are most commonly used to. a. hire and promote clerical workers. b. hire and promote managers. c. train semi-skilled and skilled workers. d. evaluate the performance of current employees.

b

10. You are hired by a small company to provide evaluation and counseling services to employees. The company owner tells you that employees routinely sign a waiver of confidentiality when they are hired. In this situation, you should. a. tell the employer that you can give her only very limited and relevant information about an employee even though the employee has signed a waiver. b. clarify the company policy with the employees that you see and determine if they want to continue in counseling. c. provide the employer with information only after you have confirmed that the employee has actually signed the waiver. d. refuse to give the employer any information about the employees that you see.

b

A child with Oppositional Defiant Disorder is least likely to exhibit which of the following. a. frequent arguments and conflicts with his or her parents. b. high degree of physical aggression toward his or her peers. c. consistently blames others for his or her mistakes. d. views his or her problems as due to the unrealistic demands of others.

b

A child would be most likely to do which of the following first. a. walk alone. b. show separation anxiety. c. pretend that a wood cylinder is a cup. d. say her first word.

b

A client presenting memory loss must be carefully evaluated to determine if the loss is due to dissociative amnesia, a medical or substance-induced disorder, or malingering. Clare R., a 46-year-old housewife, is brought to treatment by her husband. Among other symptoms, Clare displays a memory loss for both recent and distant events. Under hypnosis, however, Clare is able to remember incidents she was unable to recall before being hypnotized. Therefore, the therapist can rule out which of the following disorders. a. dissociative amnesia only. b. amnestic disorder only. c. dissociative amnesia and malingering only. d. amnestic disorder and dissociative amnesia only.

b

A leader who can recognize the need for change and create a vision that motivates followers to effectively achieve that change is referred to as a. a. transactional leader. b. transformational leader. c. superleader. d. dreamer.

b

A listener is least likely to change her attitude as the result of a communicator's message if. a. the listener accidentally overhears the message. b. the listener has prior knowledge about the content of the message. c. the communicator is arguing against his or her own best interests. d. there is a moderate discrepancy between the listener's initial position and the communicator's position.

b

A psychologist administers the MMPI-2 to a 36-year old man who has exhibited a number of neurotic symptoms. The man receives a very low K Scale score. On the basis of this score, the psychologist can tentatively conclude that the man. a. tries to present himself in a favorable light. b. is very self-critical. c. is psychopathic. d. may have an organic disorder.

b

According to DSM-FV,for approximately 30 to 40% of the individuals with Mental Retardation seen in clinical settings, no clear cause can be identified. For those individuals for whom the etiology of the disorder is known, heredity is responsible in what percentage of cases? a. less than 1%. b. about 5%. c. about 20%. d. more than 50%.

b

According to Kohlberg, moral development is directly related to. a. changes in cognitive skills. b. changes in cognitive skills and social perspective-taking. c. changes in cognitive skills and socioemotional status. d. sociocultural influences and changes.

b

According to Patterson and his colleagues, aggression in children is linked to the use of coercive discipline by their parents. In turn, the parents' use of coercive discipline is directly related to. a. the parents' misinterpretations of the child's intentions when he or she misbehaves. b. the parents' personality characteristics and the child's temperament. c. the nature of the attachment between the child and the parent. d. the effectiveness of the discipline in stopping the child's misbehavior.

b

According to the James-Lange theory. a. I think, therefore I am. b. I'm trembling, so I must be scared. c. It's noon so I must be hungry. d. That was a great birthday surprise, so I must be happy.

b

After playing the role of a child abuser in a feature film, a well-known and formerly well-liked actor begins receiving a lot of "hate mail." The writers of these letters are exhibiting. a. psychological reactance. b. the fundamental attribution error. c. the availability bias. d. defensive attribution.

b

Androgens and estrogens are secreted by the. a. gonads only. b. gonads and adrenal glands. c. gonads and pituitary gland. d. gonads and pineal gland.

b

During the first session with a client, you learn that he has a history of offenses that you find difficult to deal with and feel, therefore, that you would not like working with him. As an ethical psychologist you should. a. disregard your personal preferences and accept the young man into therapy. b. refer the young man to another professional. c. accept the young man into therapy but seek supervision. d. tell the young man that you cannot accept him into therapy.

b

First-and second-grade children who have been identified as being at risk for academic under achievement are provided with a special after-school program. This is an example of. a. primary prevention. b. secondary prevention. c. tertiary prevention. d. crisis intervention.

b

Following the assessment of a child and a meeting with the child's parents, a school psychologist working for a public school district concludes that child abuse has occurred. The psychologist should. a. make a report to the school principal and let the principal handle the situation. b. make a report to the appropriate child protective agency. c. confront the parents with his suspicions and insist that they seek counseling. d. do nothing as it is not within the realm of his responsibilities and would represent a breach of confidentiality.

b

For a family systems (Bowenian) therapist, when afamily exhibits a high degree of fusion but one member is more differentiated than the others, the likely approach would be to: a. work with the least differentiated family members. b. work with the most differentiated family member. c. work with all family members as a group. d. work with all family members in individual therapy until they reach a similar level of differentiation.

b

Subtracting the mean from each score in a distribution, squaring each result, and then summing the squared results. a. yields the distribution's standard deviation. b. provides a measure of dispersion. c. yields a coefficient of determination d. provides a measure of central tendency

b

Systematic desensitization, a behavioral technique originally developed by Wolpe, is based on which of the following. a. aversive counterconditioning. b. counterconditioning. c. habituation. d. operant extinction.

b

The College Entrance Examination Board has conducted several well-controlled studies to assess the effects of commercial coaching courses on SAT (Scholastic Aptitude Test) performance. Results of these studies have generally found that coaching. a. has no effect on SAT performance b. has a slight (but insignificant) positive effect on SAT performance c. has a significant positive effect on the SAT performance of lower-achieving students only d. has a significant positive effect on the SAT performance of students at all levels of achievement

b

The best way to reduce the likelihood of the development of PTSD following a crisis or other unusual event is. a. stress-inoculation training prior to the event. b. immediate debriefing with opportunities to reprocess the event. c. in vivo or imaginal exposure with response prevention. d. systematic desensitization.

b

To help a depressed client identify the cognitive errors and distortions that are affecting her interpretation of current life events, a practitioner of Beck's cognitive therapy would. a. rely primarily on didactic techniques. b. make use of Socratic questioning. c. rely on verbal persuasion. d. use paradox.

b

When a program to reduce aggression in children is based on the work of Patterson and his colleagues, it will include which of the following? a. providing the child with opportunities for catharsis. b. training parents in child management skills. c. increasing the child's perspective-taking skills. d. individual insight-oriented therapy for each parent.

b

When taking a practice test in preparation for the licensing exam, your performance on the test is least likely to be adversely affected by. a. a loudly ticking clock. b. the stereo that you're too lazy to turn off. c. neighbors arguing in the apartment next door. d. a thunderstorm.

b

When using covert sensitization to treat a sexual fetish, the fetish object acts as a or an. a. unconditioned stimulus. b. conditioned stimulus. c. primary reinforcer. d. secondary reinforcer.

b

42. A patient with which of the following Personality Disorders is most likely to exhibit symptoms that could be erroneously interpreted as signs of depression a. Avoidant Personality Disorder. b.Obsessive-Compulsive Personality Disorder. c. Dependent Personality Disorder. d. Narcissistic Personality Disorder.

c

A client you have been seeing for two months suddenly informs you that this will be his last session. You believe this is not a good time for the client to stop therapy and are concerned about the potential negative consequences of doing so. You should. a. allow the client to quit since he has the right to do so. b. provide the client with appropriate referrals. c. discuss with him the reasons you believe he should continue and recommend that he re-think his decision. d. inform him that you are not responsible for any negative consequence of terminating prematurely.

c

A correlation between X and Y of .60 represents BLANK as much shared variability asa correlation of .30. a. twice. b. three times. c. four times. d. six times.

c

A feminist therapist is least likely to advocate the use of which of the following as a therapeutic technique. a. self-disclosure. b. sex-role analysis. c. diagnosis. d. role-modeling.

c

A mother of a newborn is most likely to say that, based on the nature of her baby's cry, she can tell if her baby is. a. hungry, tired, or uncomfortable. b. hungry, thirsty, or tired. c. hungry, angry, or in pain. d. hungry, lonely, or irritable.

c

A researcher designs a study in which subjects will be hospitalized mental patients. The hospital research committee has approved the project, which involves several treatments, some of which include experimental drugs that may cause harmful side effects. Before beginning the study, the researcher should. a. obtain informed consents from the patients if possible. b. obtain informed consents from the patients' guardians. c. obtain informed consents from the patients' guardians and assent from the patients. d. none of the above are necessary since the study has been approved by the hospital's research committee.

c

A strategic family therapist is most likely to use which of the following therapeutic techniques in dealing with a husband and wife who complain that they are constantly arguing with each other. a. obtain a detailed family history from each partner. b. ask the couple to identify the benefits they obtain from arguing. c. tell the couple to argue for at least two hours each evening. d. work individually with the partner who is most "differentiated".

c

A test developer assesses the reliability of an achievement test using the test-retest method. She obtains a reliability coefficient of .58. The researcher should consider this coefficient to be. a. high for this type of test. b. about average for this type of test. c. low for this type of test. d. high for any type of test.

c

According to Erikson's theory of psychosocial development, middle-aged adults are in which stage of development? a. ego integrity versus despair. b. initiative versus guilt. c. generativity versus stagnation. d. industry versus inferiority.

c

Although a number of longitudinal studies suggest that assessment centers have a high degree of predictive validity, some critics have argued that the high validities reported by these studies are actually the result of. a. a "contrast effect". b. the Hawthorne effect. c. criterion contamination. d. sample size.

c

Among bilingual speakers, those who exhibit "code-switching". a. tend to obtain lower scores on measures of reading comprehension. b. have less trouble switching between the grammatical rules of the majority and minority languages. c. go back and forth between languages during the course of a conversation as a way to better express themselves. d. unconsciously confuse the majority and minority languages while speaking, especially in high-stress situations.

c

Dr. Colombu, a licensed psychologist, is working in a community mental health center. The parents of a 15-year old girl bring their daughter for counseling. The parents will, of course, be paying Dr. Colombu's fee. As an ethical psychologist, Dr. Colombu should. a. caution the girl that, since she is a minor, anything she says may be told to her parents. b. advise the parents that he is required to maintain confidentiality and that he will not be able to advise them of anything the girl says during therapy sessions without her consent. c. clearly establish with the girl and her parents prior to therapy what the policy will be with regard to sharing information with the parents. d. use his discretion with regard to what information to reveal to the girl's parents and what information to keep confidential.

c

Electroconvulsive shock therapy. a. is as or more effective than drug therapy for acute Schizophrenia. b. has been found to be more effective in treating Mania than Depression. c. is considered useful for treating Depression, especially when the Depression involves suicidal preoccupation. d. is no longer used as a treatment for Depression because of its substantial impact on memory processes.

c

Failure to initiate a sequence of responses (a "behavioral chain") is often a problem because. a. the contingencies (reinforcers) have not been consistently applied. b. habituation has made the reward less desirable. c. the final (primary) reinforcer is remote. d. the final consequence is unknown.

c

For women suffering from PMS, the most severe physical and emotional symptoms usually occur. a. during the last days of menses. b. four to ten days prior to menses. c. during the week prior to and the first few days of menses. d. during the first three to four days of menses.

c

Investigations into the relationship between client race or ethnicity and premature termination from therapy have shown that. a. matching minority clients and therapists in terms of race/ethnicity substantially reduces premature dropout rates. b. premature dropout rates among minority clients arc, in fact, due to the confounding effects of socioeconomic status rather than to race or ethnicity. c. minority clients do have higher premature dropout rates, which seem to be attributable to negative attitudes toward the therapist and/or a perception of therapy as nonbeneficial. d. premature dropout rates among minority clients is largely a myth.

c

Of 100 students surveyed on a particular college campus, 50 voted Republican and 50 voted Democratic in the last election. On another college campus, 60 voted Republican and 40 voted Democratic. The correct statistical test to use to determine if the difference in voting preferences at the two colleges is significant is which of the following. a. two-way ANOVA. b. t-test. c. chi-square test. d. Mann-Whitney U test.

c

Presence of which of the following symptoms suggests that the appropriate diagnosis is Alcohol Withdrawal as opposed to Cocaine Withdrawal? a. insomnia. b. dysphoria. c. seizures. d. psychomotor agitation.

c

Research on mnemonic devices suggests that they are. a. superior to rote rehearsal in terms of both short-and long-term retention. b. superior to rote rehearsal and elaboration in terms of both short-and long-term retention. c. superior to rote rehearsal in terms of short-term but not long-term retention. d. superior to rote rehearsal and elaboration in terms of short-term but not long-term retention.

c

Research on the effects of maternal employment suggests that it. a. has positive effects on the cognitive development of all children. b. has negative effects on the cognitive development of all children. c. may have negative effects on the cognitive development of middle-class boys. d. may have negative effects on the cognitive development of middle-class girls.

c

Research suggests that, in comparison to popular children, less popular children tend to be all of the following except. a. less intelligent. b. less physically attractive. c. less friendly and sociable. d. less cooperative.

c

To be consistent with the Federal Educational Rights and Privacy Act (aka the Buckley Amendment), a school psychologist must keep in mind that. a. it is necessary to obtain the signed consent of a student's parents before releasing any information from the student's file to officials of another school where the student is planning to enroll. b. student records, including identifying information, must be released to designated federal and state educational authorities if needed in connection with the evaluation of federally-sponsored educational programs. c. parents have the right not only to inspect and review their children's school records but also to challenge the contents of records. d. all of the above.

c

To evaluate an intervention designed to increase the time a hyperactive child stays on-task when doing schoolwork, the best technique would be. a. situational sampling. b. event recording. c. interval recording. d. sequential analysis.

c

When a psychologist is requested by a court to provide psychological assessment of a defendant for a competency hearing, the psychologist is primarily asked to evaluate. a. the defendant's psychic functioning as it relates to his or her psychological development. b. the defendant's vocational history and probability of rehabilitation. c. the defendant's ability to understand and participate in a court proceeding. d. the psychological status of the defendant at the time of the alleged crime.

c

When administering a standardized 60-item test that has a time limit of 60 minutes, a test administrator accidentally gives the examinees only 50 minutes to complete the test. To get accurate test results, the test administrator should. a. add 10 points to each examinee's score. b. administer the last 10 items to examinees on another day. c. administer an alternate form of the test to examinees. d. do any one of the above.

c

When using Beck's cognitive approach to therapy with a client with Obsessive-Compulsive Disorder. a exposure in vivo or in imagination is not considered beneficial because it produces strong emotions that interfere with cognitive work. b. exposure in vivo or in imagination is conducted only when the symptoms are so severe that they are interfering with cognitive work and must be alleviated quickly. c. exposure in vivo or in imagination is considered useful for eliciting "hot cognitions" related to the disorder so that they are available for cognitive work. d. exposure in imagination is considered more useful than exposure in vivo because it elicits target cognitions without the strong emotions that may interfere with cognitive work.

c

Which of the developmental changes is most likely to occur in males between the ages of 40-45. a. a decrease in work satisfaction. b. significant personality change and emotional disturbance due to the "mid-life" crisis. c. a shift in perspective from "time-since-birth" to "time-left-to-live". d. peaceful acceptance of the impending black doom of the grim reaper.

c

160. Recent research on goal-setting in organizations has shown that all of the following are true except. a. providing feedback on goal achievement is a necessary but not sufficient condition for improved job performance. b. overall, participation in goal setting is not as important as acceptance of goals. c. employees who are told to "do their best" will usually exhibit lower levels of productivity than employees who are given difficult goals. d. participation in goal-setting is more important for educationally-advantaged workers.

d

A 34-year old woman is often very irritable, has trouble controlling her anger, has a history of frequent job changes, and often complains that she is bored. Based on these symptoms, the best diagnosis is. a. Schizoid Personality Disorder. b. Narcissistic Personality Disorder. c. Histrionic Personality Disorder. d. Borderline Personality Disorder.

d

A girl has a punishing father who is in the army reserves. Eventually, the girl becomes extremely anxious whenever she sees a man in uniform. The girl's reaction to uniformed men is an example of. a. higher-order conditioning. b. negative transfer. c. pseudoconditioning. d. stimulus generalization.

d

A person with apraxia has. a. deficits in memory. b. trouble recognizing objects by touch. c. impaired procedural memory. d. difficulty carrying out intentional movements.

d

A split-brain patient would be able to do which of the following? a. say the word "spoon" after it is projected to his left visual field. b. repeat a series of letters that have been whispered in his left ear. c. identify an odor that has been presented to his right nostril. d. identify a spoon with his left hand after an image of a spoon is projected to his left visual field.

d

After speaking at a workshop at a large university, Dr. A., a renowned expert on eating disorders, is approached by a woman who asks him about her daughter who she believes is beginning to exhibit symptoms of anorexia nervosa. Dr. A. gives the woman the phone numbers of three therapists in the area who have experience with this disorder. Six months later, Dr. A. finds out that the woman is planning to file a malpractice suit against him. During that time, her daughter received treatment from one of the therapists on Dr. A.'s list but became much worse and is currently hospitalized. The woman feels that Dr. A. is responsible for not adequately warning her of the potential course of her daughter's disorder. In terms of malpractice. a. Dr. A. is guilty because he had a "duty to warn" the woman about possible complications of anorexia nervosa. b. Dr. A. is guilty because he apparently did not give the woman adequate referrals. c. Dr. A. is not guilty because he gave the woman the names of several therapists. d. Dr. A. is not guilty because he didn't have a professional relationship with the woman or her daughter.

d

An 12-year old who has received a diagnosis of Oppositional Defiant Disorder is LEAST likely to be exhibiting which of the following symptoms. a. swearing. b. low frustration tolerance. c. alcohol use. d. lying and cheating in games.

d

Dr. Werner, a psychologist, is conducting a research study and obtains subjects for his study by placing advertisements in a local newspaper. Dr. Weiner informs potential subjects about all aspects of the study and tells them that they should not agree to participate unless they are willing to stay for its duration (six sessions over a six-week period), since once they sign up, they will not be permitted to withdraw from the study. According to APA's Ethics Code, the proposed arrangement is. a. clearly ethical. b. ethical as long as the withdrawal restriction is included in the informed consents that all subjects must sign. c. ethical as long as the withdrawal restriction has been determined to be crucial to the study. d. unethical.

d

Jung's theory differs from Freud's in all of the following ways except. a. Jung's perspective reflects a more optimistic view of human nature. b. Jung promoted a more active interchange between analyst and patient. c. Jung conceptualized the psyche as continuing to evolve throughout the lifespan. d. Jung replaced the concept of the ego with the concepts of the self and the ideal-self.

d

Research investigating outcome following a traumatic head injury suggests that within 12 to 18 months following the trauma BLANK percentage of total recovery of functions directly related to the activity of the brain can usually be expected. a. 20 to 30%. b. 40 to 50%. c. 60 to 75%. d. 80 to 95%.

d

Research investigating the effects of maternal employment on cognitive development in children has revealed that such employment. a. has beneficial effects for middle-class girls and boys only. b. has beneficial effects for middle-class boys only. c. has statistically insignificant adverse effects on children from all social classes. d. has no substantial effects.

d

Shortly after a school psychologist begins working for a small rural school district, she finds that there are a number of children in a class for retarded children who are emotionally disturbed but who appear to have an average or above average level of intelligence. The county has no class for emotionally disturbed children. The psychologist should. a. report the situation immediately to the psychology ethics committee in her state. b. report the situation immediately to the teacher's standards and practices commission in her state. c. threaten to quit if the children are not placed in a regular classroom. d. evaluate each student and recommend an appropriate action regardless of available options.

d

The Ethics Committee is investigating a complaint against a psychologist and requests, by letter, that you provide the Committee with information about the complainant who is a former client of yours. When you check your records, you realize that you stopped seeing this client nine years ago. As an ethical psychologist, you should. a. respond immediately to the Committee's request by supplying them with the information they have asked for. b. respond immediately to the Committee's request but provide them only with the information you feel is relevant to the complaint. c. respond to the Committee's request by informing them that the information you have about the client is obsolete. d. respond immediately to the Committee's request only after verifying that the client has signed a release of information.

d

The psychoanalyst Adolph Stern provided the first organized clinical description of the borderline patient. Of the ten basic characteristics Stern delineated, which of the following did he consider to be the most primary? a. inhibited aggression. b. fixation. c. emotional dysregulation. d. narcissism.

d

The research suggests that which of the following would be least useful for the treatment of alcoholism? a. social-skills training. b. stress management training. c. antidepressant medication. d. anxiolytic medication.

d

The target of an organizational development (OD) intervention depends, of course, on the nature of the organization's problems. However, most OD interventions share in common an overarching concern with. a. structure. b. function. c. task and technology. d. human/social relationships.

d

There are a number of widely-accepted myths about old age. Which of the following is not a myth? a. Older adults are more difficult and rigid than younger adults. b. Older adults are more similar than younger adults in terms of personality, cognitive skills, interests, etc. c. Older adults have higher rates of depression and anxiety than younger adults. d. Older adults tend to exhibit more stability in personality traits than younger adults.

d

Tolman's demonstration of latent learning suggests that. a. a student studying algebra in September may not learn algebra until he begins to study geometry. b. a student studying algebra will come to a sudden and whole understanding of it. c. a student will not learn algebra unless the proper incentive for learning is provided. d. a student might appear to know little about algebra until he takes an algebra test.

d

When asked to recall an accident you witnessed two weeks ago, which of the following will be involved. a. procedural memory. b. semantic memory. c. representational memory. d. episodic memory.

d

Which of the following is true about the retrograde amnesia that occurs as the result of moderate to severe head injury? a. it is limited to recent long-term memories. b. it is limited to remote long-term memories. c. when it begins to resolve, the most recent memories return first. d. when it begins to resolve, the most remote memories return first.

d

Which one of the following is true regarding treatment for premature ejaculation. a. premature ejaculation is one of the more difficult sexual dysfunctions to treat. b. treatment usually involves temporary abstinence from sexual contact. c. treatment will involve helping the client maintain an erection for at least four minutes. d. treatment will probably include the use of the "stop-start" or "squeeze" technique.

d

With regard to bartering, APA's Ethics Code. a. clearly prohibits bartering of services only. b. clearly prohibits bartering of goods only. c. discourages bartering unless alternative methods of payment for services are not feasible. d. discourages bartering unless it is not clinically contraindicated and is not exploitative.

d

With regard to the effects of age on global job satisfaction, the best conclusion is that these variables have. a. a U-shaped relationship. b. an inverted U-shaped relationship. c. an inverse relationship. d. a positive relationship.

d

Work by Perry and Busey (1977) suggests that highly aggressive children. a. were exposed prenatally to high levels of androgens. b. are less sensitive to physical discomfort than less aggressive children. c. exhibited a disoriented/disengaged attachment pattern as young children. d. show little remorse after hurting another child.

d

Adverse Impact

* 4/5 or 80% rule * percentage of minorities hired must be 4/5 of percentage of non-minorities hired

Ecological Theories of Development

* Bronfrenbrenner

Gordon Allport Trait Theory

* Cardinal Traits - dominate a person's life * Central Traits - general characteristics that form the basic foundations of personality * Secondary Traits - sometimes related to attitudes or preferences and appear only in certain situations

Executive Coaching

* Executive coaching is designed to help facilitate professional and personal development to the point of individual growth, improved performance and contentment. Most important, the coach attempts to stimulate the client's self-discovery by posing powerful questions and/or assigning homework that may take the form of "thought experiments" with written product or "field experiments" which are actions to try in the real world that may result in experiential learning and development of new approaches to situations.

Personal Coaching

* Personal coaching is a process which is designed and defined in a relationship agreement between a client and a coach. It is based on the client's expressed interests, goals and objectives. A professional coach may use inquiry, reflection, requests and discussion to help clients identify personal and/or business and/or relationship goals, and develop action plans intended to achieve those goals. The client takes action, and the coach may assist, but never leads or does more than the client. Professional coaching is not counseling, therapy or consulting.[26] These different skill sets and approaches to change may be adjunct skills and professions.

Constructivist Theories of Development

* Piaget

Levinson

* Seasons of a Man's Life. * time from birth to time till death

Attachment - Ainsworth

* Strange Situation 1. Secure (65%) - infants seek closeness and contact. Moderate distress upon separation and greet mother with enthusiasm when she returns 2. Avoidant (20%) - do not seek closeness with mother, don't cry when she leaves the room and ignore upon return. Associated with aloofness or overstimulation. 3. Ambivalent/ Resistant (10%) - clingy and upset when mother leaves the room. Happy upon return and reestablish contact, but show resistance by avoiding mother's comforting behaviors. Associated with inconsistent and insensitive caregiving. 4. Disorganized/Disoriented - no clear strategy. Exhibit fear and confusion towards mothers. Least secure of attachments. Associated with abuse of the infant or unresolved abuse issues of the caretaker.

Rescorla-Wagner Model

* The Rescorla-Wagner model is a model of classical conditioning in which the animal is theorized to learn from the discrepancy between what is expected to happen and what actually happens * This is a trial-level model in which each stimulus is either present or not present at some point in the trial. * The prediction of the unconditioned stimulus for a trial can be represented as the sum of all the associative strengths for the conditioned stimuli present during the trial * This is the feature of the model that represents a major advance over previous models, and allowed a straightforward explanation of important experimental phenomena such as blocking.

Peripheral Route

* a focus on aspects that are not central to the message

Ganser Syndrome

* a rare dissociative disorder previously classified as a factitious disorder * characterized by nonsensical or wrong answers to questions or doing things incorrectly, other dissociative symptoms such as fugue, amnesia or conversion disorder, often with visual pseudohallucinations and a decreased state of consciousness * sometimes called nonsense syndrome, balderdash syndrome, syndrome of approximate answers, pseudodementia, hysterical pseudodementia or prison psychosis.

Mediator variable

* accounts for the relation b/w the predictor and the criterion. Identifies how or why such effects occur

MRI: Communications Family Therapy

* addresses communication problems * techniques include: teaching, pointing out problematic communication patterns, paradoxical interventions

Consistency Theories

* attitude formation and change are organized by a need to impose structure and order on one's understanding of the environment

Marital Behavioral Therapy

* behavioral analysis of the couple * establish positive reciprocity * teach communication skills (I statements, staying present) * improve problem solving skills * social exchange theory * caring days

Discrete variable

* categorical data - nominal or ordinal

Vygotsky Social Development

* cognitive development results from social interaction * zone of proximal development * scaffolding * reciprocal teaching

Haley: Strategic Family Therapy

* combination of structural (focus on hierarchies) and communication and interactions * normal functioning includes: large behavioral repertoire for problem solving and life-cycle passage, clear rules re: hierarchy * views presenting problem and symptoms as a communicative act within an interactional pattern * treatment focuses on resolving the presenting problem, which is defined by behavioral objectives and criteria (underlying conflicts or patholgy are not addressed) * attempt to upset rigid feedback cycle

Self Perception Theory (Daryl Bem)

* competing theory to consistency theories * people infer attitudes as well as their emotions by observing their own behavior * overjustification hypothesis

Dyslexia

* deep dyslexia - reads a word as one w/similar meaning (coat vs jacket) * surface dyslexia - cannot recognize words, and sounds them out (come vs dome vs home) * phonological dyslexia - cannot read non-words aloud (squilfish) * neglect - misreading of first or last half of word (slap as slit)

Raymond Cattell's 16 Personality Factor Questionnaire

* developed Sixteen Personality Factor Questionnaire * abstractedness, apprehension, dominance, emotional stability, liveliness, openness to change, perfectionism, privateness, reasoning, rule consciousness, self reliance, sensitivity, social boldness, tension, vigilance, warmth

Kohut

* development of narcissism * primary narcissism, healthy narcissism * empathic attunement

Main Effect

* effect of a single independent variable. There can be as many main effects as there are independent variables

Berne: Transactional Analysis

* ego states * transactions * games * strokes * life scripts

Schacter's Two-Factor Theory

* emotion results from info from 2 systems: internal and external * emotion is a function of physiological arousal and cognitive labeling

Cannon-Bard Theory

* emotions and bodily reactions occur at the same time

James-Lange

* emotions result from perceiving bodily reactions or responses * individual perceives the bodily reactions and interprets these as the emotion of few

Beck: Cognitive Therapy

* empirical hypothesis testing as a means of changing beliefs * behavioral assignments / homework * Socratic questioning * collaborative empiricism * automatic thoughts, logical errors, faulty conceptions, self-signals

enzymatic polymorphisms

* enzymes are frequently polymorphic * population differences in genetic information * Polymorphism biology occurs when two or more clearly different phenotypes exist in the same population of a species—in other words, the occurrence of more than one form or morph.

Minuchin: Structural Family Therapy

* family viewed as interrelated system * assessed along dimensions: hierarchy of power, clarity and firmness of boundaries, alliances and splits * therapist seeks to unbalance / reorganize family's structure * triangulation * detouring * stable coalition

Ellis: Rational Emotive Behavior Therapy

* first cognitive approach * emotional disturbances are thought to result from irrational beliefs * A=activating, B=belief, C=consequence, D=disputing intervention, E=effective philosophy, F=(new) feelings * direct instruction, persuasion, logical dispution

Solution-Focused Therapy

* focus on strengths to identify solutions * attempt to set up positive expectations * start with small changes * miracle question * exception question * scaling question

Attachment - Bowlby

* formulated attachment theory 1. Protest 2. Despair 3. Detachment

Congruity Theory

* further extends balance theory by making predictions about which attitudes will change * a person will focus the object toward which he or she already feels the most affinity

Winnicott

* good enough mother * pathology results from adopting false self and abandoning one's true self * transitional object

Bowen: Family Systems Therapy

* healthy families have clearly differentiated family members * balance of intellectual and emotional forces * multigenerational transmission process * goal of treatment is differentiation and analysis of emotional triangles * originated the genogram

Interaction Effect

* how the variables interact. The effect of each variable across the levels of the IV

Attachment - Lorenz

* imprinting w/goslings

Milan Group: Systemic Family Therapy

* includes elements of general systems, cybernetics, strategic theory * treatment includes circular questioning (gathering information as well as introducing new information) - transform thinking from linear and causal to reciprocal and interdependent, and prescription of rituals (alter direction from its current course)

Psychodynamic Family Therapy

* individual maturation in context of family system * freeing members from unconscious patterns of anxiety and projection * clarify communication and honestly admit feelings * marital schism * marital skew

Glasser: Reality Therapy

* influenced by both existentialism and REBT * tx focuses on clarifying client's values and help to evaluate their current behavior and plans in relation to these values

Rehm: Self-Control Model of Depression

* integrate cognitive and behavioral models of depression * importance of reinforcement * depression and its concomitant low rate of behavior as a result of negative self-evaluations, lack of self-reinforcement and high rates of self punishment

Gerstmann's Syndrome

* lesions of left parietal lobe * primary symptoms include: agraphia, acalculia, right-left disorientation, finger agnosia

Primacy Effect

* long gap b/w speech and desired action, first speaker will be remembered best

WHO

* maintains internationally accepted classification systems - esp. ICD

Availability heuristic

* mental shortcut that relies on immediate examples that come to mind. Operates on the notion that if something can be recalled, it must be important.

Moderator variable

* moderates a correlation. Specifes when certain effects will hold. Affects the direction / strength of the correlation

Noam Chomsky

* nativist view of language development * born with innate language acquistion device (LAD)

Karen Horney

* neurosis is a culturally defined construct * develops from feelings of alienation, anxiety and hostility * neurotic trends - moving towards others, moving against others, moving detachedly away from others

Kluver-Bucy Syndrome

* occurs with removal or damage of amygdala * described in monkeys that had undergone bilateral temporal lobectomies * results in placidity, apathy, hypersexuality, hyperphagia, and agnosias

Kelly's Personal Construct Theory

* ordinary people are scientist, forming, testing and revising hypothesis about the world around them * "a person's processes are psychologically channelized by teh ways in which he anticipates events" * as our experiences changes, we revise our expectations *Repertory Grid Technique

Behavioral Confirmation

* people are motivated to confirm the expectations that others have of them

Self Verfication Theory

* people are motivated to confirm their self concept, even if it is negative

Self Enhancement Theory

* people are motivated to think favorably of themselves and behave in ways that cause others to see them favorably as well

Sleeper Effect

* people forget the source of the message, but remember the message

Reactance Theory

* people will not comply with requests or attempts to be persuaded if they feel their freedom is threatened

Margaret Mahler

* process of separation and individuation 1. normal infantile autism 0-1 mo. Unaware of the external world 2. Symbiosis 2-4 mo's. Baby feels that she and mother are one 3. Differentiation 5-10 mo's. Child is able to distinguish self from other objects. Stranger anxiety. 4. Practicing 10-16 mo's. Child discovers ability to physically separate self from mother 5. Rapprochement 16-24 mo's. Increased need for mother to share child's new experiences 6. Object Constancy 2-3 years. Ability to maintain image of mother when not present, and unify the good and bad into whole

Harry Stack Sullivan

* prototaxic (0-7 mos) * parataxic (8-11 mos) * syntaxic (12 mos - 2 years) * IPT developed from his work

Information Processing

* quantitative differences * occur more smoothly and gradually over the life span

Behavioral and Cognitive-Behavioral Family Therapy

* rewarding adaptive behavior * benefits of being part of family outweigh the costs * relationship-related cognitions are seen as the underlying cause of feelings and behaviors

Recency Effect

* short gap b/w speech and desired action, last speaker wil be remembered best

SEC

* socio economic classification

Autokinetic Effect

* tendency for people to experience a stationary point of light to be moving in a darkened room

Differential Validity

* test is more valid for predicting performance of one group, than it is for predicting the performance of another

Unfairness

* when minorities and non-minorities score differently on predictor, but similarly on the criterion

In a single-subject research design, which of the following is the most significant problem? • 1. Autocorrelation. • 2. Multicollinearity. • 3. Regression to the mean. • 4. Practice effects.

1. Autocorrelation. (correct answer) When the same subject is measured repeatedly, the measures demonstrate a high degree of correlation, which is termed autocorrelation. Autocorrelation can either appear to enhance or to decrease the effect of the independent variable. Multicollinearity (Response 2) is a problem in multiple regression analysis when the predictors are highly correlated. Response 3 and 4 are general threats to internal validity and are not relevant here. While single-subject designs do involve repeatedly measuring subjects, the measures are not always tests (e.g., the measure can be frequency of observed behavior); hence, practice effects (Response 4) are not always a problem.

In a single-subject research design, which of the following is the most significant problem? • 1. Autocorrelation. • 2. Multicollinearity. • 3. Regression to the mean. • 4. Practice effects.

1. Autocorrelation. (correct answer, your response) When the same subject is measured repeatedly, the measures demonstrate a high degree of correlation, which is termed autocorrelation. Autocorrelation can either appear to enhance or to decrease the effect of the independent variable. Multicollinearity (Response 2) is a problem in multiple regression analysis when the predictors are highly correlated. Response 3 and 4 are general threats to internal validity and are not relevant here. While single-subject designs do involve repeatedly measuring subjects, the measures are not always tests (e.g., the measure can be frequency of observed behavior); hence, practice effects (Response 4) are not always a problem.

Agranulocytosis is associated with which of the following medications? • 1. Clozapine. • 2. Thorazine. • 3. Phenelzine. • 4. Lithium Carbonate.

1. Clozapine. (correct answer, your response) Agranulocytosis is a potentially lethal side effect associated with Clozapine (Clozaril), a novel antipsychotic. Agranulocytosis is a sudden drop in the granulocyte count, usually occurring within hours to 12 weeks of initial administration, and manifesting as a sore throat and high fever. Thorazine (Chlorpromazine) (Response 2) is a traditional antipsychotic and is not known to bring about agranulocytosis. Phenelzine (Nardil) (Response 3) is an MAOI. MAOIs should not be mixed with foods high in tyramine because they can induce a hypertensive emergency, which requires immediate medical attention. Symptoms of a hypertensive crisis include headache, stiff neck, sweating, nausea, and vomiting. The most severe side effect of Lithium Carbonate (Response 4) is lithium toxicity, which is potentially fatal and is always a medical emergency. Lithium toxicity may initially mimic the flu and can include vomiting, abdominal pain, severe diarrhea, severe tremor, ataxia, coma, seizures, confusion, and irregular heart beat.

Minority Identity Development Model

1. Conformity - prefers the dominant culture 2. Dissonance - begin to appreciate aspects of the minority culture and question values of dominant culture 3. Resistance (Immersion) - endorse minority held views only, reject dominant culture 4. Introspection - deeper analysis of attitudes and feelings. Negative feelings towards dominant culture is draining. Less likely to accept own culture values w/out critical thinking 5. Synergetic articulation & awareness (Integrative awareness) - ability to both appreciate and be critivcal of one's own culture and the dominant culture

The best predictor of treatment outcome among adult substance abusers is: a. age b. ethnicity c. history of criminal behavior d. severity of substance abuse problems

1. D-- Most studies have found that the best predictors of treatment outcome for substance abusers are psychiatric severity and problem severity. Other predictive factors include motivation and coping skills at baseline

What is the most significant problem in using a series of t-tests to analyze a data set? • 1. Experiment-wise error (Type I error). • 2. Difficulty distinguishing between interaction and main effects. • 3. Low power (1 - Beta). • 4. Violating assumptions for the use of parametric tests.

1. Experiment-wise error (Type I error). (correct answer) The advantage of the analysis of variance tests (ANOVA) over multiple t-tests is that they reduce Type I error. Type I error is additive, so the more tests that are run, the larger Type I error becomes. For example, if three t-tests are used, each with alpha set at .05, the overall level of error for the experiment (i.e., experiment-wise error) would be .15 (.05 + .05 + .05). In contrast, if an ANOVA with three groups is used and alpha is set at .05, the overall level of error for the experiment remains at .05. The issue of main and interaction effects (Response 2) applies only to factorial ANOVAs (two-way ANOVAs and above). Response 3 is the opposite of what's true; power increases as alpha increases. Response 4 is not related to the problem of running multiple t-tests.

Fifty subjects are given a boring book to read, and are then asked to say they liked the book when they speak to a confederate posing as a book reviewer. Half of the people (Group A) are paid $5 to tell the lie, and the other half of the people (Group B) are paid $50. What will happen when the subjects are asked to report their true reactions to the book? • 1. Group A will report greater liking for the book than will group B. • 2. Group B will report greater liking for the book than will group A. • 3. Both groups will say they enjoyed the book equally. • 4. Neither group will say that they enjoyed the book.

1. Group A will report greater liking for the book than will group B. (correct answer) This question is about Festinger's cognitive dissonance theory. According to this theory, people experience dissonance when they hold conflicting cognitions, or when they behave in a manner which conflicts with existing cognitions. In order to resolve this unpleasant feeling of dissonance, people change their cognitions. Here, those paid $5 experience dissonance between their attitude (the book is boring) and their behavior (lying for a mere $5); they therefore feel discomfort and change their attitude to reduce the discomfort. By contrast, those paid $50 experience much less dissonance between their attitude (the book is boring) and their behavior (it's worth $50 to lie); they therefore have no incentive to change their cognitions.

Which of the following statements correctly summarizes Cattell's theory of intelligence? • 1. Intelligence consists of a factor with two subtypes, crystallized and fluid intelligence. • 2. Intelligence consists of one general factor, or "g factor." • 3. Intelligence involves processes rather than factors. • 4. There are eight distinct intelligences, including linguistic, logical-mathematical, musical, interpersonal, etc. Feedback: CORRECT! Spearman initially proposed that intelligence is a single factor (Response 2). Cattell went on to propose that this single "g factor" is made up of two subtypes, called crystallized and fluid intelligence. Sternberg's triarchic theory (Response 3) focuses on process rather than product, specifically the thinking processes used to arrive at answers to problems. Sternberg has proposed three aspects of intelligence: the internal components (e.g., those we use to acquire, store, perceive, and remember), the capacity to adapt to environmental changes, and the ability to apply past experience to current novel problems. Gardner (Response 4) has identified eight distinct intelligences, which are linguistic, logical-mathematical, spatial, musical, bodily-kinesthetic, interpersonal, intrapersonal, and naturalistic. Gardner argues that people differ in their profile of intelligence, or their relative strengths and weaknesses in various areas.

1. Intelligence consists of a factor with two subtypes, crystallized and fluid intelligence. (correct answer, your response) Spearman initially proposed that intelligence is a single factor (Response 2). Cattell went on to propose that this single "g factor" is made up of two subtypes, called crystallized and fluid intelligence. Sternberg's triarchic theory (Response 3) focuses on process rather than product, specifically the thinking processes used to arrive at answers to problems. Sternberg has proposed three aspects of intelligence: the internal components (e.g., those we use to acquire, store, perceive, and remember), the capacity to adapt to environmental changes, and the ability to apply past experience to current novel problems. Gardner (Response 4) has identified eight distinct intelligences, which are linguistic, logical-mathematical, spatial, musical, bodily-kinesthetic, interpersonal, intrapersonal, and naturalistic. Gardner argues that people differ in their profile of intelligence, or their relative strengths and weaknesses in various areas.

Which of the following is incorrectly matched? • 1. Melanie Klein: focus on the impact of social relations and cultural factors in determining personality. • 2. Alfred Adler: disturbance is viewed as a result of a faulty life-style involving a struggle for power. • 3. Anna Freud: focus on capacities for integration and adaptation and what is interfering with these processes. • 4. C. G. Jung: views neurosis not as an illness but as striving toward psychological maturity.

1. Melanie Klein: focus on the impact of social relations and cultural factors in determining personality. (correct answer, your response) Melanie Klein is an Object Relations theorist, and treatment focus is on recognizing and modifying the impact of problematic early object-relationships. Social relations are a key aspect of this theory, but cultural factors are not. The focus on both social and cultural factors is more characteristic of the neo-Freudians (e.g., Sullivan, Horney), who view psychological disturbance as the result of faulty learning and as consisting of a characterologically maladaptive style of interacting with the environment. Adlerian therapy deals with faulty life-styles that involve a struggle for power (Response 2) as a means of compensating for feelings of inferiority. A focus on capacities for integration and adaptation (Response 3) is characteristic of Ego Psychology (e.g., Anna Freud, Heinz Hartmann). Response 4 describes Jungian therapy. Jung believed that neurosis represents the struggle of people to free themselves from the interference of the archetypes, interference that otherwise impedes progress toward personality integration and fulfillment of potential.

Of the following, which describes the type of memory that can retain for a second an exact copy of what is seen? • 1. Sensory memory. • 2. Eidetic (photographic) memory. • 3. Echoic memory. • 4. Short-term memory.

1. Sensory memory. (correct answer) Sensory memory stores information as a visual (iconic) or auditory (echoic) image. Although the information is retained in sensory memory for a very brief period of time (up to one second for visual information and up to four seconds for auditory information), this information is retained accurately. Note that this question is asking about what is seen, therefore it is referring to iconic memory, not echoic (Response 3). Response 2 is incorrect because photographic memory refers to the ability to recall information for a substantial period of time, not just one second. Short-term memory (Response 4) typically is not an exact copy and holds 7 + or - 2 bits of information for up to 30 seconds.

A researcher wants to test her model of remarriage. What statistical technique should she use if her model includes multiple pathways involving multiple predictors and multiple criterion variables? • 1. Structural equation modeling. • 2. Hierarchical multiple regression. • 3. MANOVA. • 4. Trend analysis.

1. Structural equation modeling. (correct answer) Structural equation modeling (one type is known as LISREL) enables researchers to make inferences about causation. It can be used to test out many different causal pathways, involving multiple predictor and criterion variables. MANOVA (Response 3) involves one or more independent variables with at least two dependent variables; it is a test of group differences and would be inappropriate for testing a causal model. Similarly, trend analysis (Response 4) is also a test of group differences, used when the outcome data is non-linear. Although hierarchical multiple regressions (Response 2) are used to test theories, they can only include one criterion variable and do not test multiple pathways.

In a split brain patient, the word suitcase is projected in such a manner that suit is seen only in the left visual fields and case is seen only in the right visual fields. If the patient is asked to point with his left hand to what he sees, which one of the following four cards will he point to? • 1. Suit. • 2. Case. • 3. Suitcase. • 4. A blank card, indicating he saw nothing.

1. Suit. (correct answer) In all persons, sensory experiences on the left side of the body (e.g., an object placed in the left hand) are always processed by the right hemisphere, whereas, sensory experiences on the right side of the body (e.g., an image flashed in the right visual fields) are always processed by the left hemisphere. In split brain patients (i.e., patients with their corpus callosum severed), the two hemispheres operate independently, and are unable to communicate, thus eliminating the response "suitcase" (Response 3) which would require integration of the input to the two hemispheres. In this example, "suit" is projected to the left visual fields. This stimulus is processed by the right hemisphere. Since the right hemisphere is also linked to the left hand, the person, when asked to point with the left hand, will point to "suit." If the question had asked what would he say he saw, because language is left hemisphere dominant, "case" which is projected to the right visual fields, would be processed in the left hemisphere, and he would only be able to say he saw the word "case" (Response 2).

When Test A is correlated with Test B, the correlation is greater than Test A's correlation with itself. This would indicate that: • 1. Test A is more valid than it is reliable. • 2. Test A is more reliable than it is valid. • 3. Test A is less valid and reliable than Test B. • 4. neither Test A nor Test B should be used.

1. Test A is more valid than it is reliable. (correct answer, your response) Test A's correlation with itself describes reliability. Test A's correlation with Test B describes criterion-related validity. So, this question means that Test A is more valid than it is reliable. It is counterintuitive that validity can be higher than reliability because we so often hear what sounds like the reverse: reliability is a necessary but not sufficient condition for validity. Nevertheless, the formula for the relationship between validity and reliability states that validity is equal to or less than the square root of the reliability coefficient. The reliability coefficient is almost always a decimal (ranging from 0.0 to 1.0), and the square root of a decimal results in a larger number (e.g., the square root of .25 is .50). Hence, a test with reliability of .25 could have a validity coefficient of .50 and be more valid than it is reliable.

Which one of the following theories of leadership addresses decision-making and the continuum of leader behavior, from autocratic to complete participation? • 1. Vroom and Yetton's normative theory. • 2. House's path-goal theory. • 3. Fiedler's contingency theory. • 4. Graen's vertical dyadic linkage theory.

1. Vroom and Yetton's normative theory. (correct answer, your response) Vroom and Yetton's Normative Model states that there are five styles of leader behavior that can be placed on a continuum, ranging from autocratic (decisions made entirely by the leader) to complete participation (decisions made through consensus). The most effective style of leadership in any given situation depends upon the importance of the decision, the time needed to reach the decision, and the extent to which the decision is accepted by the employees. House's Path-Goal Theory (Response 2) deals with the ways in which leaders can help employees achieve their goals, using one of four styles: directive leadership, supportive leadership, participative leadership, and achievement-oriented leadership. The most effective leadership style is dependent on the subordinates and the situation. According to Fiedler's Contingency Theory (Response 3), the effectiveness of the style of leadership is based on the type of leader and the nature of the situation. A leader is classified as either task or relationship-oriented. A task-oriented leader is one who is most effective in highly favorable or highly unfavorable situations. A relationship-oriented leader is most effective in moderately favorable situations. Graen's leader-member exchange theory (Response 4) focuses on the impact of the leader-subordinate relationship on the leadership process. In this theory, employees are classified as in-group (those who are viewed as competent and motivated), and out-group (considered incompetent and unmotivated). Leadership style is classified as 'supervisory,' which is based on formal authority, and 'leadership' in which the leader exerts influence through persuasion. Leaders use the 'leadership' approach with in-group subordinates, and use the 'supervisory' approach with out-group employees.

Berry's Theory of Acculturation

1. assimilation - do not maintain their cultural identity, but seek regular interaction with other cultures 2. separation - when individuals place value on their original culture and avoid interaction with other cultures 3. integration - people maintain their original cultural identity, while interacting with other cultures 4. marginalization - people do not to maintain their cultural identity and do not seek interaction with people from other cultures

In later life, the person who successfully gets through Erikson's psychosocial crisis of generativity versus stagnation has likely gained: • 1. care. • 2. wisdom. • 3. competence. • 4. purpose.

1. care. (correct answer)

The most common symptoms of Undifferentiated Somatoform Disorder are: • 1. chronic fatigue, loss of appetite, gastrointestinal symptoms, genitourinary symptoms. • 2. headaches and chest pain. • 3. deficits in voluntary motor or sensory function. • 4. multiple somatic complaints beginning before the age of 30 years.

1. chronic fatigue, loss of appetite, gastrointestinal symptoms, genitourinary symptoms. (correct answer, your response) The diagnosis of Undifferentiated Somatoform Disorder is made when there are one or more physical complaints that either cannot be explained by medical findings or are in excess of what would be expected for an established medical condition. The symptoms of this disorder can therefore be quite diverse. However, the DSM-IV lists the most common symptoms as chronic fatigue, loss of appetite, gastrointestinal symptoms, or genitourinary symptoms. Pain symptoms (Response 2) are usually coded as a Pain Disorder. Symptoms that suggest a neurological deficit, such as deficits in voluntary motor or sensory function (Response 3), are part of the diagnostic criteria for Conversion Disorder. Somatization Disorder is characterized by multiple somatic complaints beginning before the age of 30 (Response 4).

When the Health Insurance Portability and Accountability Act (HIPAA) and state law contradict, a practitioner should: • 1. comply with HIPAA. • 2. comply with state law. • 3. make known one's commitment to HIPAA, and take reasonable steps to resolve the conflict. • 4. make known one's commitment to HIPAA, and then comply with state law.

1. comply with HIPAA. (correct answer) HIPAA is federal law. When HIPAA and state law are contradictory, making it impossible to follow both, psychologists must comply with HIPAA. In addition, when HIPAA provides stricter privacy safeguards than state law, psychologists should comply with HIPAA. However, when state law provides stricter safeguards than HIPAA, psychologists should comply with state law (Response 2). In other words, when the issue is that either state law or HIPAA provide stricter privacy protections, psychologists must comply with whichever regulations are the strictest. Making known one's commitment and taking reasonable steps to resolve the conflict is the language that applies to a conflict between ethics and law (ruling out Response 3 & 4).

In a multiple baseline design, treatment is applied: • 1. consecutively. • 2. following a Latin square protocol. • 3. concurrently. • 4. in alternating sequences with baselines.

1. consecutively. (correct answer, your response) Multiple-baseline, a type of single-subject design, is applied either across subjects, situations, or behaviors. Treatment is applied consecutively (or sequentially, or successively). For example, in a multiple baseline across settings design, treatment to reduce head-banging of an autistic child might first be applied in the lab, then in the home, then in the school, etc. The Latin square (Response 2) is used when more than one treatment or condition is consecutively applied to a group of subjects. Treatment is applied in alternating sequence with baseline (Response 4) in the ABAB single-subject design.

A 28-year-old Latina woman enters therapy with you because of her difficulty in finishing community college. She reports that she lives at home and has many conflicts with her parents over whether she should be concentrating on college or starting a family. You should: • 1. explore her attitudes toward family duty and autonomy. • 2. provide empathy for the lack of support she is experiencing. • 3. recognize that individual achievement may be discouraged in her culture. • 4. support her process of individuation.

1. explore her attitudes toward family duty and autonomy. (correct answer) This patient is experiencing conflict between her parents' values and some of her own aspirations. A therapist's best course of action would be to help her more fully understand her own attitudes and feelings about these values without the therapist imposing his or her own values. Providing empathy for her lack of support (Response 2) and supporting the individuation process (Response 4) both implicitly support one side of her conflict and endorse the values of the dominant culture, such as individualism and achievement. Understanding the culture she comes from (Response 3) is important, but does not address the action that the therapist should take. Response 1 is therefore the best response.

Quasi-experimental designs affect: • 1. internal validity. • 2. external validity. • 3. both internal and external validity. • 4. neither internal nor external validity.

1. internal validity. (correct answer) The hallmark of a quasi-experimental design is non-random assignment of subjects. For example, two different treatments designed to improve critical thinking skills are administered to two first grade classes. Since the students are already in pre-existing classrooms, the students are not randomly assigned. Non-random assignment is sometimes referred to as a "selection bias," and it affects internal validity only (Response 1). Internal validity pertains to whether something other than treatment may have caused the results. In this case if one class improves significantly more than another, it would be hard know whether this was actually due to the treatment or due to preexisting differences among the students in the two classrooms. External validity (Response 2) pertains to the generalizability of results. In this case, if significant results were found we would be confident generalizing to other first graders; thus external validity is not threatened by non-random assignment.

The Kappa coefficient would be used to express: • 1. interrater reliability. • 2. test-retest reliability. • 3. parallel forms reliability. • 4. internal consistency reliability.

1. interrater reliability. (correct answer, your response) The Kappa coefficient is a measure of interrater reliability, or the degree of agreement between raters on an instrument that is subjectively scored. Other measures of interrater reliability include percent agreement between raters, Pearson r between scores given by the raters, and Yule's Y. Test-retest reliability (Response 2) is typically expressed by the coefficient of stability. Parallel forms reliability (Response 3) is typically expressed by the coefficient of equivalence. Kuder-Richardson and Cronbach's coefficient alpha are measures of internal consistency reliability (Response 4).

According to Piaget, when a child is not able to see that there is the same amount of water when it is poured from a container of one shape into a container of a different shape, he is in the pre-operational state. This is a consequence of: • 1. irreversibility. • 2. centration. • 3. conservation. • 4. phenomenalistic causality.

1. irreversibility. (correct answer) According to Piaget's Stage Theory, one of the characteristics of the pre-operational stage is irreversibility (Response 1) which refers to the inability to mentally undo something. In this case, the water was poured into a different container in front of the child, who was then unable to think back to the initial stage of the action. Centration (Response 2) is another aspect of the pre-operational stage and refers to the tendency to focus on only one aspect when observing a stimulus. In this situation, centration is also occurring, when the child focuses only on the level of the water. Nevertheless, irreversibility is the better answer because the key difficulty the child experiences in this scenario is the inability to think back to the initial situation. Phenomenalistic causality (Response 4) is also associated with the pre-operational stage and involves a sense of magical thinking where events that co-occur in time are thought to be causally connected. Conservation (Response 3) is the ability to recognize that objects conserve their characteristics regardless of a change in shape or form, a development associated with the concrete operational stage. In this scenario the child is unable to conserve.

The shape of the distribution of percentile scores: • 1. is rectangular. • 2. is leptokurtic. • 3. involves a linear transformation. • 4. is normal.

1. is rectangular. (correct answer, your response) The shape of the distribution of percentile scores is always flat or rectangular. When a distribution of scores is transformed to percentile ranks, regardless of the shape of the original distribution of scores, the distribution of ranks assumes a flat shape. This is because the frequency is identical at each rank. For example, if 500 people take a test and are ranked, there will be five people at each percentile rank. Response 4 is therefore ruled out. A leptokurtic distribution (Response 2) is one that appears more peaked in the middle than the normal curve; in other words, a great number of the scores fall in the middle of the distribution. When a distribution of raw scores is converted to percentile ranks, the transformation is non-linear (Response 3).

A 72-year-old client with long-standing, uncontrolled hypertension recently experienced a stroke, with resulting weakness of her left hand and arm. The client would probably also demonstrate: • 1. left visual field damage and visuospatial deficits. (correct answer) • 2. right visual field damage and visuospatial deficits. (your response) • 3. left visual field damage and language deficits. • 4. right visual field damage and language deficits.

1. left visual field damage and visuospatial deficits. (correct answer) The weakness in the left arm and hand would indicate damage to the right hemisphere, since each side of the body is controlled by the opposite hemisphere. Damage to the right hemisphere would affect functioning on the left side of the body, therefore this person can be expected to have deficits in the left visual field, as well as deficits in visuospatial abilities (the latter function is also controlled by the right hemisphere). Response 2 is only partially correct; the person would have visuospatial deficits, but not right visual field damage. Language deficits (Responses 3 and 4) would be expected with damage to the left hemisphere, not the right.

A 72-year-old client with long-standing, uncontrolled hypertension recently experienced a stroke, with resulting weakness of her left hand and arm. The client would probably also demonstrate: • 1. left visual field damage and visuospatial deficits. • 2. right visual field damage and visuospatial deficits. • 3. left visual field damage and language deficits. • 4. right visual field damage and language deficits.

1. left visual field damage and visuospatial deficits. (correct answer, your response) The weakness in the left arm and hand would indicate damage to the right hemisphere, since each side of the body is controlled by the opposite hemisphere. Damage to the right hemisphere would affect functioning on the left side of the body, therefore this person can be expected to have deficits in the left visual field, as well as deficits in visuospatial abilities (the latter function is also controlled by the right hemisphere). Response 2 is only partially correct; the person would have visuospatial deficits, but not right visual field damage. Language deficits (Responses 3 and 4) would be expected with damage to the left hemisphere, not the right.

Preliminary research suggests that a new test is resulting in too many false negatives. To counteract this problem, the test developer should: • 1. lower the predictor cutoff. • 2. raise the predictor cutoff. • 3. lower the criterion cutoff. • 4. raise the criterion cutoff.

1. lower the predictor cutoff. (correct answer) Decision making theory takes the predictions based on the predictor tests and compares them with the actual criterion outcome. There are four possible outcomes or categories: true positives, false positives, true negatives and false negatives. The classification of "positive" and "negative" is based on whether the person falls above or below the predictor cutoff. The classification of "true" and "false" is based on whether the decision was correct or incorrect. While there are two possible ways to reduce the number of false negatives (by reducing the predictor cutoff and by increasing the criterion cutoff) the preferred way is always to adjust the predictor cutoff. To reduce the number of false positives the preferred way would be to increase the predictor cutoff (Response 2).

When there is greater concern about obtaining false negatives than false positives, one would choose to: • 1. lower the predictor cutoff. • 2. raise the predictor cutoff. • 3. lower the criterion cutoff. • 4. raise the criterion cutoff.

1. lower the predictor cutoff. (correct answer) False negatives occur when a score falls below the predictor cutoff (i.e., is labeled "negative") but above the criterion cutoff; thus, it is a false negative. To remedy the situation, one should lower the predictor cutoff so that it is easier to be labeled a positive. In the real world, we often cannot change criterion cutoffs (Responses 3 & 4). For example, if a lab test for pregnancy gave us false negatives could we change the criterion and make pregnant woman non-pregnant? So, even though raising the criterion cutoff would also reduce false negatives, Response 1 is the better answer.

Tourette's disorder would be the appropriate diagnosis for a person who exhibits: • 1. multiple motor tics and one or more vocal tics. • 2. multiple vocal tics and one or more motor tics. • 3. either one or more motor tics or vocal tics. • 4. both one or more motor and vocal tics.

1. multiple motor tics and one or more vocal tics. (correct answer, your response) The diagnostic criteria for Tourette's disorder include both multiple motor and one or more vocal tics. If either motor tics or vocal tics are present, but not both, the diagnosis is either chronic motor or chronic vocal tic disorder (Response 3). Examples of motor tics include shrugging, touching, squatting, twirling, and eye blinking. Examples of vocal tics include clicks, grunts, snort, barks and copralalia (uttering obscenities).

Criterion contamination refers to: • 1. obtaining a spuriously high validity coefficient because ratings on the criterion are contaminated by knowledge of ratings on a predictor. • 2. obtaining an underestimate of the validity coefficient because criterion ratings are contaminated by knowledge of predictor ratings. • 3. carryover effects of measuring people repeatedly on the same criterion following successive exposure to a series of interventions. • 4. the tendency for criterion-related validity to be contaminated when reliability is unacceptably low.

1. obtaining a spuriously high validity coefficient because ratings on the criterion are contaminated by knowledge of ratings on a predictor. (correct answer Criterion contamination occurs when the rating given on the criterion is affected by knowledge of the score on the predictor. For example, in an attempt to correlate IQ with grades, criterion contamination would occur if the teacher giving the grades (the criterion) had knowledge of his students' IQ scores (the predictor). The teacher might inflate the grades of the high IQ students as compared to the low IQ students, causing a spuriously (falsely) high correlation between IQ and grades. Response 2, then, is the opposite of what actually occurs. Carryover effects (Response 3) are a threat to external validity when repeated measures are used, and are not relevant here. Response 4 makes little sense as worded. Reliability always affects validity and very low reliability would always pose a problem.

As opposed to normative conformity, informational conformity is most likely to: • 1. occur when the task is ambiguous. • 2. occur when minority members perceive themselves as similar to the majority. • 3. result in public compliance but not private acceptance. • 4. take place when the group does not have consensus.

1. occur when the task is ambiguous. (correct answer) A normative influence on conformity occurs when people comply with the majority's standards for acceptable behavior because they want to be liked. By contrast, informational conformity refers to the tendency for people to conform to the majority decision because they want to perceive reality correctly. Both normative and informational conformity are more likely to occur when the majority group has consensus, ruling out Response 4. Normative conformity occurs when members perceive themselves as similar to the majority (Response 2), when they are in cohesive groups, or when there is a fear of rejection. Normative conformity results in public compliance, but not in private acceptance (Response 3). For example, in the Asch line-judging study, 76% of the participants gave at least one "obviously incorrect answer" when asked to give their responses aloud (public compliance), while subjects given the opportunity to write their answers down (private acceptance) did not give incorrect answers. Informational conformity is most likely to occur when the task is difficult and ambiguous (Response 1). It is least likely to occur when the task is relatively simple because then the person feels confident that reality is being perceived accurately.

In psychological testing, some distinctions among types of tests include power versus speed, and aptitude versus ability. When attempting to ascertain the reliability of a speeded test, the best estimate would be: • 1. parallel forms. • 2. split-half. • 3. internal consistency. • 4. interrater reliability.

1. parallel forms. (correct answer) The reliability for speeded tests is best assessed through parallel forms. Because all the items on speeded tests are generally easy, measures of internal consistency (Response 3), of which split-half (Response 2) is one example, will tend to overestimate reliability. Interrater reliability (Response 4) is not appropriate here because speeded tests are objective and interrater reliability should be 100%.

Whenever Johnny whines, his father slaps him. Over time, Johnny whines more and more. Johnny's behavior is under the control of: • 1. positive reinforcement. ( • 2. negative reinforcement. • 3. positive punishment. • 4. negative punishment.

1. positive reinforcement. (correct answer) This is a very difficult question. Johnny's behavior is whining. After Johnny emits the target behavior his father slaps him. The slap is added therefore it is positive not negative, which occurs when something is removed (ruling out Responses 2 & 4). Over time the target behavior (whining) is increasing. Reinforcement always results in an increase in the target behavior (ruling out Responses 3 and 4). In contrast, punishment results in a decrease in target behavior. What makes this question so difficult is that typically slapping is positive punishment (Response 3). The father added a stimulus (i.e., slapping) in the hopes of decreasing Johnny's behavior. In addition, most children would experience the slaps as pain, and most would decrease their whining. However, reinforcement and punishment are always determined by the results in the specific situation, not by the intentions. We can hypothesize that the slaps are a form of attention that is important to Johnny.

An assumption common to both parametric and nonparametric statistics is: • 1. random selection. • 2. random assignment. • 3. normally distributed data. • 4. equivalence of sample value with population parameter.

1. random selection. (correct answer) Random selection (or random sampling) involves ensuring that all members of the population have an equal chance of being sampled, so that our sample is in fact representative of the population that we are hypothesizing about. Both parametric and non-parametric tests require random selection. Random assignment (Response 2) is a key distinguishing factor between a true-experimental and quasi-experimental design. Random assignment is not a requirement of parametric or non-parametric tests. Only parametric tests require normally distributed data (Response 3). Neither test requires equivalence of sample value (e.g., mean, SD) with population parameter (Response 4), although it is hoped that sample values are close to population values.

Troiden Gay and Lesbian Identity Development

1. sensitization - before puberty. Sense of differentness is related more to sexuality than to gender 2. identity confusion - 17/18 years old. Growing sense of homosexual feelings/impulses and fear of being excluded 3. identity assumption - manage social stigma. 19/22 years old. Reduction in social isolation and increased contact with other gays and lesbians 4. commitment - 22/23 years old. Integration of homosexual identity. Make same-sex commitments.a

In developing a test designed to measure the trait of perseverance, to optimize Cronbach's alpha one would ideally develop items that are _______, and utilize subjects that are _________: • 1. similar in content, different in temperament. • 2. different in content, different in temperament. • 3. different in content, similar in temperament. • 4. similar in content, similar in temperament.

1. similar in content, different in temperament. (correct answer) Cronbach's alpha is a measure of internal consistency. It would be highest when the items on the test are consistent or similar in content. The Cronbach's alpha, like all correlation coefficients, is highest when the range of values is unrestricted and wide. So, the more widely varying the subjects are in temperament, the higher would be the Cronbach's alpha. Put another way, reliability is highest when test items are homogenous and test takers are heterogeneous (on the variable being measured).

A researcher would use LISREL, a form of structural equation modeling, in an attempt to: • 1. test a causal model of relationships among variables. • 2. derive a causal model of relationships among variables. • 3. determine whether underlying variables are latent or manifest. • 4. determine the non-linear relationship between multiple IVs and multiple DVs.

1. test a causal model of relationships among variables. (correct answer) LISREL, which stands for LInear Structural RELations, is a computer program that can determine whether a given causal model of relationships among variables is correct. It does not generate a causal model (Response 2). Although LISREL does make distinctions between latent (unobserved) and manifest (observed) variables, it is not used to determine whether a specific variable is latent or manifest. The experimenter must make this determination as part of the causal model. As the name implies, LISREL only analyzes linear relationships (Response 4). It is able, however, to look at direct and indirect effects, as well as both unidirectional and bi-directional paths.

When running an ANOVA, a pooled error term is justified when: • 1. variance is equal. • 2. sample size is unequal. • 3. all cells have the same number of subjects. • 4. the F-ratio is significant.

1. variance is equal. (correct answer) This is a difficult question, requiring an advanced understanding of statistics. It is best to simply know that a pooled error term is used when there is homogeneity of variance (i.e., the variance is about equal for each group). When variance is not equal, a separate error term should be used for each group. A mnemonic here might be: when things are equal, they can be pooled together; when unequal, they must be treated separately. Sample size (the number of subjects in each group) is not the determining factor in the use of a pooled error term (ruling out Responses 2 and 3). The significance of the F-ratio is determined after the ANOVA is run; logically then, this cannot be the basis of determining whether or not to pool the error term in running the ANOVA.

101. According to Janet Helms' (1995) White Racial Identity Development Model, the final stage of racial identity development is: a. autonomy b. disintegration c. reintegration d. immersion-emersion

101. A-- According to Helms, autonomy is the final stage of racial identity development. Autonomy is characterized by a nonracist White identity based on a realistic understanding of the strengths and weaknesses of White culture. At this stage, Whites value and seek out cross-racial interactions.

102. According to DSM-IV, patients who meet some but not all of the criteria for a particular diagnosis can still be assigned that diagnosis. This reflects the fact that: a. DSM-IV relies on a dimensional rather than a categorical approach to diagnosis. b. DSM-IV relies on a categorical rather than a dimensional approach to diagnosis. c. DSM-IV combines aspects of categorical and dimensional diagnosis. d. DSM-IV's approach to diagnostic classification is not valid.

102. B-- DSM-IV uses a categorical approach to diagnosis. This means that mental disorders are divided into types based on criteria sets with defining features. An alternative to the categorical approach would be a dimensional approach, which would involve quantifying patients' symptoms based on their severity. Some have suggested that the categorical approach is inappropriate for diagnosis of mental disorders because it misleadingly suggests that the categories of disorders and individuals within the same category are homogeneous in all relevant ways. To provide some degree of compensation for these limitations, DSM-IV diagnoses often include a polythetic criteria set, which means that, for a diagnosis to be made, a person may present with some but not all of the diagnostic criteria. In addition, DSM-IV reminds the clinician to include other information besides diagnoses in evaluation reports.

103. A week before a controversial episode of a television sitcom is to be shown, a prominent Christian fundamentalist group calls for a sponsor boycott of the show and urges people not to watch it. The group's actions are highly publicized and the group's leader appears on a different talk show every day for the week before the show debuts to discuss the reasons why people shouldn't watch the show. The show's episode turns out to be the most watched episode in its history. This situation illustrates the phenomenon of a. psychological reactance. b. cognitive dissonance. c. overjustification. d. overcompensation.

103. A-- Psychological reactance refers to the phenomenon whereby pressure to behave in a particular way causes individuals to behave in the opposite manner. Reactance occurs when a person perceives that his or her freedom of choice or sense of control is being threatened by attempted influence. In this question, it's possible that many people watch the show due to psychological reactance. They feel that their freedom of choice is threatened by the fundamentalist group, and they react by doing the opposite of what the group wants them to do.

104. You are seeing a family consisting of a mother, a father, and a daughter in therapy. Every time the mother says something, the father and the daughter argue with whatever she says. The behavior of the father and the daughter can be best described by the term a. triangulation. b. fusion. c. coalition. d. joining.

104. C-- All of these choices sound good, so to answer the question, you have to understand what each of these terms mean in the language of family therapy, and also not miss the fact that the question is asking about the behavior of the father and the daughter (as opposed to the behavior of the father and the mother). In family therapy, the term coalition refers to an alliance of two family members against a third. According to Minuchin, dysfunctional families are sometimes characterized by stable coalitions of a parent and a child against the other parent. The other terms in the question don't apply to the behavior of the father and the daughter. Triangulation refers to a situation in which another family member is brought into a conflict that actually exists only between two members, such as when two parents constantly try to get their child to take their side in a conflict the parents are having with each other. Fusion, a term used by Bowen, refers to an inability to separate intellectual from emotional functioning, or an inability to separate one's own thoughts and feelings from those of other family members. And joining is a technique described by Minuchin that involves adopting the family's affective style in order to help establish a working relationship, gain greater understanding of the family structure, and gain more leverage in bringing about family change.

105. A study is conducted to determine the effectiveness of 3 different reading programs on reading comprehension. The participants are 5th grade students who are divided into 3 levels based on their past reading comprehension (below average, average, and above average). Results from a factorial ANOVA indicate that there are significant main effects of each variable and a significant interaction effect. Based on these results, one could conclude that: a. each of the reading programs is equally effective for students at every reading level b. only one of the reading programs is effective for students at every reading level c. the reading programs are only effective for students at a particular reading level d. the most effective reading program is dependent on the student's reading level

105. D-- A factorial ANOVA is used when a study has more than one independent variable. Factorial designs also allow for the assessment of both main effects (the effects of each independent variable considered individually) and interaction effects (the effects of each variable at the different levels of the other variable). The study described in this question has two "significant main effects" for the independent variables: type of reading program and past level of reading comprehension. And a "significant interaction effect" means that the effects of the different reading programs varied significantly for students at different reading levels. For example, "Reading Program A" may have been highly effective for above average students, moderately effective for average students, yet ineffective for below average students. On the other hand, "Reading Program B" may have been only effective for below average students, while "Reading Program C" may not have been effective for any students.

106. Latane called this a social disease. It occurred when high-level employees were assessed on a combined effort. It turned out they produced less than when they were working individually. It's called: a. Social loafing b. Group think c. Social polarization d. Social facilitation

106. A-- Social loafing or Latane's "social disease" is the discovery that in regard to work, individual output declines when people are working together as a group. However, social loafing does not occur under all conditions. It is reduced or eliminated when participants believe that their individual contributions are identifiable or uniquely necessary for the group to succeed. (B) "groupthink" is the tendency for a group to make an irrational or impulsive decision in order to reach consensus. (C) "Group polarization" refers to the tendency of individuals who start with a similar view to end with a more extreme position after group discussion.

107. You are starting a stop-smoking group and tell a newspaper reporter who smokes that he can participate in the group for free if he'll write an article about it for the local newspaper. This arrangement is a. unethical. b. ethical as long as you don't tell the reporter what to write about. c. ethical as long as you check to make sure that information about the group is accurate. d. ethical as long as the confidentiality of the group participants is not violated.

107. A-- This issue is addressed in Standard 5.02(b) of the Ethics Code, which states that "Psychologists do not compensate employees of press, radio, television, or other communication media in return for publicity in a news item."

108. Differences in communication style can act as a barrier in cross-cultural counseling. For example, some cultures have a high-context style, while others are more low-context. Which of the following is true about high-context communication and culture? a. The communication found in high-context cultures reflects an emphasis on rules of law and procedures. b. People belonging to high-context cultures rely heavily on nonverbal cues when communicating with others. c. People in high-context cultures make extensive use of elaborative codes in their speech. d. High-context communication changes rapidly and easily.

108. B-- The notions of high- and low-context communication were introduced by Edward T. Hall, the author of The Silent Language (1959) and The Hidden Dimension (1969). In the United States, the communication style of the Anglo majority is low-context, but many racial/ethnic minorities (e.g., Latino, Asian, African-American) make use of a high-context style. High-context communication is based on nonverbal language. Low-context communication on the other hand is based upon the verbal part of messages.

109. According to the theory underlying Gestalt therapy, introjection involves: a. viewing other people's emotions as one's own b. failing to develop a healthy "social interest" c. blaming oneself for the problems of others d. interpreting the values of the larger society as one's own

109. D-- Introjection involves absorbing the values or behaviors of others, including the larger society, without really understanding or assimilating those values or behaviors. Introjection represents a disturbance in the boundary between self and others -- the person does or believes things that are not reflective of a self that is clearly distinct from others in the environment. Choice A is incorrect because it is primarily values, not emotions, that are absorbed from the environment in Gestalt therapy's view of introjection.

By the time an individual diagnosed with __________________ reaches middle age, particularly the fourth decade of life, the symptoms often become less evident or remit to no longer meeting all the diagnostic criteria. a. Histrionic Personality Disorder b. Avoidant Personality Disorder c. Antisocial Personality Disorder d. Schizotypal Personality Disorder

11. C-- The symptoms of Antisocial Personality Disorder (APD), particularly criminal behaviors, often become less evident as an individual grows older (DSM-TR-IV, p. 704). APD has a chronic course and while some symptoms like criminal behaviors may decrease, other symptoms such as difficulties with interpersonal relationships may persist.

110. Delinquency in adolescence is most associated with: a. low income families and low IQ score. b. a mother who works outside the home and an unemployed father. c. weak parental supervision and parental rejection. d. harsh physical punishment and unreasonable rules.

110. C-- The research is far from consistent on this issue. However, lack of parental supervision is frequently found as a correlate of delinquency along with a hostile relationship between parent and adolescent and parental rejection. Low SES, low IQ score, single-parent homes, and harsh discipline have weaker relationships with delinquency.

111. The best initial strategy for teaching complex motor skills that require speed and accuracy to be successfully performed is to: a. emphasize accuracy over speed b. emphasize speed over accuracy c. emphasize accuracy and speed equally d. emphasize an alternation between speed and accuracy

111. B-- Research on speed and accuracy in learning complex motor skills suggests the best approach is to emphasize speed of performance initially although, to a certain degree, the optimal approach depends on the specific skill. (See: Engelhorn, R. (1997). Speed and accuracy in the learning of a complex motor skills, Perceptual and Motor Skills, 85, 1011-1017.)

112. To reduce a child's aggressive behavior, the best approach is to: a. punish the aggression consistently and harshly. b. use differential reinforcement of alternative behaviors. c. identify the consequences of the behavior and alternatives to it. d. have the child vent his or her aggressive feelings by hitting a pillow.

112. C-- There are a variety of approaches to the treatment of aggression in children; the cognitive-behavioral approach is probably most common. In children who are older or developmentally advanced enough to understand, cognitive approaches tend to focus on helping the person accurately interpret external cues, so that he or she does not respond inappropriately with aggression. In younger children, the goal is often to identify the child's goals, the negative consequences of using aggression to meet those goals, and alternatives to aggression. The other choices are not as likely to be helpful: consistent discipline is good, but contrary to choice A, overly harsh discipline can serve to increase the child's aggression. Reinforcement for alternative behaviors is not a proven method of directly addressing aggression. And the notion that aggression is something that can be released through venting it on a safe target (or, by the way, watching it on TV) has not been supported -- apparently, engaging in aggressive behavior only leads to more of the same.

113. The recurrence of hallucinations long after intoxication is most likely to result from the use of: a. Mescaline b. PCP c. Methamphetamine d. Cocaine

113. A-- All of the drugs listed in this question can cause hallucinations during intoxication. However, hallucinations, especially after discontinued use ("flashbacks") are most likely the result of a true hallucinogen. Of the drugs listed, only mescaline is clearly classified as a hallucinogen. PCP (phencyclidine) is sometimes classified as a hallucinogen; other times it's considered a stimulant or pain killer, and it is less likely to cause flashbacks. The DSM-IV-TR diagnosis "Hallucinogen Persisting Perception Disorder" involves the reexperiencing of hallucinations following cessation of use of a hallucinogen. In contrast, "Substance-Induced Psychotic Disorder" can be caused by intoxication or withdrawal from many different substances but the symptoms must occur within a month of intoxication or withdrawal and the person must not have insight that the hallucinations (or delusions) are substance induced.

114. Consuming foods containing tyramine while taking a monoamine oxidase inhibitor would most likely result in: a. depression b. hypotension c. hypertension d. no adverse reactions

114. C-- Eating foods containing tyramine while taking MAOIs is likely to cause a hypertensive crisis, that is, a severe rise in blood pressure. The person may also experience headache, diaphoresis (perspiration), and palpitations. On rare occasions the combination has caused cardiac failure and cerebral hemorrhage. Some of the foods that must be avoided while taking MAOIs include: beer and wine, aged cheese, beef or chicken liver, orange pulp, pickled or smoked meats, packaged soups, and yeast vitamin supplements.

115. Which of the following is not true about members of self-directed work teams? a. Each member of a self-directed team has unique, specialized works skills to contribute to the group product. b. Members plan the group's work processes and set the group's work goals. c. Members may be authorized to hire their own replacements. d. Members of self-directed work teams tend to be absent more than members of traditional work groups.

115. A-- Self-directed work teams are self-directed; i.e., they determine their own goals, plan their own work processes, and may even hire their own replacements. A distinguishing characteristic of self-directed work teams is that members are generalists (versus specialists), and each member has (or learns) a broad range of skills. A possible downside of self-directed work teams is that they are associated with higher absenteeism than more traditional work groups.

116. The theory that proposes convergent and divergent thinking as dimensions of intelligence is associated with: a. Galton b. Thurstone c. Guilford d. Cattell

116. C-- J.P. Guilford identified 120 elements using factor analysis that he proposed in sum comprise intelligence. Convergent thinking is the ability to group or analyze divergent ideas usually leading to a unifying concept or single solution. Divergent thinking is the ability to generate creative, new ideas or to elaborate or branch off from traditional approaches, such as in brainstorming or "thinking out of the box." Galton (a.) postulated that intelligence is an inherited trait distributed normally across the population. Thurstone (b.) applied his method of factor analysis to intelligence leading to his proposed theory of Primary Mental Abilities (that individuals possess varying degrees of sub-components of intelligence). Cattell's (d.) theory distinguished between fluid and crystallized intelligence.

117. Following a stroke or head trauma, which of the following memory functions is most likely to be affected? a. iconic memory b. long-term store c. memory for newly learned information d. verbal memory

117. C-- Questions like this can be frustrating because it is really impossible to make blanket generalizations. When the brain is compromised by a medical problem (or by substance use), the nature of impairment always depends on the location in the brain of damage and the extent of damage. However, if memory is impaired, the ability to retain newly learned information is most likely to be affected. Impairment in other types of memory, such as long-term episodic memory or verbal (semantic) memory, usually indicates more severe memory damage in which impairment in learning new information is also present.

118. Which of the following disorders is most likely to respond to pharmacotherapy alone? a. Bipolar I Disorder. b. Major Depressive Disorder c. Obsessive-Compulsive Disorder. d. Anorexia Nervosa.

118. A-- Pharmacotherapy (specifically, Lithium therapy in most cases) is the treatment of choice for manic-depressive illnesses such as Bipolar I Disorder. Pharmacotherapy may be supplemented with adjunctive psychotherapy (e.g., to provide support and coping skills). However, psychotherapy is not considered that useful in treating the core symptoms of Bipolar I Disorder. By contrast, although the disorders in the other choices are commonly treated with medication, they also respond to psychotherapy.

119. Journal reviewers who show strong bias against manuscripts that report results contrary to their theoretical perspective are demonstrating: a. self-serving bias b. confirmatory bias c. fundamental attribution bias d. self verification theory

119. B--"Confirmatory bias" or "confirmation bias" is the tendency to seek, interpret, and create information that verifies our existing beliefs. Self-serving bias (A) is the tendency to attribute one's successes to internal factors and one's failures to external factors. The fundamental attribution bias or error (C) refers to the tendency to overestimate dispositional (personality) factors and underestimate situational factors in explaining a person's behavior. Self verification theory (D) proposes that people seek confirmation of their self-concept

The term "group polarization" refers to the tendency of groups to make decisions that are a. more risky than those that might be made by individual members. b. more conservative than those that might be made by individual members. c. more risky or more conservative than those that might be made by individual members. d. more illogical than those that might be made by individual members.

12. C-- A group's decisions tend to be more extreme (in one direction or the other) than those that would be made by individuals in the group acting alone. This phenomenon is referred to as group polarization. One explanation for group polarization is that group members are more willing to support extreme decisions because, as group members, they won't have to take as much personal responsibility for their decisions as they would if they were acting alone.

120. A researcher studying the relationship between age and job satisfaction collects data for a group of young, middle-aged, and older workers. The researcher can expect to find that a. the young people are most satisfied with their jobs. b. the middle-aged people are most satisfied with their jobs. c. the older people are most satisfied with their jobs. d. there is no difference between the three groups in terms of their job satisfaction.

120. C-- One of the most stable findings in job satisfaction research is that age is positively correlated with satisfaction. The older the employee, the higher the level of satisfaction he or she tends to report.

121. If a man and a woman are each heterozygous for phenylketonuria (PKU), the probability that their next child will have PKU is: a. 0% b. 25% c. 50% d. 100%

121. B-- Knowing that PKU is caused by two autosomal (i.e., not sex-linked) recessive genes would have helped you answer this question correctly. Because it is caused by recessive genes, a child would need to receive the PKU gene from both parents in order to develop the disease. Since both parents are heterozygous they each carry one dominant gene and one recessive gene for PKU, but do not have the disease themselves. Thus, the probability that the father will contribute the PKU gene is 50% and the probability that the mother will contribute the gene is also 50%. Fifty percent times 50% is 25% (0.5 x 0.5 = 0.25). People born with PKU lack the enzyme needed to digest the amino acid phenylalanine, which undigested causes severe brain damage. Most to all of the PKU symptoms can be prevented by early detection and adherence to a diet low in phenylalanine during the first 9 years of life.

122. Although conclusions will vary across different studies, which of the following statements is most supported by the overall body of research on therapy outcome for African-American and Caucasian patients? a. There is no significant difference in outcome between African-American and Caucasian patients. b. African-American patients have better outcomes overall than Caucasian patients. c. Caucasian patients have better outcomes overall than African-American patients. d. Treatment outcomes for African-American patients are better but only when the therapist is also African-American.

122. A-- Questions like these are difficult to answer, because research in this area is contradictory, and the issue is not resolved. Research has clearly identified a number of variables that potentially interact with race in influencing therapy outcome. For instance, African-American patients tend to have poorer outcomes when working with therapists who are insensitive to or unknowledgeable about racial or cultural issues. There are also studies that show that African-Americans are more likely to terminate therapy prematurely than Caucasians, and even a few studies which show they are likely to have poorer outcomes. However, the bulk of the literature and thinking on this issue supports the notion that race, in and of itself, is not a good predictor of therapy outcome.

123. Stimulus A is paired with stimulus B. Stimulus B is then paired with stimulus C until stimulus C elicits the same response that was elicited by stimulus A. This is an example of: a. chaining b. shaping c. secondary reinforcement d. higher-order conditioning

123. D-- Higher-order conditioning is a classical conditioning procedure in which two stimuli (CS and US) are paired until the conditioned stimulus (CS) produces the conditioned response (CR) and then the CS (which is now referred to as a US) is paired with another CS to elicit the same response. All of the other choices in this question are operant conditioning terms. Operant conditioning involves behaviors and the consequences that follow, rather than the pairing of stimuli. Chaining (A) is the operant procedure that enables complex behaviors to develop through reinforcement of a sequence of simple behaviors. That is, Behavior A is followed by a reinforcer, which serves as a discriminative stimulus for Behavior B, which is followed by a reinforcer, and so on. Shaping (B) involves reinforcing successive approximations of a desired behavior.

124. Recent longitudinal studies investigating the relationship between menopause and psychological well-being have suggested that: a. women experiencing menopause are more likely to report significant depression than pre-menopausal or post-menopausal women. b. post-menopausal women are likely to have higher general levels of psychological well-being than either pre-menopausal women or women in the midst of menopause. c. pre-menopausal are likely to have higher general levels of psychological well-being than either post-menopausal women or women in the midst of menopause. d. there is no evidence that menopause is linked to either depression or psychological well-being.

124. D-- Choice D describes the results of a longitudinal study published in 1994. Though the study did not concern itself with short-term mood swings that may be caused by the menopausal transition, it did suggest that menopause does not have any effect on long-term psychological functioning.

125. Transactional leaders who wait for mistakes to be brought to their attention before taking corrective action are engaging in: a. corrective transactions b. contingent rewarding c. passive management-by-exception d. active management-by-exception

125. C-- Transactional leadership, in its simplest form, is leadership by contingent reinforcement. The leaders' rewards, promises and/or threats of disciplinary actions or punishments motivate the followers. In constructive transactions, the leader may participate in discussing what is to be done in exchange for implicit or explicit rewards and the allocation of desired resources, or they give out assignments, negotiate or contract with followers. The actions of the leader are contingent on whether the followers do what they have been "contracted" to do. Leaders that take corrective action with negative feedback or reprimand only after followers' mistakes have been called to their attention are engaging in passive management-by-exception. In corrective transactions (a.) or contingent rewarding (b.), leaders engage in active management-by-exception (d.) which means they monitor followers' performance and correct their mistakes if and when they occur.

127. You see a substance abuser who has been referred to therapy as a condition of probation. The man tells you that he is seeking therapy only to avoid jail time and that he does not think he needs therapy. Regarding informed consent in this situation, which of the following statements is true? a. You don't have to get informed consent because the judge ordered treatment. b. You don't have to obtain informed consent because convicted criminals don't have the right to refuse treatment. c. You don't have to obtain informed consent because there's no way that consent can truly be informed in this situation. d. You cannot treat the patient unless you obtain his informed consent.

127. D-- In working with court-referred patients, psychologists must obtain informed consent from the patients before proceeding. This may seem ridiculous, since many court-referred clients face serious consequences if they don't comply with the court's recommendation to seek therapy. However, in court-referred cases, the client could, theoretically at least, decline treatment and choose jail time or other consequences instead. By contrast, in court-ordered evaluation or treatment, there is no need to get informed consent because the client is mandated by law to participate. However, in such cases, psychologists have other obligations, such as informing the client of the limits to confidentiality.

128. A person's reactions to stress are most negative when the stress is unexpected. Moreover, a person's reactions are most affected by a. his or her actual capacity to cope with the stress. b. his or her beliefs regarding his or her ability to cope with the stress. c. the total number of stressors. d. the actual availability of adequate social support.

128. B-- It is a person's perception of control or ability to cope that determines his or her response to stress. When a person believes he or she has no control over the situation or does not have the necessary coping mechanisms, stress will have a more detrimental effect.

129. Which of the following is an example of assimilation? a. Arranging flowers in a vase b. Changing your clothes to please your spouse c. Calling the daisies and ferns arranged in a vase "posies" d. Putting on a coat in the winter

129. C-- Assimilation is when a child incorporates and interprets new information in terms of his or her existing schema.

The Solomon four-group design is: a. a quasi-experimental design b. used to analyze the difference scores among four different treatment groups c. used to reduce practice effects d. used to evaluate the effects of pretesting

13. D-- The Solomon four-group design is a true experimental design used to evaluate the effects of pretesting, since some groups are pretested and others are not

130. A three-year-old sees a tree with droopy leaves and tells her father that the tree is "tired." This is an example of: a. magical thinking b. animistic thinking c. egocentricity d. centration

130. B-- All of the responses are factors in Piaget's preoperational stage. Animistic thinking is believing that objects, like the tree with droopy leaves, have lifelike qualities such as thoughts and feelings. Magical thinking is believing one has control over objects or events. Egocentricity is believing that others experience the world in the same way and centration refers to the tendency to focus on one detail while neglecting others

A MANOVA is used to statistically analyze data when: a. a study includes two or more independent variables b. a study includes two or more dependent variables c. there are more than two levels of a single independent variable d. a study includes at least one independent variable that is a between-groups variable and another independent variable that is a within-subjects variable

14. B-- A MANOVA (multivariate analysis of variance) is used to analyze the effects of one or more independent variables on two or more dependent variables that are each measured on an interval or ratio scale. A factorial ANOVA (a.) is used to analyze data when a factorial design, which includes two or more independent variables, is used and the dependent variable is measured on an interval or ratio scale. A one-way ANOVA (c.) is used when a study has one independent variable and more than two independent groups. The split-plot (mixed) ANOVA (d.) is the appropriate technique when at least one independent variable is a between-groups variable and another independent variable is a within-subjects variable.

The Larry P. v. Riles case is important because it: a. defined the relationship between standard intellectual assessments and placement in Mental Retardation classes. b. was the first time a court addressed bias in job selection in private industry. c. relates to how much information must be revealed to research participants. d. limited certain restraint of trade clauses in the APA Ethics Code, such as using patient testimonials.

15. A-- This was a California case related to the use of standardized intelligence tests for assessing children for placement in special classes for the mentally retarded. It's one of several famous cases and rulings which questioned the validity of using these tests for this purpose. The ruling, which continues to be challenged, banned the use of IQ tests in the placement of minority children in special education classes.

A new professor is teaching her first class on feminist theory. What is she required to do? a. Present a balanced view of feminist theory vs. other theories b. Concentrate solely on feminist theory; it is a given that there are other views of psychology c. Use it as a forum for presenting her dissertation findings d. Present other theories, but explain why feminist theory is the most appropriate paradigm for women

16. A-- While B and D could both possibly be an option, (A) is more in line with Standard 6.03 (a) which deals with objectivity: "When engaged in teaching or training, psychologists present psychological information accurately and with a reasonable degree of objectivity."

A psychologist sees clients at a facility where fees are capitated. What does this mean? a. providers are paid a fee each time a service is performed b. clients are billed according to their ability to play c. clients are required to meet a deductible before their insurance will pay d. providers receive a fixed dollar amount over a specific period of time to cover the service needs of a fixed number of clients

18. D-- The term "capitation" refers to a fixed amount of money paid per person, not by the visit or procedure. Managed care companies usually express capitation in terms of cost per member per month. As stated in response "D," providers are paid a specific dollar amount, for a specific time period, to cover the service needs of a specific number of people. If a provider exceeds his or her capitated payment, he or she may not be able to cover his or her costs and, consequently, may limit his or her services.

Postpartum depression occurs in about _____ of new mothers. • 1. 0 - 3%. • 2. 10 - 15%. • 3. 30 - 50%. • 4. 50 - 80%.

2. 10 - 15%. (correct answer) Postpartum depression, which can continue for months if untreated, occurs in about 10% to 15% of new mothers. Postpartum depression involves severe anxiety, irritability, insomnia, disinterest in the infant, guilt, suicidal ideation, poor concentration, as well as weepiness and sadness. In contrast, postpartum blues is relatively short-lived and tends to be widespread, occurring in 50% - 80% (Response 4) of new mothers. Postpartum blues involve crying, irritability, poor sleep, weeping and a sense of emotional vulnerability. Typically, postpartum blues begin in the first week after birth, and last no longer than a few weeks after birth. Postpartum psychosis occurs in less than 1% of new mothers (Response 1).

If the WAIS-IV has a standard error of measurement of five points on full scale IQ (FSIQ), when a person obtains an FSIQ of 105, what is the likelihood that the true score falls between 100 and 110? • 1. 1 in 2. • 2. 2 in 3. • 3. 3 in 4. • 4. 4 in 5.

2. 2 in 3. (correct answer) This question may seem tricky at first because it asks about confidence intervals in a novel way. The question asks about the confidence interval created by plus or minus one standard error of measurement (i.e., between 100 and 110). We know that on a normal curve, one standard error of measurement provides the confidence interval of 68%. We can say that there is a 68% chance that the person's true score falls between 100 and 110. A 68% chance is very close to saying a "2 out of 3" chance (2/3 is 66.67%).

A test is given to a group of students. The student with the highest score obtained a score of 92 on the test. If the test's standard error of measurement is 4, and its mean is 86, what are the chances that her true score is between 88 and 96? • 1. 3 out of 4. • 2. 2 out of 3. • 3. 1 out of 2. • 4. 4 out of 5.

2. 2 out of 3. (correct answer) In order to come up with the right answer, you need to know that this question deals with the standard error of measurement and how it can be used to determine the confidence band for true scores. Here, the standard error of measurement for the test is 4. At one standard error, there is a 68% likelihood that this person's obtained score is 92 + 4, or ranges from 88 to 96. 68% is roughly equivalent to odds of 2 out of 3. Note that the mean of 86 is extraneous information for this question.

Two predictor tests are developed. The first has a validity coefficient of .5, and the second a validity coefficient of .7. The amount of shared variability with the criterion is approximately how many times greater in the second test as compared with the first? • 1. 1.5. • 2. 2.0. • 3. 3.0. • 4. 4.0.

2. 2.0. (correct answer, your response) To calculate the amount of shared or explained variability with a validity coefficient (or any Pearson r correlation), you must square the correlation coefficient. The first validity coefficient is .5, and .52 = .25. The second validity coefficient is .7, and .72 = .49. This question is asking how many times greater .49 is than .25, which is approximately 2.0 times greater.

Dementia due to Head Trauma a. is usually progressive in cases of moderate to severe trauma. b. is usually progressive only in cases of repeated head trauma. c. unlike other forms of Dementia, does not usually involve disturbances in executive functions. d. unlike other forms of Dementia, is associated more with deficits in executive functions than with memory impairment.

2. B-- As described in DSM-IV, Dementia due to Head Trauma is usually nonprogressive when it is the result of a single head trauma. If it is progressive, this usually indicates a superimposed condition (e.g., hydrocephalus).

You are asked to evaluate a 15-year-old boy who exhibits poor concentration, impulsivity, and increased psychomotor activity. In addition, he has begun to engage in risky behaviors and exhibits hypersexual behavior. Based on this information, the most likely diagnosis is: • 1. Attention-Deficit/Hyperactivity Disorder. • 2. Bipolar Disorder. • 3. Normal puberty. • 4. Impulse-Control Disorder.

2. Bipolar Disorder. (correct answer) This question is a bit difficult because the boy isn't clearly exhibiting the elevated mood associated with a manic episode. Nevertheless, he exhibits several symptoms that suggest at least a hypomanic episode: distractibility ("poor concentration"), psychomotor agitation ("increased psychomotor activity"), excessive involvement in pleasurable activities that have a high potential for painful consequences ("risky behaviors," "hypersexual behavior"). The recent onset of the risky and hypersexual behavior makes ADHD (Response 1) an unlikely diagnosis. This boy's symptoms are more wide-ranging than those of an Impulse-Control Disorder (Response 4), which are narrowly focused on "the failure to resist an impulse, drive, or temptation to perform an act that is harmful to the person or others." The symptoms are also more extreme than those of normal puberty (Response 3).

According to the James-Lange theory of emotions, what are peripheral factors? • 1. Factors in the physical environment. • 2. Bodily sensations. • 3. The opinions of others. • 4. Past emotions.

2. Bodily sensations. (correct answer)

According to Herzberg, improvements in which of the following are likely to increase satisfaction? • 1. Peer relationships. • 2. Content. • 3. Supervision. • 4. Pay.

2. Content. (correct answer)

Which of the following is not likely to decrease the validity coefficient? • 1. Restricted range of predictor values. • 2. Criterion contamination. • 3. Revalidation on a new sample. • 4. Heteroscedasticity.

2. Criterion contamination. (correct answer) Criterion contamination occurs when the criterion is subjectively rated, and raters know subjects' scores on the predictors, prior to criterion ratings. Criterion contamination typically results in an inflated, or spuriously high, criterion-related validity coefficient. For example, if an employer knows how workers scored on a predictor test, and then is asked to rate the workers in terms of competence (criterion), knowledge of predictor test scores may influence criterion rating, thus artificially inflating the degree of correlation. All the other responses do actually reduce validity coefficients. A restricted range of values (Response 1) always lowers correlation coefficients. Revalidation on a new sample (Response 3) typically results in shrinkage, or a reduction in the validity coefficient. Heteroscedasticity (Response 4) violates one of the assumptions underlying correlation and consequently, reduces the value of correlation coefficients.

Which of the following is not likely to decrease the validity coefficient? • 1. Restricted range of predictor values. • 2. Criterion contamination. • 3. Revalidation on a new sample. • 4. Heteroscedasticity.

2. Criterion contamination. (correct answer) Criterion contamination occurs when the criterion is subjectively rated, and raters know subjects' scores on the predictors, prior to criterion ratings. Criterion contamination typically results in an inflated, or spuriously high, criterion-related validity coefficient. For example, if an employer knows how workers scored on a predictor test, and then is asked to rate the workers in terms of competence (criterion), knowledge of predictor test scores may influence criterion rating, thus artificially inflating the degree of correlation. All the other responses do actually reduce validity coefficients. A restricted range of values (Response 1) always lowers correlation coefficients. Revalidation on a new sample (Response 3) typically results in shrinkage, or a reduction in the validity coefficient. Heteroscedasticity (Response 4) violates one of the assumptions underlying correlation and consequently, reduces the value of correlation coefficients.

The phenomenon of "coaction" or social facilitation is most likely to occur in which of the following situations? • 1. More than one person witnesses a crime, however, one person is designated as in charge. • 2. Cyclists make better times racing against each other as compared to racing against the clock. • 3. Licensure candidates perform better on the exam while in a large room with other candidates as compared to taking the exam on their own. • 4. Individual productivity increases when group members' behaviors are observed by the supervisor.

2. Cyclists make better times racing against each other as compared to racing against the clock. (correct answer) Social facilitation refers to the combined effects of coaction (the presence of others working alongside an individual) and audience (the presence of a passive observer). Social facilitation enhances the performance of simple, well-learned behaviors (such as cycling in Response 2) but actually impairs performance on complex, newly-learned behaviors (such as taking the licensing exam in Response 3). The phenomenon of a supervisor's presence increasing productivity (Response 4) relates to social loafing (i.e., the tendency of people in groups to produce less than if they were working individually -- a tendency that can be reduced by having a supervisor watch). Interestingly, the origins of the concept of social facilitation actually come from the psychologist Triplett observing the speeds of bicyclists.

Which of Erikson's stages best corresponds to the stage in Freud's theory that involves a cessation in psychosexual development? • 1. Initiative vs. guilt. • 2. Industry vs. inferiority. • 3. Autonomy vs. shame and doubt. • 4. Identity vs. role confusion.

2. Industry vs. inferiority. (correct answer) Freud described the latency stage, between the ages of six to twelve, as a time of sexual quiescence. This stage corresponds to Erikson's stage of industry vs. inferiority. Initiative vs. guilt (Response 1) corresponds to Freud's phallic stage, ages three to six. Autonomy vs. shame and doubt (Response 3) corresponds to the anal stage, ages one to three. Identity vs. role confusion (Response 4) corresponds to the genital stage, ages to twelve to eighteen

As validity approaches the value of 1.0, what happens to the standard error of the estimate? • 1. It approaches 1.0. • 2. It approaches 0.0. • 3. It approaches the standard deviation of the predictor. • 4. It approaches the standard deviation of the criterion.

2. It approaches 0.0. (correct answer) When a test's validity approaches 1.0, it means that the test is close to being perfectly valid, or being a perfect predictor. In this case, there would be almost no error in estimating, thus the standard error of the estimate would approach 0.0. The formula for the standard error of estimate is Sest = SDy√ 1-rxy2. The range of the standard error of estimate is from a minimum value of 0.0 to a maximum value of SDy. When a test is a perfect predictor (validity = 1.0), the standard error of estimate is 0.0, or, in other words, the test has no error in prediction. Conversely, when a test has no ability to predict (validity = 0.0), the value of the standard error of estimate is equal to the standard deviation of the criterion (SDy) (Response 4).

What is R²? • 1. It is derived by the line of best fit. • 2. It represents the amount of variability shared by the tests. • 3. It is a measure of multicollinearity. • 4. It is a measure of item difficulty.

2. It represents the amount of variability shared by the tests. (correct answer, your response) R², the coefficient of multiple determination, provides an index of the amount of variability in the criterion (y) accounted for by the predictor variables (x's). Response 1 describes the regression equation. Multicollinearity (Response 3) is a problem that can be associated with multiple regression, and occurs when the predictors (x's) are highly correlated with one another. The item difficulty level (Response 4) is the proportion of people that get any given item correct. Item difficulty can range in value from .01 (an extremely difficult item) to .99 (a very easy item), and is symbolized with the letter p. Keep in mind that when using computerized adaptive testing, the p value (item difficulty) for each item is used to generate item-characteristic curves, a plot of the relationship between item performance and total score.

What is R²? • 1. It is derived by the line of best fit. • 2. It represents the amount of variability shared by the tests. • 3. It is a measure of multicollinearity. • 4. It is a measure of item difficulty.

2. It represents the amount of variability shared by the tests. (correct answer, your response) R², the coefficient of multiple determination, provides an index of the amount of variability in the criterion (y) accounted for by the predictor variables (x's). Response 1 describes the regression equation. Multicollinearity (Response 3) is a problem that can be associated with multiple regression, and occurs when the predictors (x's) are highly correlated with one another. The item difficulty level (Response 4) is the proportion of people that get any given item correct. Item difficulty can range in value from .01 (an extremely difficult item) to .99 (a very easy item), and is symbolized with the letter p. Keep in mind that when using computerized adaptive testing, the p value (item difficulty) for each item is used to generate item-characteristic curves, a plot of the relationship between item performance and total score.

A 14-year-old ignores rules, loses his temper, and is angry and vindictive. What would be the most appropriate diagnosis? • 1. Conduct Disorder. • 2. Oppositional Defiant Disorder. • 3. Adolescent Antisocial Behavior. • 4. Disruptive Behavior Disorder.

2. Oppositional Defiant Disorder. (correct answer) Oppositional defiant disorder is diagnosed in the presence of a recurrent pattern of negativistic, defiant and disobedient behaviors toward authority figures. Deliberate defiance or disregard of rules, frequent loss of temper, anger and vindictiveness, and frequent arguments with adults are all symptoms of oppositional defiant disorder. Conduct disorder (Response 1) may include these symptoms, however, the essential feature of conduct disorder is behavior that violates the basic rights of others and societal norms, such as aggression toward people and animals, destruction of property, or theft. Adolescent antisocial behavior (Response 3) is diagnosed in the presence of antisocial behavior that is not due to a mental disorder. In this situation, however, the symptoms presented meet the criteria for oppositional defiant disorder. Disruptive behavior disorder NOS (Response 4) would be diagnosed if the behaviors did not meet the criteria for oppositional defiant disorder or conduct disorder.

Which of the following theories did Donald Super apply to career development? • 1. Self-esteem theory. • 2. Self-concept theory. • 3. Social learning theory. • 4. Two-factor theory.

2. Self-concept theory. (correct answer) Because this question is probably unfamiliar to many of us, a process of elimination can be a useful approach. Although Response 4 could be stretched to apply, two-factor theory is a theory of job motivation, not career development. Social learning theory (Response 3) is a widely-discussed theory, and chances are that if it had been applied to career development, you would have heard about it. Self-concept is a broader term, which can encompass self-esteem, and is the correct answer here.

According to Cross's Black Racial Identity Development Model what is the hallmark of the Internalization stage? • 1. The individual is no longer conflicted and identifies as just another human being. • 2. The individual experiences his or her Black identity as positive, while tolerating and respecting differences in Whites. • 3. The individual experiences an increased desire to become more connected to a Black identity. • 4. The individual idealizes Black culture.

2. The individual experiences his or her Black identity as positive, while tolerating and respecting differences in Whites. (correct answer) According to Cross, Black Racial Identity Development consists of several stages. In the first stage, Preencounter, the individual depends on White society for approval, and the person's has a negative attitude toward Black racial identity. The person is preoccupied with how to be perceived as "just another human being" (Response 1). While resolution of conflict, the first half of Response 1, accurately describes the Internalization stage, the second half of Response 1 spoils the answer. In the second stage, Encounter, the individual experiences confusion about the meaning and significance of race, and experiences an increased desire to become more connected to a Black identity (Response 3). In the third stage, Immersion-Emersion, the individual idealizes and is absorbed in the Black experience and rejects the White world (Response 4). In the fourth stage, Internalization, the individual recognizes that both Blacks and Whites have strengths and weaknesses, and views his or her Black identity as positive, while respecting differences in Whites (Response 2).

The time required to complete a mechanical test was recorded. About 25% of the people took the maximum allowed amount of time. What would be the best measure of central tendency? • 1. The mean. • 2. The median. • 3. The mode. • 4. The mean, median, and mode would be equally good measures.

2. The median. (correct answer)

A patient requests that you list "Major Depressive Disorder" as her diagnosis on her insurance form because her insurance company does not cover her diagnosis of "Adjustment Disorder with Depressed Mood." She states that otherwise she will not be able to attend treatment. If you agree to her request, you are: • 1. acting ethically, if she has experienced depressed mood most of the day, nearly every day, for at least two weeks. • 2. acting unethically. • 3. acting ethically, as you are furthering the "best interests" of the client. • 4. acting ethically, if you believe this woman is at risk if she does not receive treatment.

2. acting unethically. (correct answer) Deliberately providing an inaccurate diagnosis to an insurance company is insurance fraud; it is both illegal and unethical. Standard 6.06 of the APA Ethics Code (2002) specifically mandates "accuracy to payors and funding sources." Even if the goal is to secure continued treatment of someone in need (Responses 3 & 4), fraud is still considered unethical. A more ethical course of action would be to reduce fees, provide treatment pro bono, or connect the client with other available community resources. To meet the criteria for a diagnosis of Major Depressive Disorder, at least five symptoms are required (ruling out Response 1).

You find yourself extremely attracted to one of your clients and fantasize about having a relationship. You find out that your client is extremely attracted to you as well. You are able to rationalize to yourself that in this particular situation having a relationship would not hurt, but rather would benefit the client as well as you. This situation best exemplifies a(n): • 1. approach-approach conflict. • 2. approach-avoidance conflict. • 3. avoidance-avoidance conflict. • 4. win-win situation.

2. approach-avoidance conflict. (correct answer) In this situation, you want to have a relationship with the client ("approach") but you (one hopes) recognize the ethical problems and consequences ("avoidance"). In other words, you are both drawn to and repelled by the same possibility, hence it is an "approach-avoidance" situation. "Approach-approach" (Response 1) conflicts occur when you are choosing between two equally attractive options. "Avoidance-avoidance" conflicts occur when you are forced to choose between two negative options (e.g., most presidential elections). A "win-win" situation (Response 4) is usually used as a term in negotiation and describes the situation in which both parties to the negotiation can benefit by the same thing (e.g., you get a lot of money for a car you no longer want and the buyer gets a good price on it).

According to the theory of schematic memory, someone robbed while at a restaurant would most likely have the best recall for: • 1. things commonly associated with eating in a restaurant. • 2. aspects of the event related to the robbery. • 3. the most emotionally charged aspects of the event. • 4. events during and following the robbery.

2. aspects of the event related to the robbery. (correct answer) As the name implies, the theory of schematic memory posits that memory for events is structured according to mental schemas. The schemas affect what information is selected to be remembered. Memories for events can also be altered and elaborated upon in order to fit the schema (e.g., the appearance of a robber is remembered as more menacing than it was actually experienced at the time). Here, the essential schema would be the robbery, not the ordinary experience of eating in a restaurant (Response 1). This theory does not concern the emotionally charged nature of memories (Response 3), nor does it suggest that one best remembers that which occurs during and following a significant event (Response 4).

According to Beck's cognitive theory of depression: • 1. cognitive distortions only occur during depression. • 2. cognitive distortions are gradually made. • 3. only depressed individuals have cognitive distortions about relationships. • 4. depressed individuals manifest the depressive triad of distortions: guilt, anxiety, and hopelessness.

2. cognitive distortions are gradually made. (correct answer) Even if you don't know the correct answer to this question, you may be able to arrive at it through a process of elimination. Cognitive distortions do not occur only during depression; they are present in other disorders and, to some extent, in non-depressed individuals (Responses 1 and 3). The depressive triad includes a negative view of the self, of the world, and of the future (Response 4). Eliminating these responses leaves Response 2 as the correct answer. Response 2 implies that cognitive distortions are developed over time and after a series of experiences.

A test developer of EPPP practice tests is told by licensure candidates that the content of the practice tests does not accurately reflect the types of questions that appear on the licensing exam itself, and that these tests therefore lack validity. The test developer retorts that scores on the practice tests do tend to reflect scores on the EPPP. The test developer is arguing for: • 1. content validity. • 2. concurrent validity. • 3. construct validity. • 4. face validity.

2. concurrent validity. (correct answer) The test developer is arguing that his practice tests (the predictor) are correlated with a criterion measure (the licensing exam) that is concurrent, hence we are looking at concurrent validity. The developer does not retort: "Yes, the items are the same as on the licensing exam." If he did, he would then be talking about the content of the test or content validity (Response 1). Nor does he address the issue of how well his practice tests (or the licensing exam for that matter) measure a hypothesized underlying trait, such as minimum competence. If he did, he would be talking about construct validity (Response 3). Finally, the candidates' complaints reflect a problem with face validity (Response 4) in that the test does not appear valid but the test developer's response does not address the issue of the appearance of validity.

You are a psychologist who specializes in working with children and adolescents. You are referred a 16-year-old boy who is diagnosed with an Autism Spectrum Disorder (ASD). While you studied extensively about ASDs in graduate school, you have not treated a client with this disorder. To behave ethically you: • 1. could treat this adolescent while securing consultation. • 2. could treat this adolescent while securing supervision. • 3. could treat this adolescent while securing additional training. • 4. must refer this adolescent out.

2. could treat this adolescent while securing supervision. (correct answer) The question states you specialize in working with children and adolescents, and you have studied extensively about ASDs in graduate school. However, you do not have specific experience treating this disorder. Given your general experience with children and adolescents, and your training in ASDs, you could see this client while securing supervision (e.g., weekly meetings to discuss this case, course of treatment, etc.). Consultation (Response 1) is typically a one-time meeting, which would be inadequate here. Additional training is not needed (Response 3) since you already have adequate training. Given your background, while you might choose to refer the client out, it is not something you must do (ruling out Response 4).

A researcher is interested in the relationship between morality and criminal behavior among Christians. The researcher measures morality based on rate of church attendance, and finds that many of the non churchgoers have no criminal record. The best explanation for this finding is: • 1. criterion contamination. • 2. criterion deficiency. • 3. criterion relevance. • 4. criterion-related validity.

2. criterion deficiency. (correct answer)

A PET scan of the brain of someone with schizophrenia would show: • 1. increased metabolic rates in the frontal lobes. • 2. decreased metabolic rates in the frontal lobes. • 3. increased metabolic rates in the temporal lobes. • 4. decreased metabolic rates in the temporal lobes.

2. decreased metabolic rates in the frontal lobes. (correct answer) PET scans of brains of persons with schizophrenia show decreased activity in the frontal lobes. If you consider the deficits involved in this disorder, such as difficulty with executive functioning, poor attention and concentration, concrete thinking, poor judgment and insight, you will recognize that these are functions of the frontal lobes. Therefore, PET scans would show that this area is less active in schizophrenia.

If a test is made more difficult, this will have the effect of: • 1. decreasing the true positives and decreasing the true negatives. • 2. decreasing the false positives and increasing the true negatives. • 3. increasing the true positives and decreasing false negatives. • 4. increasing the false positives and increasing the false negatives.

2. decreasing the false positives and increasing the true negatives. (correct answer) In order to answer this question, you need to know that making the test more difficult would have the same effect as raising the cutoff on the predictor test. When the predictor cutoff is raised, both the number of false positives and the number of true positives are decreased. The number of true negatives and the number of false negatives are increased.

Empirical criterion-keying is a means for: • 1. empirically confirming that a subject's responses on a test are valid. • 2. developing a scoring key in terms of external standards. • 3. interpreting a test score in terms of predicted performance on an external criteria. • 4. establishing criterion-related validity.

2. developing a scoring key in terms of external standards. (correct answer)

In using an empirical criterion keying approach to construct a personality inventory, a researcher would select items that: • 1. have a high correlation with each other and a low correlation with outside criteria. • 2. discriminate among various criterion groups. • 3. have high criterion-related validity coefficients. • 4. provide empirical support for the personality traits measured.

2. discriminate among various criterion groups. (correct answer) Empirical criterion keying was most notably used in the development of the original MMPI. It involves choosing items based on their ability to discriminate between different criterion groups. For example, items were included in the schizophrenia scale of the MMPI if people with schizophrenia responded positively to those items while people without schizophrenia responded negatively to them.

An intern complies with a supervisor's request because the supervisor has received a degree from a university with a good reputation and completed several years of postgraduate training at a well-respected institute. In this example, the supervisor has: • 1. legitimate power. • 2. expert power. • 3. referent power. • 4. incremental power.

2. expert power. (correct answer) Psychologists have identified five types of power that can affect a subordinate's willingness to comply with a supervisor's request. Expert power is based on the perception that the supervisor has expertise. The education and training of the supervisor demonstrate expertise. Legitimate power (Response 1) is based on the hierarchy of the organization (e.g., complying with the supervisor because they are in the position of supervisor). Referent power (Response 3) is based on a sense of liking for and identification with the supervisor, about which nothing is mentioned in this question. Incremental power (Response 4) is the combination of expert and referent power. It is cited as the most common reason for subordinates complying with supervisors' requests.

All of the following treatments contain elements of counterconditioning except: • 1. assertiveness training. • 2. flooding with response prevention. • 3. sensate focus. • 4. aversion therapy.

2. flooding with response prevention. (correct answer) Counterconditioning is based on the principle of reciprocal inhibition which states that two incompatible responses cannot be experienced at the same time, but rather that the stronger response will dominate or inhibit the weaker. The focus of treatment is to weaken a maladaptive response by strengthening an incompatible or antagonistic response. In assertiveness training (Response 1), assertiveness is used as the incompatible response (e.g., with shyness or chronic anger). In sensate focus (Response 3), non-pressured sensual pleasure is incompatible with performance anxiety. Aversion therapy (Response 4) uses responses to aversive stimuli (e.g., electric shock, nausea-inducing drugs) to inhibit a previously conditioned response of pleasure (e.g., to pictures of prepubescent girls or to the first sip of a drink). In flooding with response prevention (Response 2), no incompatible experience is being provided. The person is merely being "flooded" with a feared stimulus while being prevented from escaping.

A person with AIDS dementia complex (ADC) would most likely exhibit: • 1. cognitive deficits without motor symptoms. • 2. forgetfulness, slowness, weakness, apathy and withdrawal. • 3. difficulty with attention and concentration, and uncontrollable motor movements. • 4. personality changes, lack of coordination, and hemiparesis.

2. forgetfulness, slowness, weakness, apathy and withdrawal. (correct answer)

A test of cognitive reasoning is administered to 240 third grade students, and a reliability coefficient of .83 is obtained. If the same test were to be administered to 2nd, 3rd, and 4th grade students (80 students per grade), the new reliability coefficient would most likely be: • 1. in agreement with the original coefficient obtained. • 2. higher than the original coefficient obtained. • 3. lower than the original coefficient obtained. • 4. either higher or lower than the original coefficient, but probably not identical to it

2. higher than the original coefficient obtained. (correct answer) This question is asking what will happen to the reliability coefficient when you increase the heterogeneity of your sample, which has the effect of decreasing the restriction in the range of scores. Having heterogeneous subjects or a less restricted range, increases the reliability coefficient, and conversely homogeneous subjects or a more restricted range, decreases reliability. In this case, going from 3rd graders only to 2nd, 3rd, and 4th graders will make the sample more heterogeneous and increase the reliability coefficient. Also keep in mind that while sample size affects reliability, in this scenario, sample size remained the same. The initial sample was 240 3rd graders, and the new sample was 80 each of 2nd, 3rd, and 4th graders (3 x 80 = 240).

Single subject designs involve an approach that can be described as: • 1. ipsative. • 2. idiographic. • 3. normative. • 4. nomothetic.

2. idiographic. (correct answer)

Increased stress is associated with all of the following except: • 1. increased blood cortisol levels. • 2. increased blood flow and temperature in extremities. • 3. increased blood cholesterol. • 4. decreased immune function.

2. increased blood flow and temperature in extremities. (correct answer) Under stress, blood flow to the extremities (such as the fingers, toes, and nose) is reduced and consequently temperature decreases in these areas. An example is how the body responds to freezing temperatures: in an attempt to conserve warmth, the body increases blood flow to the vital organs and withdraws it from the outer areas. For this reason, frostbite typically begins with toes or fingers. The association between finger temperature and stress is the basis for mood rings, temperature stress cards, and temperature biofeedback. All the other responses (1, 3, & 4) are true of stress reactions: blood cortisol (i.e., epinephrine, norepinephrine) and cholesterol levels increase and immune function decreases.

According to Deci, the effects of extrinsic rewards on intrinsic motivation is such that: • 1. extrinsic rewards decrease intrinsic motivation. • 2. intrinsic motivation may not be decreased when the extrinsic reward is based on competency. • 3. intrinsic motivation may not be decreased when the extrinsic reward serves only a controlling function. • 4. there is no conclusive evidence with regard to the effects of extrinsic rewards on intrinsic motivation.

2. intrinsic motivation may not be decreased when the extrinsic reward is based on competency. (correct answer) Again, you may not have ever heard of Deci, however, you might have been able to apply logic and the process of elimination to this question. Research indicates that extrinsic rewards can sometimes decrease intrinsic motivation (ruling out Response 1 which is too absolute). In the classic study, children were given artist's paper and felt-tipped pens. One group was promised a "Good Player" certificate for their drawings; the second group received an unexpected reward when they were done; and the third group was not promised and did not receive a reward. A few weeks later, the same children were allowed to draw during free-play time. The group that was promised and received the reward showed significantly less interest in drawing than the other two groups, indicating that an extrinsic reward lowered intrinsic motivation. However, sometimes extrinsic rewards can actually increase intrinsic motivation. For example, receiving a raise at work can work as positive feedback about one's performance and thereby increase intrinsic motivation. Deci points out that it depends on whether the rewards are used to control (in which case intrinsic motivation decreases) or inform (in which case intrinsic motivation increases).

On a computer-administered test, the items automatically change, getting easier or harder, depending on whether the subject gets the current item correct or incorrect. This type of testing is a development of: • 1. classical test theory. • 2. item response theory. • 3. ipsative testing. • 4. nomothetic designs.

2. item response theory. (correct answer) Computer-adaptive testing (CAT) involves tests which are individually tailored to the specific test-taker's ability. This method is based on item response theory, according to which every test item represents a certain degree of a latent trait. For example, scoring correctly on a very difficult statistics question indicates a high degree of statistics knowledge (i.e., latent trait), whereas scoring correctly on a very easy statistics question indicates less statistics knowledge (i.e., less of the latent trait). Classical test theory (Response 1) is primarily concerned with issues of reliability. In this theory, every score is understood as partially accurate (true score variability, or reliability) and partially inaccurate (error score variability). On ipsative tests (Response 3), a subject is only compared to himself or herself (i.e., Betty likes gardening more than car mechanics). Many vocational interest tests are ipsative. Ipsative tests are contrasted to normative tests in which the person is compared to the group (i.e., Betty likes gardening more than most other people). Nomothetic (Response 4) designs involved looking at groups. Nomothetic designs are contrasted to idiographic designs, in which one or a few subjects are studied intensively.

In comparing job enrichment to job enlargement: • 1. both would likely satisfy motivator needs. • 2. job enrichment involves more responsibility while job enlargement increases variety. • 3. job enrichment increases variety of tasks, while job enlargement increases decision-making. • 4. both are thought to impact satisfaction and performance.

2. job enrichment involves more responsibility while job enlargement increases variety. (correct answer)

Beta-blockers, such as Inderal (propranolol), are commonly prescribed for blood pressure and heart problems. All of the following are common side effects except: • 1. nightmares. • 2. loss of appetite. • 3. tiredness. • 4. impotence.

2. loss of appetite. (correct answer) Although beta-blockers are generally well tolerated, they can cause a variety of side effects. They can cause sexual dysfunction, with up to 10% of male patients developing impotence (Response 4) on certain beta-blockers. They can also cause dizziness, drowsiness (Response 3), shortness of breath, angina, cold hands and feet, difficulty sleeping, and nightmares (Response 1). Less common side effects include depression, anxiety, and thought disturbances.

The association between two variables, when each variable's association with another variable has been removed, is known as: • 1. analysis of covariance. • 2. partial correlation. • 3. semi-partial correlation. • 4. coefficient of determination.

2. partial correlation. (correct answer) This is a tricky question that would be difficult to guess correctly. A partial correlation is the correlation ("association") between two variables when the association between a third variable and each of the two original variables has been partialed out ("removed"). In a semi-partial correlation (Response 3), the association with the third variable is partialed out for only one of the two initial variables. Analysis of Covariance or ANCOVA (Response 1) uses a similar process, but ANCOVAs, which are a variant of ANOVAs, examine the difference between groups whereas this question calls for a measure of association or correlation. Finally, the coefficient of determination (Response 4) is the term for the proportion of variance shared by two variables and is the square of the correlation coefficient.

People are most likely to accurately remember a sentence if they are required to use their: • 1. sensory memory. • 2. primary memory. • 3. working memory. • 4. long-term memory.

2. primary memory. (correct answer, your People are most likely to accurately repeat sentences if the delay is less than 30 seconds; in other words, if they are relying on their short-term memory. Short-term memory is subtyped as either primary memory or working memory. Primary memory is like a passive holding tank for small amounts of information that do not require manipulation. Thus, repeating a sentence verbatim involves primary memory (Response 2). Working memory (Response 3) not only holds information but manipulates it (e.g., repeating digits in reverse order). Sensory memory (Response 1) is largely accurate but very short-lived (up to a couple of seconds) and involves transforming incoming data into visual (iconic) or auditory (echoic) images that can only be very briefly remembered. Long-term memory (Response 4) is prone to distortions.

You have trouble learning Spanish in college because the French you learned in high school keeps coming to your mind. This phenomenon is known as: • 1. retroactive inhibition. • 2. proactive inhibition. • 3. memory decay. • 4. memory retrieval failure.

2. proactive inhibition. (correct answer) With proactive inhibition, prior learning interferes with new learning. With retroactive inhibition, recent learning interferes with older learning. Proactive inhibition is occurring because what you learned previously (French) is inhibiting your current learning (Spanish). Retroactive inhibition would be present if what you were learning now (Spanish) was interfering with what you had previously learned (French).

A student studies for an exam for 18 hours. She then decides to sleep in the hours remaining before the exam to make sure that the material remains fresh in her mind. She is trying to reduce the effects of: • 1. proactive interference. • 2. retroactive interference. • 3. retrograde amnesia. • 4. anterograde amnesia.

2. retroactive interference. (correct answer, your response) Retroactive inhibition (or interference) occurs when newly learned information interferes with the recall of previously learned information. Here, the student is minimizing the impact of retroactive inhibition by going to sleep, which keeps her from being exposed to new information. Proactive interference (Response 1) occurs when previously learned material interferes with recall of newly learned material. Retrograde amnesia (Response 3) refers to amnesia for memories of events prior to a trauma or accident. Anterograde amnesia (Response 4) refers to amnesia for memories after a trauma or accident.

A physician informs a fifty-year-old male patient that he should be screened for prostate cancer. Such a screening is an example of: • 1. primary prevention. • 2. secondary prevention. • 3. tertiary prevention. • 4. primary or secondary prevention, depending on whether or not the patient had factors placing him at high risk for prostate cancer.

2. secondary prevention. (correct answer) Primary prevention (Response 1) prevents a disorder from occurring in the first place (i.e., reduces the incidence of a disorder). Here, the screening will only reveal whether or not the patient has cancer, but will obviously not prevent cancer. Secondary prevention involves identification and prompt treatment, with the goal of stopping relatively mild disorders from becoming more serious and prolonged (i.e., reduces the prevalence of a disorder). The screening may identify cancer earlier and enable aggressive treatment to reduce the mortality risks. Tertiary prevention (Response 3) aims to reduce the residual effects of a disability or to minimize further negative consequences of an established, serious disorder. For example, the patient might be sent to a workshop after undergoing prostate surgery to assist him in adapting to his condition without depression or relationship problems. Whether or not the patient has risk factors (Response 4) is irrelevant to determining the type of prevention that is being used.

A researcher is interested in how well IQ predicts success in high school (high school GPA). She selects a random sample of 8th graders. The standard error of measurement is: • 1. the average amount of error in predicting each subject's high school GPA. • 2. the average amount of error in calculating each subject's IQ score. • 3. the average amount of error in the group's mean IQ score. • 4. the average amount of spread in the group's IQ scores

2. the average amount of error in calculating each subject's IQ score. (correct answer) The standard error of measurement is the average amount of error in each person's IQ score, as measured by the IQ test. The average amount of error in prediction (Response 1) describes the standard error of the estimate. The average amount of error in the group's mean (Response 3), in relation to the population mean, is the standard error of the mean. The average amount of spread of IQ scores (Response 4) is the standard deviation of the IQ scores.

Coefficient alpha is best described as: • 1. a form of split-half reliability. • 2. the average of all possible split halves. • 3. a formula to correct for splitting the test in half. • 4. measuring the same subjects on similar tests at different points in time.

2. the average of all possible split halves. (correct answer) Coefficient alpha (or Kuder-Richardson) is a subtype of internal consistency reliability that is calculated in a very sophisticated manner, by taking the mean of all possible split halves. Split-half reliability (Response 1), as it names implies, is only the correlation between two halves of the test (i.e., only one possible division, not all possible divisions). Split-half reliability is also a subtype of internal consistency reliability. The Spearman-Brown prophecy formula calculates how much more reliable a test would be if it were longer; thus it corrects for the decrease in number of items that occurs when a test is split in half (Response 3). Alternate-forms (parallel-forms) reliability involves measuring the same subjects on similar tests at different points in time (Response 4).

KR-20 is best described as: • 1. a form of split half reliability. • 2. the average of all possible split halves. • 3. a formula to correct for splitting the test in half. • 4. measuring the same subjects on similar tests at different points in time.

2. the average of all possible split halves. (correct answer) Kuder-Richardson 20 (KR-20) is very similar to coefficient alpha, and is one subtype of internal consistency reliability. It differs from split-half reliability (Response 1), in that it is much more complex and is calculated by taking the mean of all possible split halves. The Spearman-Brown prophecy formula is used to correct for splitting a test in half (Response 3). Measuring the same subjects on similar tests at different points in time (Response 4) describes alternate-form (parallel-form) reliability.

The correction for attenuation would most likely be used to adjust for: • 1. the impact of differences in an individual's obtained score (X) and true score (T). • 2. the impact of a change in reliability on validity. • 3. sampling error and differences between a sample mean and actual population mean. • 4. shrinkage subsequent to cross validation.

2. the impact of a change in reliability on validity. (correct answer) When two variables (the predictor and criterion) are correlated, their respective standard errors of measurement lower criterion-related validity as compared with what it would have been had the two measures been perfectly reliable. We say that the degree of correlation between the predictor and criterion has been attenuated (lessened). The correction for attenuation is a formula that tells us how much better our validity would be if our predictor and criterion had been perfectly reliable.

The correction for attenuation would most likely be used to adjust for: • 1. the impact of differences in an individual's obtained score (X) and true score (T). • 2. the impact of a change in reliability on validity. • 3. sampling error and differences between a sample mean and actual population mean. • 4. shrinkage subsequent to cross validation.

2. the impact of a change in reliability on validity. (correct answer, your response) When two variables (the predictor and criterion) are correlated, their respective standard errors of measurement lower criterion-related validity as compared with what it would have been had the two measures been perfectly reliable. We say that the degree of correlation between the predictor and criterion has been attenuated (lessened). The correction for attenuation is a formula that tells us how much better our validity would be if our predictor and criterion had been perfectly

A manager has decided to perform a utility analysis in his department in order to measure: • 1. the costs versus the benefits of implementing a training program. • 2. the improvement in productivity from implementing a training program. • 3. the cost-effectiveness of each of several different training programs. • 4. employee performance.

2. the improvement in productivity from implementing a training program. (correct answer) Utility analysis is a quantitative research method that measures the effectiveness of an intervention in the workplace in terms of the dollar value of the benefits generated. This dollar value is determined based on the improvement in worker productivity. Response 1 refers to a cost-benefit analysis in which the costs and benefits of a new project or training program are compared. Response 3 describes a cost-effectiveness analysis which is a technique for selecting among competing options, for example, selecting among several different training programs based on what gives the greatest return for the least amount of money.

The standard error of the mean will decrease when: • 1. the population standard deviation is increased and the sample size is decreased. • 2. the population standard deviation is decreased and the sample size is increased. • 3. both the population standard deviation and sample size are increased. • 4. both the population standard deviation and sample size are decreased.

2. the population standard deviation is decreased and the sample size is increased. (correct answer) The formula for the standard error of the mean is that it equals the population standard deviation divided by the square root of sample size. This is a must know formula. There is a direct relationship between standard error of the mean and population standard deviation, such that as standard deviation increases so does error. There is an indirect relationship between standard error of mean and sample size. This makes sense logically as well. The larger your sample, the more likely it is to be representative of the population and the less prone to error it is. To decrease standard error one would increase sample size and decrease standard deviation. Since this is not specifically given as a choice, Response 2 is best and would in fact result in a decrease of standard error.

When referred to in the Taylor-Russell tables, the term "selection ratio" is defined as: • 1. the number of successful employees to the total number of employees. • 2. the proportion of available openings to available applicants. • 3. the success rate in selecting employees using a predictor test compared to the success rate in selecting employees without using a predictor test. • 4. the proportion of those who are correctly identified as successful employees.

2. the proportion of available openings to available applicants. (correct answer, your response) The Taylor-Russell tables are a book of tables that numerically describe the amount of improvement that occurs in selection decisions when a predictor test is introduced. Selection ratio is defined as the proportion of available openings to available applicants. For example, if there are 3 openings and 10 applicants, the selection ratio is 30% or .30. The proportion of current employees that are successful out of the total number of employees (Response 1) refers to the base rate. Incremental validity is the amount of improvement in success rate that results from using a predictor test (Response 3). The proportion of those who are correctly identified as successful employees (Response 4) refers to true positives, a concept derived from decision-making theory.

For any given sample, the average amount of error in scores obtained on any given test is best reflected by: • 1. the standard error of the mean. • 2. the standard error of the measurement. • 3. the standard error of the estimate. • 4. one minus the reliability coefficient.

2. the standard error of the measurement. (correct answer) This question presents the definition of the standard error of the measurement: the average amount of error or spread of scores. The standard error of the mean (Response 1) refers to the average amount of error or spread in a hypothetical sampling distribution of means. It provides the basis for hypothesis testing and determining the region of rejection and acceptance of the null hypothesis. The standard error of the estimate (Response 3) is the average amount of error in estimating or predicting from the predictor (X) to the criterion (Y). Finally, one minus the reliability coefficient (Response 4) does give information about the overall amount of error involved in a test but not the average amount of error in the scores. In fact, one minus the reliability coefficient is square-rooted and multiplied by the standard deviation of the scores in order to determine the standard error of the measurement.

According to the preamble, the goal(s) of APA's Ethical Principles of Psychologists and Code of Conduct (2002) can best be summarized as: • 1. the welfare and protection of consumers of psychological services. • 2. the welfare and protection of consumers of psychological services and education regarding ethical standards of conduct. • 3. beneficence and nonmaleficence. • 4. establishment of the highest possible ethical standards.

2. the welfare and protection of consumers of psychological services and education regarding ethical standards of conduct. (correct answer) APA's 2002 Ethics Code describes two goals in the preamble: "The welfare of and protection of the individuals and groups with whom psychologists work" (Response 1), and "The education of members, students, and the public regarding ethical standards of the discipline." Beneficence and nonmaleficence (Response 3) constitute the first ethical principle. The Ethics Code does not state that it seeks to establish the highest ethical standards (Response 4).

The Solomon Four-Group design is used: • 1. to control for carryover effects when repeated measures are used. • 2. to control for the effects of testing. • 3. to control for the effects of history and failure to return to baseline in single-subject designs. • 4. to control for the effects of experimenter expectancies and demand characteristics.

2. to control for the effects of testing. (correct answer)

The Solomon Four-Group design is used: • 1. to control for carryover effects when repeated measures are used. • 2. to control for the effects of testing. • 3. to control for the effects of history and failure to return to baseline in single-subject designs. • 4. to control for the effects of experimenter expectancies and demand characteristics.

2. to control for the effects of testing. (correct answer, your response) The Solomon Four-Group is a type of research design that controls for the effects of testing (also called practice effects). Subjects are divided into four groups: the first group is tested before and after treatment (pretest/intervention/posttest); the second group is tested twice without any treatment (pretest/no intervention/posttest); the third group is treated and measured afterwards (intervention/posttest); and the fourth is only tested once (posttest). The design accounts for all possible effects of testing. The Latin Square is the most sophisticated counterbalancing design and controls for carryover effects when repeated measures are used (Response 1). Multiple baseline designs eliminate the threat of history and failure to return to baselines (as well as ethical concerns) in single-subject designs (Response 3). Experiment expectances and demand characteristics (Response 4) are controlled by making experiments double-blind.

A researcher is investigating the effect of dose of the antidepressant Zoloft (e.g. 50, 100, 150, or 200 mg) on level of depression. The ANOVA shows a significant effect. To assess whether the results are linear or nonlinear (e.g., quadratic, cubic), she should further analyze the data with: • 1. eta. • 2. trend analysis. • 3. ANCOVA. • 4. MANOVA.

2. trend analysis. (correct answer, your response) With an IV that is incremental (e.g., different doses of medication), to analyze experimental research results in order to determine whether the data are linear or non-linear, the statistic of choice is trend analysis. Trend analysis is a further application of the one-way ANOVA. Eta (Response 1) should be used when correlating two variables, for example, performance and arousal. ANCOVA (Response 3) is used when there is indication of a covariate, or potential confound. MANOVA (Response 4) is used with multiple dependent variables, which is not the case here.

The correction for attenuation informs us how much: • 1. reliability could be corrected for by increasing the number of items in our test. • 2. validity could increase by increasing reliability. • 3. shrinkage has occurred on cross validation. • 4. error can be corrected for in attempting to estimate criterion score from predictor score.

2. validity could increase by increasing reliability. (correct answer) When two variables (the predictor and criterion) are correlated, their respective standard errors of measurement lower criterion-related validity as compared with what it would have been had the two measures been perfectly reliable. We say that the degree of correlation between the predictor and criterion has been attenuated (lessened). The correction for attenuation is a formula that tells us how much better our validity would be if our predictor and criterion had been perfectly reliable. The Spearman Brown formula is used to estimate what reliability would be if there were more test items (Response 1).

When running an ANOVA, a pooled error term is justified when: • 1. sample size is unequal. • 2. variance is equal. • 3. all cells have the same number of subjects. • 4. homoscedasticity is violated.

2. variance is equal. (correct answer) This is a difficult question, requiring an advanced understanding of statistics. It is best to simply know that a pooled error term is used when there is homogeneity of variance (i.e., the variance is equal). When variance is not equal, a separate error term should be used. Homoscedasticity also refers to equal variance. If it were violated (Response 4), the variance would not be equal and a pooled error term could not be used. A mnemonic here might be: when things are equal, they can be pooled together; when unequal, they must be treated separately. Sample size (Response 1) and the number of subjects per cell (Response 3) are not the determining factors in the use of a pooled error term.

When running an ANOVA, a pooled error term is justified when: • 1. sample size is unequal. (your response) • 2. variance is equal. (correct answer) • 3. all cells have the same number of subjects. • 4. homoscedasticity is violated.

2. variance is equal. (correct answer) This is a difficult question, requiring an advanced understanding of statistics. It is best to simply know that a pooled error term is used when there is homogeneity of variance (i.e., the variance is equal). When variance is not equal, a separate error term should be used. Homoscedasticity also refers to equal variance. If it were violated (Response 4), the variance would not be equal and a pooled error term could not be used. A mnemonic here might be: when things are equal, they can be pooled together; when unequal, they must be treated separately. Sample size (Response 1) and the number of subjects per cell (Response 3) are not the determining factors in the use of a pooled error term.

The major threat to internal validity of a time-series quasi-experiment would be a. maturation b. selection. c. regression. d. history.

20. D-- To get this correct (except if you got it correct by chance), you'd need to know what a time-series design is. Basically, you take a number of measurements over time to get a longitudinal baseline trend, then somewhere along the line you introduce your experimental manipulation. If, following the manipulation, you see the trend change, you can infer that your intervention caused the change. But a major threat to the internal validity of this design is a historical event which could co-occur with your experimental manipulation. You'd have no control over these events and they could be a rival explanation for changes in your measurements.

A cross dresser who only gets aroused when wearing women's clothing would be diagnosed with: a. Transvestic fetishism b. Exhibitionism c. Transvestism d. Gender identity disorder

22. A-- According to DSM-IV, Transvetic Fetishism is diagnosed when a heterosexual male has recurrent intense sexually arousing fantasies, sexual urges, or behaviors involving cross dressing. (B) Exhibitionism, involves the exposing of one's genitals to an unsuspecting stranger (C) Transvestism is not a DSM-IV term (D) Gender identity disorder includes a strong and persistent cross-gender identification with evidence of clinically significant distress or impairment in social, occupational, or other areas of functioning.

Cognitive dissonance theory suggests that which of the following clients would be most likely to report benefitting from psychotherapy? a. a wealthy client who pays a very high fee b. a poor client who pays a very high fee c. a wealthy client who pays a very low fee d. a poor client who pays a very low fee

23. B-- According to the theory of cognitive dissonance, a person is motivated to reduce the negative, aversive state that results when his or her cognitions conflict with each other. From the perspective of this theory, the poor client paying a very high fee would experience a state of dissonance. Therefore, this client, more than those in the other responses, would be motivated to believe that he or she is benefiting from therapy.

Patients with depression typically have: a. more rapid onset of REM sleep b. decreased slow wave sleep c. increased percentage of REM sleep d. all of the above

24. D-- Research has found that depression is most associated with a more rapid onset of REM sleep, decreased percentage of slow wave sleep, and increased percentage of REM sleep. The research also suggests that individuals with no prior history of depression but who have rapid REM onset have an increased risk of developing depression (e.g., D. Giles, D. Kupfer, A. Rush, & H. Roffwarg, Controlled comparison of electrophysiological sleep in families of probands with unipolar depression. American Journal of Psychiatry, 1998, 155(2), 192-199).

In an experiment, a psychologist establishes a conditioned startle response to a flashing red light by pairing presentation of the light with a loud noise that naturally elicits a startle reaction. Subsequently, the red light is simultaneously presented with a strong odor just before the loud noise. After many of these conditioning trials, which of the following is likely to occur when the strong odor is presented alone? a. The strong odor will produce a startle response that is even stronger than the response produced by the red light due to sensitization of the subject. b. The strong odor will produce a very weak or no startle response due to habituation. c. The strong odor will not produce a startle response because blocking will have occurred. d. The strong odor may or may not produce a startle response depending on the original strength of the subject's response to the loud noise.

25. C-- Blocking occurs when a CS is presented simultaneously with a second stimulus just before the US. Although it would seem that the second stimulus should acquire the properties of a CS from this procedure, that's not what happens. Instead, the second stimulus does not produce a conditioned response. This is referred to as blocking.

According to Piaget, when a child accommodates new information by forming a new schema or modifying an existing one, this results in a. decentration. b. equilibration. c. assimilation. d. symbolic representation.

26. B-- As defined by Piaget, equilibration is a state of cognitive balance. The need for balance is what motivates the individual to assimilate and accommodate new information.

An adolescent's behavior is motivated primarily by her needs for power and attention. Her problems would probably be best addressed by a therapist adopting the approach of: a. Adler. b. Ellis. c. Perls. d. Freud.

27. A-- Probably the best way to approach this question is to see if the notions of power and attention are linked to any of the people listed in the responses. For Adlerians, a desire to belong is a primary motivator of behavior, but this desire may be channeled into the mistaken goals of power, attention, inadequacy, or revenge.

In ten trials in which no auditory stimulus is presented, a subject reports hearing something on one of the trials. This can best be described as: • 1. 10% false alarms, 90% hits. • 2. 10% misses, 90% correct rejections. • 3. 10% false alarms, 90% correct rejections. • 4. 10% misses, 90% hits.

3. 10% false alarms, 90% correct rejections. (correct answer) Answering this question becomes much easier when we translate it into familiar terms. "False alarm" means false positive (e.g., it's a false alarm if you think someone has HIV but he doesn't); "hit" means true positive; "miss" means false negative (e.g., it's a miss if you think someone is HIV- when she's actually HIV+); and "correct rejection" means true negative. In this example, the subject has had nine true negatives and one false positive, or 90% correct rejections and 10% false alarms.

The concordance rate for Schizophrenia for dizygotic twins is: • 1. 50%. • 2. 20-25%. • 3. 10-15%. • 4. 1%.

3. 10-15%. (correct answer) The concordance rate for dizygotic (fraternal) twins is 10 to 15%. The concordance rate for monozygotic (identical) twins is 50% (Response 1). The concordance rate for dizygotic twins for bipolar disorder and major depression is about 20 to 25% (Response 2). The prevalence of schizophrenia in the general population is 1% (Response 4).

Which of the following statements is most accurate regarding the therapist's countertransference? • 1. Countertransference is generally detrimental to the patient. • 2. The main benefit of countertransference is to inform the therapist about his or her own unresolved issues. • 3. A significant benefit of countertransference is that it enables the therapist to provide the patient with valuable information about the reactions he or she elicits from others. • 4. Countertransference results from projection on the part of the patient.

3. A significant benefit of countertransference is that it enables the therapist to provide the patient with valuable information about the reactions he or she elicits from others. (correct answer) In its broadest definition, countertransference refers to all the emotions and thoughts that are stirred up in a therapist by an encounter with a patient. In classical psychoanalysis, countertransference was seen solely as a negative phenomenon (Response 1), stemming from unresolved issues in the therapist (Response 2). Currently, countertransference is seen as potentially beneficial if managed appropriately. A key benefit is that it can provide patients with feedback about the kinds of reactions they elicit from others. Many theorists posit that the feelings evoked in a therapist are the result of projections by the patient (Response 4). However, even these theorists see projection as one of many possible sources of countertransference; Response 4 implies that it is the only source.

Which of the following would probably not be included in a treatment based on Rehm's Self-Control theory of depression? • 1. Modifying dysfunctional patterns of thoughts. • 2. Self-monitoring. • 3. Altering self-object needs. • 4. Adjusting self-reinforcement behaviors.

3. Altering self-object needs. (correct answer, your response) Rehm's Self-Control model of depression attempts to integrate cognitive as well as behavioral models of depression. Rehm views depression as a result of negative self-evaluations, lack of self-reinforcement, and high rates of self-punishment. Treatment based on Rehm's Self-Control theory includes modification of self-monitoring, self-evaluation, and self-control skills, as well as modification of dysfunctional thoughts and behaviors (selective attention, standard setting, self-attribution, and self-reinforcement). If you were unfamiliar with Rehm's theory, you might have approached this question by recognizing that Response 3 comes from a different theoretical orientation, making it the most likely answer. The concept of self-object needs was developed by Kohut in his psychoanalytic Self Psychology. Self-object needs typically include needs for mirroring, idealization, and twinship.

J. Berry, who views acculturation as a multidimensional construct, would describe an integrated client as one who: a. Has a low retention of the minority culture b. Has high maintenance of the mainstream culture c. Rejects the mainstream culture but has a high retention of the minority culture d. Gets along with others in the workplace

3. B-- J. Barry conceptualizes acculturation as many models existing on a continuum, with the minority culture and the majority or mainstream culture at opposite poles. Integration would be displayed by an individual who has high retention of the minority culture and high maintenance of the mainstream culture.

On an aptitude test with a normal distribution, Don scored at the 70th percentile, Ben scored a T-score of 50, and Mike a z score of +1.0. If you were to rank them in order, from lowest to highest, the correct rank order would be: • 1. Don, Mike, Ben. • 2. Mike, Don, Ben. • 3. Ben, Don, Mike. • 4. Ben, Mike, Don.

3. Ben, Don, Mike. (correct answer, your response) The way to answer this question is to transform all three scores into the same type of score; it's usually easiest to use z-scores. The first step is to draw a bell-shaped curve. Ben's T-score of 50 places him at the mean and hence he has a z-score of 0.0 (remember that T-scores have a mean of 50 and a standard deviation of 10). Mike's z-score is +1.0. Don is at the 70th percentile rank. A z-score of 0.0 corresponds to the 50th percentile rank, and a z-score of +1.0 corresponds to the 84th percentile rank, thus Don is between a z-score of 0.0 and +1.0. It then becomes easy to rank them from lowest to highest: Ben, Don, and Mike.

A person has been diagnosed with amnesia due to chronic alcoholism. Which of the following is most accurate regarding this person's memory deficits? • 1. He is likely to have trouble driving a stick shift car if he has not driven one in the last few years. • 2. He is likely to have trouble remembering the last four presidents of the United States. r response) • 3. He is likely to have trouble remembering what he ate for lunch. • 4. He is likely to have trouble repeating four digits.

3. He is likely to have trouble remembering what he ate for lunch. (correct answer)

An African American experiences an incident of racism. Under which condition is he or she most likely to feel depressed? • 1. External responsibility, external control. • 2. External responsibility, internal control. • 3. Internal responsibility, external control. • 4. Internal responsibility, internal control.

3. Internal responsibility, external control. (correct answer) The combination most likely to cause dysphoria (a combination that is very common) is to attribute responsibility internally but control externally. In everyday language, this combination means, "It's my fault, and there's nothing I can do about it." In this position, the person experience all the blame and guilt, but feels powerless to change the situation. External responsibility, external control (Response 1) translates as, "It is society's fault, and I can't do anything about it." This position is more protective of self-esteem. External responsibility, internal control (Response 2) translates as, "It's society fault, but there's something I can do about it." This position both protects self-esteem and enhances self-efficacy. It's least likely to cause depression. Internal responsibility, internal control (Response 4) translates as, "It's my fault, and I can do something about it."

Which of the following statements is most accurate regarding tipping and group size? • 1. Amount of a tip is negatively correlated with group size. • 2. Percent tip is positively correlated with group size. • 3. Percent tip is negatively correlated with group size. • 4. There is a curvilinear relationship between amount of tip and group size.

3. Percent tip is negatively correlated with group size. Despite some conflicting research, it appears that the larger the group of diners is, the smaller the percentage of tip will be (ruling out Response 2). In other words, a group of three will leave a larger percentage of the bill as a tip than will a group of fifteen. Because the actual amount of the tip goes up as the bill goes up, there is usually a straight positive correlation between the number of diners and the actual tip (ruling out Responses 1 and 4). In other words, a group of fifteen will almost always leave a larger tip than a group of three (even though this tip may actually be a smaller percentage of the overall bill).

Scores on infant development tests (e.g., the Bayley) are poor predictors of adult IQ. Which is the least likely explanation for this phenomenon? • 1. Infant development tests emphasize sensory and motor capacities, whereas adult intelligence tests measure complex cognitive functions. • 2. Variability in maturation rates is a much more significant factor in infancy than in adulthood. • 3. The infant development scales are unreliable. • 4. Infant precursors of adult intelligence are not adequately sampled in these scales.

3. The infant development scales are unreliable. (correct answer) The infant scales are reliable for their intended purpose, the measurement of infant development. In other words, if you administer the same scale to an infant more than once, the scores will be close to each other. All of the other responses are true.

How are the mean and standard deviation affected if a constant is subtracted from every score? • 1. Both decrease. • 2. Both increase. • 3. The mean decreases and the standard deviation remains the same. • 4. The mean remains the same and the standard deviation decreases.

3. The mean decreases and the standard deviation remains the same. (correct answer, your response) All arithmetic operations (adding, subtracting, multiplying, and dividing) affect the mean, but only multiplication and division affect the standard deviation and the variance. To approach this question intuitively, you can imagine a number line with the test scores graphed along it. Subtracting a constant would shift all the scores downward (and hence the mean) but the overall spread (i.e., variability) would remain the same.

A group of students meet over dinner at a restaurant to discuss a school project. Based on the predictions of social loafing theory, under what conditions will they leave the biggest tip? • 1. Each person leaves a tip separately. • 2. The tip is pooled together, without each person knowing how the others paid. • 3. The tip is pooled together, and each person is aware of how much the others paid. • 4. Each person pays a preset amount for the tip.

3. The tip is pooled together, and each person is aware of how much the others paid. (correct answer) Social loafing refers to the phenomenon that people contribute less to a task when they are part of a group than when they are working alone, especially when the person's contribution is anonymous and not subject to individual evaluation. The best way to minimize the impact of social loafing is to make each person's contribution public, in this case, by making everyone aware of the amount of tip that each person is leaving.

Which of the following schedules of reinforcement would most likely result in a low to moderate, steady response rate? • 1. Fixed interval. • 2. Fixed ratio. • 3. Variable interval. • 4. Variable ratio.

3. Variable interval. (correct answer) Variable schedules result in response rates that are smooth and steady, both before and after reinforcement (ruling out Responses 1 & 2). In contrast, fixed schedules, both fixed interval and fixed ratio, result in scalloped patterns: increased rate of the target behavior occurs just prior to reinforcement, followed by a lull right after reinforcement. The scalloped pattern is most dramatic in the fixed interval schedule (Response 1). Of all the schedules of reinforcement, the variable ratio schedule results in the strongest response rate, the fixed ratio schedule results in a rate of responding that is moderate to high, the variable interval schedule results in a low to moderate response rate, while the fixed interval schedule yields a rate of responding that is lowest.

An elementary school student has just learned the colors of the rainbow with the acronym ROY G BIV. This is an example of which memory enhancing strategy? • 1. The method of loci. • 2. The pegword system. • 3. Word association. • 4. Substitute word system.

3. Word association. (correct answer) Word association involves forming words or sentences with the first letter of the items being memorized (acronyms), or making up stories that connect the items together. The method of loci (Response 1) involves first forming a mental image of a walk through a certain physical location, such as a house. The person then forms a visual image of the words on a list, putting each in a specific place as he or she mentally walks through the house. For example, a person trying to recall a list of grocery items may initially picture bacon on the front door, bread in the kitchen, and meat in the dining room. The pegword system (Response 2) involves first memorizing a set of ten visual images that can later be pegs on which to hang ideas, e.g., one is bun, two is shoe, three is tree. Then the person associates each image with what is to be remembered. For example, if the person is trying to remember a list of grocery items, he may associate soda with hamburger buns, or talcum powder with shoe, etc. In the substitute word technique (Response 4), the person breaks down the word to be remembered into parts, and substitutes words that are more familiar and can be visualized. For example, a person who wants to memorize the word catabolize would break it into cat, a ball, eyes, and then form a visual image of these words.

A woman feels throbbing on one side of the head that intensifies during activity. She is most likely experiencing: • 1. a muscle contraction headache. • 2. sinusitis. • 3. a migraine headache. • 4. a cluster headache.

3. a migraine headache. (correct answer) Migraine headaches, a type of vascular headache, are characterized by unilateral pain (pain on only one side of the head), that is pulsating, of moderate to severe intensity, aggravated by routine physical activity, and associated with nausea, photophobia (light sensitivity) and phonophobia (sensitivity to noise). The pain tends to be located in the frontal and temporal areas. While "common migraines" do not involve prodromal symptoms, "classic migraines" (found in about 30% of migraine sufferers) include visual auras and sensory disturbances (e.g., hemiparesis). Migraines are much more common in females. Muscle contraction headaches (Response 1), commonly known as tension headaches, are bilateral in location, and are characterized by a dull and steady tightness or pressure around the head, with pain of mild to moderate intensity. Sinusitis (Response 2) results from inflammation of the sinuses. In addition to nasal congestion, sinusitis can involve headaches, which may be described as pain over the eyes, behind the eyes, or above the eyebrows. Cluster headaches (Response 4), a type of vascular headache, typically involve unilateral pain in the orbital (around the eye) or frontal regions. These headaches, which can be precipitated by alcohol ingestion, are more frequent in men, usually begin during sleep, and recur over a period of days or weeks causing closely spaced attacks of intense pain, followed by remissions that can last weeks, months or even years. The pain is frequently described as piercing.

A patient completes the MMPI-II and receives a code of "4-9." You should be most concerned about: • 1. psychosis. • 2. suicide. • 3. acting out. • 4. somatic problems.

3. acting out. (correct answer) On the MMPI-2, scale 4 is psychopathic deviance and refers to antisocial tendencies. Scale 9 is hypomania and refers to hypomanic and manic behaviors, as well as a generally high energy level. A profile of "4-9" therefore raises concerns about acting out and danger to others. Suicide (Response 2) is a particular concern with a "2-9" profile. Scale 2 is the depression scale. Someone who receives a "2-9" is depressed but has enough energy to take action (i.e., attempt suicide). Psychosis (Response 1) is indicated by elevations on scale 8, schizophrenia, and possibly scale 6, paranoia. Somatic problems (Response 4) are reflected by elevations on scales 1 (hypochondriasis) and 3 (hysteria).

Handedness appears to be genetically determined. Preference for handedness first expresses itself at ____ and becomes firmly established by age ____: • 1. birth, four to five. • 2. age two, four to five. • 3. age two, seven to eight. • 4. age four, seven to eight.

3. age two, seven to eight. (correct answer) While hand preference typically emerges around age two, it only becomes firmly established around age seven to eight. This age corresponds to the age associated with increased brain specialization and decreased brain plasticity.

Behavioral marital therapy focuses on: • 1. enhancement of independent functioning. • 2. self-monitoring and self-management. • 3. behavior exchange and communication/problem-solving. • 4. marital contracts and participant modeling. (

3. behavior exchange and communication/problem-solving. (correct answer) Behavioral marital therapy is rooted in social learning theory and behavioral analysis. This theory contends that maladaptive behavior is a result of deficient reward exchanges and communication problems. Thus, treatment is based on changing reinforcement for concrete behaviors (in terms of interpersonal consequences), and improving communication.

Incremental validity is associated with a test's: • 1. content validity. • 2. construct validity. • 3. concurrent validity. • 4. convergent validity.

3. concurrent validity. (correct answer) Incremental validity is associated with criterion-related validity. It is defined as the amount of improvement in success rate that results from using a predictor test. For example, when a predictor test is used for the purpose of employee selection, incremental validity tells us how much improvement there is in selecting employees with this test, as compared with using a traditional method of employee selection (such as interviews). Since performance on the predictor test is correlated with employee success (the criterion), and is used to predict job success, incremental validity is associated with criterion-related validity. Concurrent validity (Response 3) is one type of criterion-related validity. Predictive validity is the other type of criterion-related validity. Content validity (Response 1) addresses how adequately a test samples a particular content area such as statistics, test construction, etc. Content validity is determined through a panel of experts. Construct validity (Response 2) looks at how adequately a new test measures a construct or trait. Convergent validity (Response 4) is used for determining construct validity, and tells us the extent to which a test correlates with other tests that measure the same construct.

Incremental validity is associated with a test's: • 1. content validity. • 2. construct validity. • 3. concurrent validity. • 4. convergent validity.

3. concurrent validity. (correct answer, your response) Incremental validity is associated with criterion-related validity. It is defined as the amount of improvement in success rate that results from using a predictor test. For example, when a predictor test is used for the purpose of employee selection, incremental validity tells us how much improvement there is in selecting employees with this test, as compared with using a traditional method of employee selection (such as interviews). Since performance on the predictor test is correlated with employee success (the criterion), and is used to predict job success, incremental validity is associated with criterion-related validity. Concurrent validity (Response 3) is one type of criterion-related validity. Predictive validity is the other type of criterion-related validity. Content validity (Response 1) addresses how adequately a test samples a particular content area such as statistics, test construction, etc. Content validity is determined through a panel of experts. Construct validity (Response 2) looks at how adequately a new test measures a construct or trait. Convergent validity (Response 4) is used for determining construct validity, and tells us the extent to which a test correlates with other tests that measure the same construct.

A researcher looks at several variables (study time, graduate school GPA, anxiety level) in an attempt to predict who is likely to pass the EPPP. The researcher will analyze her results using a: • 1. multiple regression. • 2. multiple cutoff. • 3. discriminant analysis. • 4. path analysis.

3. discriminant analysis. (correct answer, your response) A discriminant analysis is required when several independent variables are used to predict group membership. Group membership is another way of saying discrete categories. In this case, three continuous variables are being used to predict whether subjects belong in the "pass exam" or "fail exam" category. Multiple regression (Response 1) is appropriate when several independent variables are used to predict one continuous dependent variable (e.g., EPPP score). In a test with multiple cutoff (Response 2), a subject can only pass the overall test if he meets the cutoff on each subtest. Path analysis (Response 4) applies multiple regression techniques to test a theoretical model that specifies causal links among variables.

A patient at high risk for suicide states that he plans to quit therapy because it is not helping. Your most ethical course of action would be to: • 1. provide pretermination counseling. • 2. negotiate a no-suicide contract before termination. • 3. discuss the patient's views and needs, encouraging the patient to receive appropriate treatment. • 4. encourage the patient to remain in treatment with you until the suicidal crisis passes.

3. discuss the patient's views and needs, encouraging the patient to receive appropriate treatment. (correct answer) This is an example of a difficult question for which no response is ideal and several responses have some merit. Although pretermination counseling is mandated by the Ethics Code, Response 1 does not address the issue of the current crisis. Similarly, although a no-suicide contract (Response 2) may be clinically indicated, the question asks for the most "ethical" course of action, so the correct response should address the ethical ramifications of the scenario. Simply encouraging the patient to stay with you (Response 4) does not address the possibility that you are not providing him with appropriate treatment. In fact, standard 10.10 of the APA Ethics Code (2002) actually mandates the termination of a therapy relationship when "the patient or client no longer needs the service, is not benefiting, or is being harmed." Thus it would be most ethical to discuss the patient's view and needs, and to encourage the patient to secure treatment that is most appropriate for him. This may mean continuing in therapy with you, or this patient might best be served by referring him to another provider.

A 12-month-old is exposed to the Strange Situation Procedure. She exhibits fear and confusion, responds to the mother's return with initial enthusiasm, and then suddenly turns away. You go on to find out the child's mother was abused in her childhood, and has not yet worked this through. The child is most likely: • 1. anxious-avoidant. • 2. anxious-ambivalent. • 3. disorganized-disoriented. • 4. insecurely attached.

3. disorganized-disoriented. (correct answer)

An example of cluster sampling is: • 1. dividing the California population into groups based on income levels and then randomly selecting equal-sized samples from the groups. • 2. randomly selecting Californians from different income levels in proportion to their representation in the population. • 3. dividing the population into groups based on California counties and then randomly selecting samples from randomly selected counties. • 4. selecting from the population of California such that each person has an equal likelihood of being selected.

3. dividing the population into groups based on California counties and then randomly selecting samples from randomly selected counties. (correct answer) Cluster sampling involves identifying naturally occurring groups or clusters (e.g., schools in a school district, counties in a state) and then randomly selecting certain of these clusters. Typically, all the subjects within the selected clusters are then sampled. Alternatively, subjects may be randomly selected from the clusters (Response 3). Response 1 describes stratified random sampling. Response 2 describes proportional sampling. Response 4 describes simple random sampling.

An example of cluster sampling is: • 1. dividing the California population into groups based on income levels and then randomly selecting equal-sized samples from the groups. • 2. randomly selecting Californians from different income levels in proportion to their representation in the population. • 3. dividing the population into groups based on California counties and then randomly selecting samples from randomly selected counties. • 4. selecting from the population of California such that each person has an equal likelihood of being selected.

3. dividing the population into groups based on California counties and then randomly selecting samples from randomly selected counties. (correct answer, your response) Cluster sampling involves identifying naturally occurring groups or clusters (e.g., schools in a school district, counties in a state) and then randomly selecting certain of these clusters. Typically, all the subjects within the selected clusters are then sampled. Alternatively, subjects may be randomly selected from the clusters (Response 3). Response 1 describes stratified random sampling. Response 2 describes proportional sampling. Response 4 describes simple random sampling.

A problem with using extinction to reduce self-injurious behavior is that: • 1. extinction tends not to work for self-harm behavior. • 2. extinction is a classical conditioning technique while most self injurious behavior is under the control of operant conditioning. • 3. extinction does not create an immediate reduction of behavior. • 4. extinction works for certain self-injurious behaviors (e.g., head banging) but not others.

3. extinction does not create an immediate reduction of behavior. (correct answer) While extinction may work to reduce self-injurious behavior (ruling out Responses 1 and 4), it does not create an immediate reduction in behavior. In fact, the use of operant extinction is typically associated with an initial increase in target behavior (a response burst) which could be potentially life-threatening in the case of self-injurious behavior such as head banging or cutting. Extinction is both a classical and operant conditioning technique (ruling out Response 2). In classical conditioning, extinction is brought about by presenting the conditioned stimulus (CS) without the unconditioned stimulus (US), while in operant conditioning, extinction is brought about by withholding reinforcement.

Scores on the Wechsler Intelligence Scale for Children (WISC-IV) include: • 1. full scale IQ (FSIQ), verbal and performance IQs (VIQ and PIQ), four composite or index scores, and subtest scores. • 2. full scale IQ (FSIQ), verbal and performance IQs (VIQ and PIQ), and subtest scores. • 3. full scale IQ (FSIQ), four composite or index scores, and subtest scores. • 4. verbal and performance IQs (VIQ and PIQ), four composite or index scores, and subtest scores.

3. full scale IQ (FSIQ), four composite or index scores, and subtest scores. (correct answer) Unlike previous versions, the WISC-IV does not provide verbal and performance IQs (ruling out Responses 1, 2, and 4). It does provide a full scale IQ and four composite or index scores (verbal comprehension, perceptual reasoning, working memory, and processing speed). It also provides scores on individual subtests.

A gradual decrease in an unconditioned response due to repeated presentation of the unconditioned stimulus is termed: • 1. classical extinction. • 2. satiation. • 3. habituation. • 4. operant extinction.

3. habituation. (correct answer) Habituation is a concept from classical conditioning. It occurs when a person is repeatedly exposed to an unconditioned stimulus and, as a result, the unconditioned response eventually decreases. For example, people habituate to the hum of the computer and no longer notice it after a while. Classical extinction (Response 1) involves repeated presentation of the conditioned stimulus without the unconditioned stimulus; eventually, the pairing between the two is extinguished. Satiation (Response 2), an operant conditioning concept, occurs when a reinforcer loses its value over time. For example, if you were reinforced with hot fudge sundaes three times a day, you would quickly find that the sundaes were no longer very reinforcing. Operant extinction (Response 4) refers to withholding reinforcement for a previously reinforced behavior. For example, if a teacher stops paying attention (a reinforcer) to the disruptive behavior of a student, the student's behavior will slowly decrease (be extinguished).

The color red is paired with a loud noise over several trials. The color red is then paired with a tone. Afterwards, the color red is again paired with the loud noise. This is an example of: • 1. stimulus generalization. • 2. pseudoconditioning. • 3. higher-order conditioning. • 4. simultaneous conditioning.

3. higher-order conditioning. (correct answer) This situation describes higher-order conditioning, in which a neutral stimulus (the color red) is deliberately paired with a US (the loud noise). Once the neutral stimulus begins to elicit a response, it is said to become a CS, and it is then paired with a second neutral stimulus (the tone). In order to keep the higher-order conditioning in effect, the first neutral stimulus (the color red) will usually be again paired with the US at some point. Stimulus generalization (Response 1) is a process that does not involve pairing, in which a person generalizes from a CS (e.g., the color red) to a similar neutral stimulus (e.g., the color pink). Pseudoconditioning (Response 2) occurs when a response is elicited to a neutral stimulus that has not been paired with either the CS or the US. Simultaneous conditioning (Response 4) involves presenting the CS and the US at the same time, such that they completely overlap, which results in no conditioning taking place.

An employee believes that if she works hard, she will earn rewards. This is an example of: • 1. motivation. • 2. valence. • 3. instrumentality. • 4. expectancy.

3. instrumentality. (correct answer)

Scores from non-parallel measures can be equated through the use of: • 1. classical test theory. • 2. empirical criterion keying. • 3. item response theory. • 4. ipsative measures.

3. item response theory. (correct answer) Item response theory (IRT), also termed latent trait theory, is used to calculate to what extent a specific item on a test correlates with an underlying construct. Put differently, item response theory looks at a subject's performance on a test item as representing the degree to which the subject has a latent trait. For example, correctly answering a difficult statistics question would indicate a subject is high in the latent trait of "statistics ability." It can be used, therefore, to compare a subject's performance on two measures that have different types or number of items, or are scored differently. IRT is used to develop individually tailored "adaptive" tests, in which an answer to one question in a domain area determines whether another question in that area will be asked. Such tests end up having the fewest number of items necessary to assess a subject's performance. Classical test theory (Response 1) views an individual's test score as being the sum of true score variability and error score variability (X = T + E). In empirical criterion keying (Response 2), which was used in the development of the original MMPI, items are chosen based on their ability to discriminate group membership. In other words, a specific item on the MMPI was only kept on the Schizophrenia scale if people without Schizophrenia and people with Schizophrenia answered it differently. Ipsative measures (Response 4) yield only information on an individual, without any information about how that individual compares to others. For example, an ipsative measure might show that Mr. Smith likes gardening more than cooking, but doesn't tell us whether he likes gardening more or less than the average person does.

An employee who receives a preponderance of poor ratings from his supervisors would present himself in the best possible light if he reported his: • 1. mode score. • 2. median score. • 3. mean score. • 4. average score.

3. mean score. (correct answer) The best way to approach this question is graphically. The distribution of ratings for an employee who receives a preponderance of poor or low ratings will be positively skewed (many low ratings, few high ratings). In a positively skewed distribution, the mode corresponds to the lowest rating, the median the middle rating, and the mean the highest rating. While the mean is technically the arithmetic average of all scores, the term average (Response 4) is more general and can include the median. For example, when reporting "average" prices of homes, typically the median is used.

All of the following are associated with extinction except: • 1. response burst. • 2. withholding reinforcement. • 3. negative reinforcement. • 4. spontaneous recovery.

3. negative reinforcement. (correct answer) Extinction occurs in both operant and classical conditioning and is used to eliminate behavior. In classical conditioning, extinction is brought about by presenting the CS without the US. Once the behavior is extinguished, one may see a brief reoccurrence of the behavior, which is known as spontaneous recovery (Response 4). In operant conditioning, extinction is brought about by withholding reinforcement from a behavior that has been previously reinforced (Response 2). At the beginning of extinction trials, there is usually an increase in the behavior to be eliminated, which is known as a response burst (Response 1). Negative reinforcement (Response 3) is used to increase a behavior, rather than eliminate it, and involves the removal of a noxious stimulus after the target behavior has occurred.

Analysis of covariance (ANCOVA) is a statistical test that is used when you want to: • 1. covary out multiple dependent variables. • 2. covary out the effect of multiple IVs on one DV. • 3. partial out a moderator variable. • 4. transform a moderator variable into another IV.

3. partial out a moderator variable. (correct answer, your response) Feedback: CORRECT! The analysis of covariance (ANCOVA) combines standard analysis of variance (ANOVA) with the technique of partial correlation. It allows the effects of an extraneous or moderator variable to be statistically removed ("partialed out"). It is neither the dependent variable (Response 1) nor multiple independent variables (Response 2) that is being covaried. Transforming a moderator variable into another IV (Response 4) describes "blocking" which requires the randomized block ANOVA. An advantage of this latter technique is that it allows for investigation of the main and interaction effects of the blocked variable with the other IV's.

When it comes to research on performance, it's been found that: • 1. there is a direct linear relationship between performance and motivation. • 2. there is a curvilinear relationship between motivation and performance. • 3. performance is affected by a combination of environmental variables, individual attributes, and work efforts. • 4. performance is affected by a combination of expectancy, valence, and instrumentality.

3. performance is affected by a combination of environmental variables, individual attributes, and work efforts. (correct answer, your response) This response explicitly describes the general performance model which states that performance is a function of opportunity (environmental variables and organizational support), capacity/ability (individual attributes), and willingness/motivation (work effort). The relationship between performance and motivation is not linear; if a person is highly motivated but lacking in ability, he or she will perform more poorly than someone who has moderate motivation but high ability (Response 1). According to the Yerkes-Dodson law, there is a curvilinear relationship between performance and arousal, not specifically performance and motivation (Response 2). Expectancy, valence, and instrumentality (Response 4) are aspects of Vroom's valence-instrumentality-expectancy (VIE) theory, which is a theory of motivation rather than performance.

A patient with Korsakoff's syndrome would be able to: • 1. recognize the examining doctor each time he visits. • 2. recall three words (e.g., ball, flag, tree) five minutes after presentation. • 3. repeat five digits. • 4. retrieve memories of events prior to the development of the syndrome.

3. repeat five digits. (correct answer) Korsakoff's is an amnestic disorder that results from chronic thiamin deficiency associated with longstanding alcohol abuse. The most significant problem associated with Korsakoff's is anterograde amnesia (difficulty forming new memories). Thus a patient with Korsakoff's will be unable to recognize the doctor (Response 1) and unable to recall three words five minutes after presentation (Response 2). A patient with Korsakoff's will also suffer from retrograde amnesia, and may not be able to retrieve memories of events prior to developing the syndrome (Response 4). Short-term memory (Response 3), the ability to remember for 30 seconds, remains intact.

A researcher is studying the effect of different treatments for hyperactivity. First graders receive one of two treatments. Group A's pre-treatment mean is 15, and the post-treatment mean is 13. Group B's pre-treatment mean is 27, and the post-treatment mean is 24. The most likely threat to this research is: • 1. maturation. • 2. regression. • 3. selection. • 4. demand characteristics.

3. selection. (correct answer, your response) This research is most significantly affected by the threat of selection bias, or non-random assignment. Given that the pretreatment means are so dramatically different, it is unlikely that subjects were randomly assigned to the groups. Maturation (Response 1) and regression (Response 2) are typically more of concern when the study is a one-group pre-post design.

An aphasia characterized by normal fluency, impaired comprehension and paraphasias is: • 1. global. • 2. Korsakoff's. • 3. sensory. • 4. motor.

3. sensory. (correct answer)

Testing-of-limits: • 1. may be used to boost an underperforming examinee's score. • 2. should typically be done after each subtest. • 3. should typically be done at the end of a test. • 4. should typically be used when an examinee is performing very well.

3. should typically be done at the end of a test. (correct answer) In testing-of-limits, a test is first administered to a student using the standard test administration procedure. After completing the test, subtests are given again to the student with steps to facilitate performance (e.g., additional cues or structure are provided, time limits are eliminated). Testing-of-limits is done when an evaluator want to gain additional qualitative or descriptive information about an examinee. Implementation of testing-of-limits should be hypothesis driven (i.e., there should be a reason for it). For example, if there is concern that time constraints are significantly affecting an examinee's scores, allowing extra time to complete Block Design items could help ferret that out. Or, to help clarify the cause of poor Arithmetic performance, one might repeat some items while giving the examinee a pencil and paper. Experts in psychological assessment (e.g., Sattler) strongly advise that testing-of-limits take place only after the entire test has been completed. Testing-of-limits after each subtest results in a non-standardized administration (ruling out Response 2). Typically, testing-of-limits is not used when an examinee is already performing well (ruling out Response 4). Testing-of-limits provides only descriptive information and should not be used in any way to alter an examinee's scores (ruling out Response 1).

A child has a history of engaging in driven, repetitive, nonfunctional behaviors, such as body rocking and self-hitting. These symptoms are most characteristic of: • 1. autistic disorder. • 2. chronic motor tic disorder. • 3. stereotypic movement disorder. • 4. obsessive-compulsive disorder.

3. stereotypic movement disorder. (correct answer) Stereotypic movement disorder is characterized by motor behavior that is repetitive and nonfunctional. This diagnosis is made only if the repetitive behaviors cannot be accounted for by another diagnosis, such as OCD or a tic disorder. While repetitive movements are present in autistic disorder (Response 1), other symptoms must be present in order for the diagnosis to be made, such as impairment in social interactions and communication, and a restricted repertoire of behavior. In stereotypic movement disorder, the movements tend to be driven or intentional, whereas in tic disorders (Response 2), the tics have a more involuntary quality. Common examples of motor tics include eye blinking, touching, squatting and retracing steps. In obsessive-compulsive disorder (Response 4), the compulsions are more complex and ritualistic, and are performed in response to obsessions or rules, for the purpose of reducing distress. By contrast, in stereotypic movement disorder, the behaviors are non-functional.

All of the following are components of Lazarus's theory of emotion, stress, and coping except: • 1. primary appraisal. • 2. secondary appraisal. • 3. tertiary appraisal. • 4. re-appraisal.

3. tertiary appraisal. (correct answer, your response) Lazarus attempted to explain the relationship between emotions, stress, and coping with his theory of cognitive appraisal. In primary appraisal, a situation is initially judged as being positive, stressful, or irrelevant to personal well-being. Secondary appraisal involves an evaluation of the resources the individual has available to cope with the situation. The individual then goes through a process of re-appraisal as both the stimulus situation and coping strategies are monitored, and primary and secondary appraisals are modified as necessary. A person suffers stress when he believes he lacks the resources to deal with difficult events, but he does not suffer stress if he believes he has such resources. Stress and coping are thus intimately related to each other and to cognitive factors. Tertiary appraisal is not a component of Lazarus's theory.

To ensure that adverse impact does not occur: • 1. at least 80% of minority applicants should be hired. • 2. the number of minorities hired should be at least 80% of the number of non-minorities hired. • 3. the minority selection ratio should be 80% of the majority selection ratio. • 4. unfairness and differential validity should be minimized.

3. the minority selection ratio should be 80% of the majority selection ratio. (correct answer)

A key distinguishing feature in bringing about classical versus operant extinction is: • 1. one involves spontaneous recovery, the other a response burst. • 2. one involves pairing, the other reinforcement. • 3. the nature of what is withheld. • 4. the nature of what is added.

3. the nature of what is withheld. (correct answer) Crucial to answering this question correctly is to be clear on what it is asking. It is not asking about just any difference between classical and operant extinction; otherwise Response 1 would be entirely correct. It is also not asking about the difference in how classical versus operant conditioning is brought about; otherwise Response 2 would be entirely correct. It is asking about extinction and the difference between how classical and operant extinction is brought about. In classical extinction, the CS is repeatedly presented without the US, that is, the US is withheld. In operant extinction, the subject repeatedly performs the once-reinforced behavior but is no longer reinforced -- that is, reinforcement is withheld.

In terms of minimizing discrimination in hiring and promotion, Cleary's model (1968) suggests a test is fair when: • 1. criterion related validity is equivalent for minorities and non-minorities. • 2. based on the test, minority group members represent at least four-fifths of the percentage of non-minorities hired or promoted. • 3. the regression line is equivalent for minorities and non-minorities. • 4. differences on predictor scores for minorities and non-minorities result in similar criterion scores.

3. the regression line is equivalent for minorities and non-minorities. (correct answer)

Deception in research is only acceptable when: • 1. informed consent is provided • 2. there aren't physical or psychological risks to the participants. • 3. the study has strong scientific value and non-deceptive alternatives are not available. • 4. participants are debriefed as early as feasible, preferably at the conclusion of their participation, but no later than at the conclusion of data collection.

3. the study has strong scientific value and non-deceptive alternatives are not available. (correct answer) Standard 8.07 of the APA Ethics Code states that using deception in research is to be avoided unless it "is justified by the study's significant prospective scientific, educational, or applied value" and "effective nondeceptive alternative procedures" are not available. These conditions are directly addressed by Response 3. This is an example in which accurate reading of the question is important. For example, the Ethics Code does require that if deception is used, a full de-briefing should occur as soon as possible (Response 4); however, debriefing is not a condition that justifies the use of deception in the first place. Informed consent (Response 1) is required even when deception is involved (e.g., the participant must consent to participate in the research, know that they have the right to withdraw, know the risks and benefits). It is acceptable for a project involving deception to entail risks to the participants (Response 2); it is simply that the deception may not involve hiding these risks. For example, participants may be lead to believe that a study on social desirability is actually a study on preference for ice cream flavor. They should not be deceived, however, in regard to the risk of gaining weight.

A researcher rejects the null when alpha is set at .01. This indicates that: • 1. there is a 99% chance the researcher has correctly rejected the null. • 2. there is a 99% chance the researcher has correctly retained the null. • 3. there is a 1% chance the researcher has incorrectly rejected the null. • 4. there is a 1% chance the researcher has incorrectly retained the null.

3. there is a 1% chance the researcher has incorrectly rejected the null. (correct answer) The alpha level corresponds to the likelihood of making a Type I error, in other words incorrectly rejecting the null hypothesis, or rejecting the null when it is in fact true. So, alpha at .01 means that there is a 1% chance of making a Type I error, or incorrectly rejecting the null. Response 4 describes beta, or the probability of incorrectly accepting the null. Keep in mind, for exam purposes, you cannot ascertain beta unless it is explicitly given. For complicated statistical reasons, when we know that alpha level, all we know is the likelihood of a Type I error (e.g., 1% or 5%). We don't know the power level (Response 1) or the beta level -- this is a fact that is probably best memorized.

The latent trait model or item response theory is an alternative to classical test theory. Item response theory is based on the premise that: • 1. total variability in test takers' scores can be explained by a combination of the test's reliability and error variability. • 2. differences between two persons in terms of percentage correct obtained on the same exam reflect differences in underlying latent traits. • 3. when test content is of varying difficulty, uniform scales of measurement can be applied to persons of different ability levels. • 4. the test taker's response to a given item will indicate the extent of the latent trait possessed.

3. when test content is of varying difficulty, uniform scales of measurement can be applied to persons of different ability levels. (correct answer, your response) According to item response theory (latent trait model), it is assumed that item performance is related to the amount of the respondent's latent trait, e.g., statistics ability. Latent trait models are used to establish a uniform scale of measurement that can be applied to individuals of varying ability and to test content of varying difficulty. Response 1 accurately describes the premise of classical test theory. Response 2 is more in line with classical test theory than item response theory. The determination as to how much of a latent trait is possessed requires analyzing the responses to all items, not just one (Response 4).

According to Kahneman and Tversky's loss aversion model, in deciding between potential investments, the typical investor: • 1. will seek to maximize gains, and minimize losses. • 2. will seek to minimize losses. • 3. will be more swayed by concerns about losing money than by hopes of making money. • 4. is less likely to make riskier decisions when he invests alone rather than as part of a group.

3. will be more swayed by concerns about losing money than by hopes of making money. (correct answer) The loss aversion model of Kahneman and Tversky is quite straightforward: people's decisions are more affected by their desire to avoid losses than by their desire to make gains, and often this bias results in decisions that are not objectively the best ones. Not surprisingly, this theory has interesting applications to a host of economic issues, including investment and employment decisions. All investors always seek to maximize gains and minimize losses (Responses 1 and 2); the loss aversion model is more specific, positing that avoiding loss is usually the stronger motivation. Risky shift refers to the tendency to make riskier decisions when one is a member of group than when one is acting alone (Response 4).

A defendant with Schizophrenia is on trial for indecent exposure. He is deemed incompetent to stand trial. Consequently, the defendant: • 1. will never be allowed to stand trial. • 2. will be treated and re-evaluated for competency to stand trial within 180 days. • 3. will be tried if he regains competence. • 4. may be treated to help him regain competence.

3. will be tried if he regains competence. (correct answer) Incompetence to stand trial refers to a determination that a defendant is unable to understand the nature and consequences of the proceedings against him or to assist properly in his defense.The law dictates that a person who is deemed incompetent to stand trial must receive psychiatric treatment (typically either inpatient psychiatric treatment or psychiatric treatment while in county jail) aimed specifically at enabling the person to stand trial with as little delay as possible (ruling out Response 4 which suggests treatment is optional). Once the psychiatric staff believes the defendant has become competent, the court will hold a hearing. If the defendant is found competent, the court will then order the defendant's release if the defendant is in an inpatient psychiatric treatment facility, and set a date for trial. There is no requirement that the defendant be reevaluated within 180 days of treatment (ruling out Response 2).

A psychologist is consulted by the parents of a child who was referred by a pediatrician for evaluation. During the interview with the parents, they report the girl suffered a severe head injury which they haven't told the pediatrician about. Assuming appropriate consents have been obtained, the psychologist: a. should have the parents report this to the pediatrician. b. should report this to the pediatrician. c. should not report this until the parents tell the pediatrician first. d. should keep this information to herself.

32. B-- This question looks like one on confidentiality, and, indeed, there are some aspects of confidentiality embedded here. But, more than that, it's about professional relationships. You're told in the item stem that all consents have been obtained, so the confidentiality issue was already addressed. The question then is, "What should the psychologist do?" Since we're all here to help the patient, the most appropriate thing to do is to let the pediatrician know about your findings. You would call (probably call rather than write since calling is quicker) the pediatrician and report your findings. It was the pediatrician who made the referral in the first place, and you have the parents' consent.

A therapist working from the perspective of Minuchin's school of thought would examine a family system in terms of a. multigenerational transmission processes. b. subsystems and boundaries. c. fusion and differentiation. d. communication style

33. B-- Salvador Minuchin's Structural Family Therapy is based on and extends general family systems theory. The goal is to restructure maladaptive family structures, including family subsystems and boundaries.

An older brother helps his younger brother to build a fort by giving his brother verbal explanations and suggestions as they go. The researcher who would use this as an illustration of his theory is: a. Piaget b. Patterson c. Vygotsky d. Erikson.

34. C-- This technique is what Vygotsky referred to as "scaffolding" and it is based on his idea that children receive support and teaching from their parents and other more experienced children.

You are seeing a client who exhibits all the major symptoms of a Major Depressive episode. You should a. invite his wife in to aid in completing a more comprehensive family history. b. assess for early child abuse. c. refer him to a psychiatrist for a medical evaluation. d. assess for recent deaths in his family.

35. C-- While the other choices could be options, "all the major symptoms" should tip you off to your client's level of distress. Standard 1.20 (Consultations and Referrals) states, "Psychologists arrange for appropriate consultations and referrals based principally on the best interests of their patients."

A patient suffers damage to the spinal cord severe enough to cause numbness and tingling. However, the spinal cord is not severed. The patient is most likely experiencing: a. paresis. b. hemiplegia. c. quadriplegia. d. paraplegia.

36. A-- The term "paresis" means partial paralysis. Paresis can occur as a result of an injury to the spinal cord that does not result in its severation. If the spinal cord is severed, the result could be paraplegia (paralysis of the lower limbs), quadriplegia (paralysis of all four limbs), or hemiplegia (paralysis of one side of the body), depending on the location of the lesion. Note that the term paresis (or "general paresis") has been used to describe a syphilis-caused syndrome characterized by inflammation of cerebral tissue and mental and physical deterioration. However, the term's literal meaning is partial paralysis, and this is how it is often used as well.

Linnehan's dialectical behavioral therapy is a combination of psychoeducation and cognitive therapy that involves a. intensive individual sessions. b. family therapy. c. individual and group sessions. d. group therapy.

38. C-- Linnehan uses dialectical behavioral therapy (DBT) in the treatment of Borderline Personality Disorder. DBT is a type of cognitive behavioral therapy that emphasizes social skills training, self-soothing exercises, and group dynamics. The patient is offered intensive individual and group sessions that are a combination of psychoeducation and cognitive therapy. DBT has been shown to reduce hospitalization and self-mutilating behavior (Linnehan, Heard, and Armstrong, 1993).

Which of the following is most true of Type I Schizophrenia as described by Timothy Crow? a. There will be a poor prognosis. b. The symptoms are due to a neurotransmitter abnormality. c. A majority of symptoms will begin in adolescence. d. It will not respond to most antipsychotic medications.

39. B-- Timothy Crow distinguishes between two types (I and II) of Schizophrenia. Type I includes symptoms of delusions or hallucinations, inappropriate affect, and disorganized thinking. It is also thought to more likely be due to neurotransmitter irregularities. Type II symptoms are described by the other three choices, and is also considered to be more likely due to brain structure abnormalities (T.J. Crow, The two syndrome concept: Origins and current status, Schizophrenia Bulletin, 1992, 11, 471-486).

Two predictor tests are developed. The first has a validity coefficient of .3, and the second a validity coefficient of .6. The amount of shared variability with the criterion is how many times greater in the second test as compared with the first? • 1. 1.5. • 2. 2.0. • 3. 3.0. • 4. 4.0.

4. 4.0. (correct answer, your response) To calculate the amount of shared or explained variability with a validity coefficient (or any Pearson r correlation), you must square the correlation coefficient. The first validity coefficient is .3, and .32 = .09. The second validity coefficient is .6, and .62 = .36. This question is asking how many times greater is .36 than .09, which is 4.0 times greater.

A researcher correlates income with education and obtains a correlation of .90. When she correlates income with life satisfaction the correlation is .30. It would be most accurate to say that the amount of shared variability in the first correlation is about how many times greater than the shared variability in the second correlation? • 1. 2 times. • 2. 3 times. • 3. 5 times. • 4. 9 times.

4. 9 times. (correct answer) To calculate the answer, you would first need to find the amount of shared (or explained or accounted for) variability for each correlation. To obtain the shared variability in a correlation, you simply square the correlation coefficient (this is frequently called the coefficient of determination). For the first correlation, .92 = .81; for the second correlation, .32 = .09. The shared variability in the first correlation is therefore 9 times greater than the second. (9 x .09 = .81).

A man who has been drinking for the past 25 years exhibits deficits in recall of information, difficulty learning new information, as well as difficulty recognizing familiar objects. These symptoms are characteristic of: • 1. Alcohol Intoxication Delirium. • 2. Alcohol Withdrawal Delirium. • 3. Alcohol-Induced Persisting Amnestic Disorder. • 4. Alcohol-Induced Persisting Dementia.

4. Alcohol-Induced Persisting Dementia. (correct answer) Alcohol-induced persisting dementia is characterized by memory impairment, as well as at least one other symptom of cognitive impairment, such as aphasia, apraxia, or agnosia, that are etiologically related to the effects of prolonged alcohol use. In this question, the man has deficits in memory (in recall and in learning new information), as well as agnosia (difficulty recognizing familiar objects). Alcohol-induced persisting amnestic disorder (Response 3) is characterized by impairment in memory that is a result of prolonged alcohol use. Amnestic disorder does not include any other symptoms of cognitive impairment. This disorder is also known as Korsakoff's syndrome. Alcohol intoxication delirium (Response 1) is characterized by a disturbance of consciousness and a change in cognitive abilities or perceptual disturbance that develop over a short period and fluctuate. These symptoms occur during substance intoxication only. Alcohol withdrawal delirium (Response 2) is characterized by the same symptoms as in alcohol intoxication delirium, though the symptoms occur in the course of withdrawal from alcohol. In this question, there is no indication of a disturbance in consciousness, and the man's symptoms seem to be stable and do not fluctuate; therefore, delirium can be ruled out.

What is L-Dopa or Levodopa? • 1. The neurotransmitter associated with Schizophrenia. • 2. The neurotransmitter that is implicated in the movement components of Parkinson's Disease. • 3. A novel antipsychotic that is used to treat Schizophrenia. • 4. An amino acid that is used to treat the movement components of Parkinson's Disease.

4. An amino acid that is used to treat the movement components of Parkinson's Disease. (correct answer) L-Dopa or Levodopa is an amino acid that is a precursor to dopamine. It is used to treat the movement components of Parkinson's by increasing dopamine levels, as Parkinson's Disease is associated with reduced amounts of dopamine. In the case of Schizophrenia, dopamine is present in excess. Responses 1 and 2 refer to dopamine, rather than L-Dopa. L-Dopa would be contraindicated in the treatment of Schizophrenia (ruling out Response 3). In fact, psychosis can be a side effect of L-Dopa in Parkinson's patients.

Positive and negative life events are likely to have which of the following types of effect on a person's sense of satisfaction and well-being? a. neither short-term nor long-term effects b. short-term but not long-term effects c. long-term but not short-term effects d. both short-term and long-term effects

4. B-- Research suggests that positive and negative life events can create great joy or distress but don't really affect a person's long-term sense of satisfaction and well-being. Only recent life-events seem to influence a person's well-being, and this effect drops off in about three months. For example, lottery winners have not been found to be significantly happier than non-winners.

Which of the following statements is incorrect with regard to the biology of schizophrenia? • 1. CT scans of schizophrenic and normal controls show enlarged lateral and third ventricles for the schizophrenic patients, consistent with loss of brain tissue. • 2. Brain abnormalities in schizophrenics have been found in the medial temporal lobes, frontal lobes, and diencephalon. • 3. Cerebral blood flow in the frontal lobes of schizophrenic patients is lower than that of normal controls during the Wisconsin Card Sorting Test. • 4. Frontal lobe abnormalities in schizophrenics are associated with the positive symptoms of the disorder.

4. Frontal lobe abnormalities in schizophrenics are associated with the positive symptoms of the disorder. (correct answer, your response) Responses 1 through 3 are all correct. Brains of persons with schizophrenia appear to have enlargement in the lateral and third ventricles, decreased frontal lobe activity (as measured on the PET scan), as well as brain abnormalities in several areas. Frontal lobe abnormalities have been associated with the negative symptoms of schizophrenia (e.g., flat affect, avolition, amotivation, etc.).

As validity approaches the value of 0.0, what happens to the standard error of the estimate? • 1. It approaches 1.0. • 2. It approaches 0.0. • 3. It approaches the standard deviation of the predictor. • 4. It approaches the standard deviation of the criterion.

4. It approaches the standard deviation of the criterion. (correct answer) When a test's validity approaches 0.0, the test is close to being completely invalid. In this case, the average error in predicting, or the standard error of the estimate approaches the standard deviation of the criterion (SDy). The formula for the standard error of estimate is Sest = SDy √ 1 - rxy2. The range of the standard error of estimate is from a minimum value of 0.0 to a maximum value of SDy. When a test is a perfect predictor (validity = 1.0), the standard error of estimate is 0.0, or in other words, the test has no error in prediction (Response 2). Conversely, when a test has no ability to predict (validity = 0.0), the value of the standard error of estimate is equal to the standard deviation of the criterion (SDy) (Response 4).

As validity approaches the value of 0.0, what happens to the standard error of the estimate? • 1. It approaches 1.0. • 2. It approaches 0.0. • 3. It approaches the standard deviation of the predictor. • 4. It approaches the standard deviation of the criterion.

4. It approaches the standard deviation of the criterion. (correct answer) When a test's validity approaches 0.0, the test is close to being completely invalid. In this case, the average error in predicting, or the standard error of the estimate approaches the standard deviation of the criterion (SDy). The formula for the standard error of estimate is Sest = SDy √ 1 - rxy2. The range of the standard error of estimate is from a minimum value of 0.0 to a maximum value of SDy. When a test is a perfect predictor (validity = 1.0), the standard error of estimate is 0.0, or in other words, the test has no error in prediction (Response 2). Conversely, when a test has no ability to predict (validity = 0.0), the value of the standard error of estimate is equal to the standard deviation of the criterion (SDy) (Response 4).

Because surgeons must have both high intelligence and manual dexterity, a surgery residency program might require high scores on tests measuring both traits before accepting applicants. Which of the following statements is most accurate regarding the use of a multiple regression equation in this situation? • 1. It is appropriate since the predictors are related. • 2. It is appropriate if predictors are linearly related to success. • 3. It is not appropriate since the predictors are correlated. • 4. It is not appropriate because the variables are not compensatory.

4. It is not appropriate because the variables are not compensatory. (correct answer, your response) Multiple regression is a compensatory technique. For example, when predicting college GPA, a low score on the SAT can be "compensated" for by very high grades in high school. The EPPP uses compensatory grading: a low score in research methods can be compensated for by a high score in ethics (thank goodness!). In contrast, in a multiple cutoff scenario, a person needs to reach a certain level on each variable being measured. In this example, no matter how brilliant a woman is, she couldn't succeed as a surgeon with poor manual dexterity (and vice-versa). These variables are not compensatory and therefore not appropriate for a multiple regression. Note that it is preferable for predictors in a multiple regression not to be related (the opposite of Response 1). It is true that when predictors are highly correlated in a multiple regression (Response 3), a significant problem is created, termed "multicollinearity." Nevertheless, related predictors are often used in multiple regressions. The main problem here, and the one that makes multiple regression completely inappropriate, is that the tests are non-compensatory. Multiple regression is only appropriate when predictors have a linear relationship with the criterion (Response 2).

What are the effects of random selection and random assignment? • 1. They both increase internal validity. • 2. They both increase external validity. • 3. Random selection increases internal validity and random assignment increases external validity. • 4. Random selection increases external validity and random assignment increases internal validity.

4. Random selection increases external validity and random assignment increases internal validity. (correct answer) Random selection means that every subject in the population has an equal chance of being selected for the experiment. Because it ensures that subjects are representative of the population, it improves the ability to generalize the findings of the study; in other words, it increases external validity. Random assignment involves randomly assigning subjects to different treatment groups within the experiment, as opposed to using pre-existing treatment groups (e.g., patients on two hospital wards). Random assignment helps to ensure that differences in outcome between the groups are due to treatment effects and not pre-existing differences. Thus random assignment enhances internal validity, which is the ability to conclude that the difference between the groups is actually due to the intervention. Random assignment is the hallmark of true experimental research. One way to remember these terms is the phrase: "We randomly select from the population (thereby affecting external validity), and then we randomly assign to groups within the experiment (thereby affecting internal validity)."

A woman who had a psychotic break two years ago has subsequently been exhibiting mild suspiciousness, eccentric behavior, flat affect, poor motivation, and some concrete thinking. What is the most accurate diagnosis in this situation? • 1. Schizophrenia, Paranoid Type. • 2. Schizophrenia, Disorganized Type. • 3. Schizophrenia, Undifferentiated Type. • 4. Schizophrenia, Residual Type.

4. Schizophrenia, Residual Type. (correct answer) Schizophrenia, residual type, describes someone who has had at least one episode of schizophrenia, but now there is an absence of prominent delusions, hallucinations, disorganized speech, and disorganized or catatonic behavior, while there are still some negative symptoms (e.g., poor motivation), or milder forms of the positive symptoms described above (e.g., suspiciousness, eccentric behavior). Paranoid type (Response 1) involves preoccupation with one or more delusions or auditory hallucinations. The woman in this question, while suspicious, is not exhibiting delusions or hallucinations. Disorganized type (Response 2) is characterized by all of the following: disorganized speech, disorganized behavior, and flat or inappropriate affect. These symptoms, except for flat affect, are all absent in this question, and the diagnostic criteria for this subtype are therefore not met. Undifferentiated type (Response 3) is diagnosed when there are two or more symptoms that include some combination of delusions, hallucinations, disorganized speech, disorganized or catatonic behavior, and negative symptoms, yet the criteria for Paranoid, Disorganized, or Catatonic Type are not met.

Which of the following statements about expectancy tables is most accurate? • 1. They allow you to estimate the expected criterion score based on knowledge of the predictor score and the criterion-related validity coefficient. • 2. They allow you to estimate the expected criterion range based on knowledge of the predictor range. • 3. They allow you to determine the expected incremental validity when base rate, selection ratio and criterion-related validity are known. • 4. They allow you to determine the expected chance of scoring within a criterion range based on knowledge of the predictor range.

4. They allow you to determine the expected chance of scoring within a criterion range based on knowledge of the predictor range. (correct answer) Expectancy tables indicate the likelihood (expressed in terms of percentages or proportions) that given a test taker's score on a predictor, he/she will score within a given interval on the criterion. Incremental validity, base rate, and selection ratio (Response 3) are used in the Taylor-Russell scales. These scales analyze the amount of improvement in selection decisions that result from using a particular predictor test as opposed to no test at all.

A person is given a list of words to memorize. Right after she is presented with the words, she is told to do serial sevens, or count backwards by sevens. Next, she is instructed to recite as many words as she can remember from the list. Which of the following describes the purpose of having her count backwards? • 1. To assess her immediate attention and concentration. • 2. To assess the impact of proactive interference. • 3. To assess the extent to which the information memorized undergoes decay. • 4. To prevent her from rehearsing the words.

4. To prevent her from rehearsing the words. (correct answer)

A child is reinforced on average after every three minutes for on-task behavior while completing her homework. This is an example of what schedule of reinforcement? • 1. Fixed interval. • 2. Fixed ratio. • 3. Variable ratio. • 4. Variable interval.

4. Variable interval. (correct answer)

A client presents with speech that is rambling and incoherent. Her most likely diagnosis is: • 1. Alzheimer's disease. • 2. Korsakoff's syndrome. • 3. Broca's aphasia. • 4. Wernicke's aphasia.

4. Wernicke's aphasia. (correct answer) Wernicke's aphasia is a receptive language disorder (a disorder of language comprehension) that is due to damage in the superior temporal gyrus (Brodmann's area 22). It results in fluent yet incoherent speech; it can be mistaken for psychosis. Alzheimer's disease (Response 1) is not known for fluent incoherent speech. Korsakoff's Syndrome (Response 2), a memory disorder secondary to chronic alcohol abuse, is known for the presence of confabulations; such patients fill in gaps in their memory with made-up scenarios. Broca's aphasia (Response 3) is a disorder of the motor production of speech that is due to damage in the frontal lobe (Brodmann's area 44). It produces "broken speech," speech that is slow and ungrammatical with missing words.

In taking the licensing exam, two friends, Sarah and Lee score at the 50th and 90th percentile ranks, respectively. If five points are added only to Sarah and Lee's scores one would expect to find: • 1. an equivalent increase in Sarah and Lee's percentile ranks. • 2. no change in Sarah and Lee's percentile ranks, relative to other licensure candidates. • 3. a smaller change in Sarah's percentile rank relative to Lee. • 4. a smaller change in Lee's percentile rank relative to Sarah's.

4. a smaller change in Lee's percentile rank relative to Sarah's. (correct answer) If you visualize a normal curve, you'll see that the majority of scores cluster around the mean. If you move one score even a small amount, the percentile rank shifts in relationship to the other ranks very significantly. In contrast, at the farther end of the curve, there are relatively few scores. Moving one score several points will change the percentile rank only slightly. Another way to approach this question is to use what you know about the relationship between standard deviations and percentile ranks in the normal curve. Moving up just one standard deviation from the mean shifts someone from the 50th percentile to the 84th percentile, an increase of 34 ranks. By contrast, moving up one standard deviation further (to two standard deviations above the mean) only shifts the person up to the 97th percentile rank, an increase of 13 ranks. Thus, when scores are closer to the mean, adding or subtracting points make a much more substantial change to the percentile rank than when scores are far from the mean.

All of the following are commonly treated using principles of reciprocal inhibition except: • 1. specific phobias. • 2. smoking. • 3. paraphilias. • 4. agoraphobia.

4. agoraphobia. (correct answer) The treatment of choice for agoraphobia is flooding, which is based on the principle of extinction; the person confronts the feared situation (CS) until it no longer triggers an anxiety response. Reciprocal inhibition underlies counterconditioning, which states that two incompatible responses (e.g., fear and relaxation) cannot occur at the same time. Specific phobias (Response 1) are commonly treated with systematic desensitization, a form of counterconditioning, although treatments for specific phobias that involve direct exposure (e.g., flooding) are superior. Smoking (Response 2) and paraphilias (Response 3) are treated with aversive conditioning, also a form of counterconditioning.

A patient presents with Obsessive-Compulsive Disorder, multiple motor tics, and one vocal tic. The neurological basis for this patient's problems can probably be found in the: • 1. temporal lobes. • 2. amygdala. • 3. prefrontal cortex. • 4. basal ganglia.

4. basal ganglia. (correct answer, your response) This patient appears to have both OCD and Tourette's Disorder (diagnosed when there are multiple motor tics and one or more vocal tics). Tic Disorders and Obsessive-Compulsive Disorder are frequently co-morbid, and both are thought to be linked to problems in the basal ganglia. The basal ganglia consist of a group of nuclei deep within the brain that are involved in the initiation of movement. In Tourette's Disorder, specifically, the caudate nucleus appears to be smaller in children with Tourette's, and diminished caudate nucleus size is predictive of tic severity. Several other nuclei in the basal ganglia are implicated in Tourette's Disorder, including the substantia nigra, the ventral striatum, and the globus pallidus. The basal ganglia receive instructions from the prefrontal cortex (Response 3), which is the foremost section of the frontal lobes and is sometimes known as the "executive center" of the brain. The temporal lobes (Response 1) house the auditory cortex. The amygdala (Response 2), part of the limbic system, is involved in emotion, particularly aggression.

Data on two variables are collected. The data on the first variable are dichotomous, and the data on the other variable are continuous. To analyze the relationship between the two, the statistic of choice is: • 1. eta. • 2. phi. • 3. Pearson r. • 4. biserial.

4. biserial. (correct answer, your response) The question is asking for a bivariate test of relationship (or correlation), and all the choices listed are from this category. In order to respond to this question you need knowledge of which correlational tests are appropriate for what type of data. Eta (Response 1) is used when both variables are interval/ratio or continuous, and the relationship is curvilinear (e.g., the relationship between performance and arousal). Phi (Response 2) is used when both variables being correlated are true dichotomies (e.g., the relationship between sex and eye color). Pearson r (Response 3) is the correlation coefficient when both variables are continuous. As in this example, biserial coefficient is appropriate when one variable is an artificial dichotomy (e.g., depressed/not depressed) and the other variable is continuous (e.g., IQ). Incidentally, the point biserial coefficient is appropriate when one variable is a true dichotomy (e.g., male/female) and the other variable is continuous (e.g., IQ). Note that in this case the type of dichotomy is not specified and the biserial is the only possible answer among those given.

Perry's studies of bullying have shown that: • 1. most children who demonstrate high levels of aggression before the age of eight will grow out of their aggression. • 2. early childhood aggression is related to later criminal behavior but not to later child abuse. • 3. bullying is most often a normative and non-traumatic childhood event. • 4. chronic aggression by a young child is correlated with adult criminal behavior and family violence.

4. chronic aggression by a young child is correlated with adult criminal behavior and family violence. (correct answer) Perry's research has revealed that children who demonstrate chronic aggressive behavior before the age of eight are more likely to exhibit criminal behavior as adults (ruling out Response 1). They also engage in family violence to a greater extent, including abuse of children (ruling out Response 2). Perry does not believe that bullying is normal behavior (Response 3), but rather that it is a precursor to serious behavior problems in the bully, and additionally, that bullying may cause serious emotional problems in the victim.

Meta-analysis involves: • 1. combining the results of different studies into an effect size while assigning greater weight to studies that are of better quality. • 2. reanalyzing data from different studies and combining the results into an effect size. • 3. comparing different studies and calculating an effect size without taking into consideration how each study was conducted. • 4. combining the results of studies into an effect size.

4. combining the results of studies into an effect size. (correct answer, your response) Meta-analysis is a statistical procedure that is used to look at "average" outcome over a broad range of studies (e.g., whether psychotherapy is more effective than no treatment). Meta-analysis involves selecting relevant studies and combining the results to calculate an effect size. Before conducting a meta-analysis, certain criteria are established for the studies to be included in the analysis (e.g., based on the type of design of the study, or how each study was conducted) (Response 3). Meta-analysis, however, does not assign any greater weight to studies that are of better quality (Response 1). Also, meta-analysis does not reanalyze the original data from each study (Response 2), but simply combines the results of the studies into an effect size.

A researcher finds moderate correlations utilizing heteromethod, monotrait. This finding lends support to: • 1. construct validity. • 2. discriminant validity. • 3. concurrent validity. • 4. convergent validity.

4. convergent validity. (correct answer) This question deals with the multitrait-multimethod matrix, which is used to establish construct validity (Response 1) for a test. Specifically, it establishes construct validity by establishing convergent validity (i.e., the test correlates with other instruments that purport to measure the same construct) and discriminant validity (i.e., the test does not correlate with instruments that purport to measure different constructs). Correlations of heteromethod, monotrait mean that the test correlates with other measurement instruments (heteromethod) that purport to measure the same trait (monotrait); thus, these correlations point to convergent validity. Discriminant validity (Response 2) would occur with low correlations of monomethod, heterotrait, meaning that there are low correlations between the test and similar types of measurement instruments (monomethod) that purport to measure different traits (heterotrait). Concurrent validity (Response 3) is a type of criterion-related validity and occurs when a test correlates well with a concurrent criterion (e.g., a diagnostic test of correlates with current clinical diagnosis).

You have been treating Andrew for chronic pain and insomnia secondary to a work-related injury. Andrew has filed for worker's compensation, and you receive a notice of deposition and subpoena for his therapy records from the defense attorney. You should: • 1. appear at the deposition and release the subpoenaed records as requested. • 2. appear at the deposition and assert privilege on the client's behalf. • 3. discuss the situation with Andrew and then decide whether or not to appear at the deposition. • 4. discuss the situation with Andrew.

4. discuss the situation with Andrew. (correct answer) You have received a notice of deposition and subpoena for your client's therapy records. You cannot ignore this notice of deposition and subpoena (ruling out Response 3). Your first step should be to discuss with Andrew whether he does or does not want his records released (Response 4). Until you have had this discussion with Andrew you would not release the records as requested (Response 1), nor assert privilege (Response 2).

Factor analysis may be performed using oblique or orthogonal rotations. The main advantage of an oblique rotation is that: • 1. it is more easily interpreted. • 2. it permits the computation of communalities. • 3. it has greater power. • 4. factors are usually correlated in the real world.

4. factors are usually correlated in the real world. (correct answer, your response) Factor analysis is a test of structure or fit. It seeks to reduce complex correlations to an underlying set of explanatory factors. It can be performed using an orthogonal or oblique rotation. In an orthogonal rotation, the factors are considered not to be correlated. This lack of correlation makes the analysis easier to interpret (Response 1). It also enables the computation of communalities (Response 2). The communality of a test refers to how much variance in the test has been explained by all the factors. In an oblique analysis, the factors are considered to be correlated. This correlation makes the analysis harder to interpret but more accurately reflects the real world (Response 4). Neither the orthogonal nor the oblique rotation has more power (Response 3).

According to Rehm's self-control model of depression, depression results from: • 1. a lack of perceived control in one's environment. • 2. a lack of real control in one's environment. • 3. a lack of external reinforcement and high rates of self-punishment. • 4. high rates of self-punishment and low rates of self-reinforcement.

4. high rates of self-punishment and low rates of self-reinforcement. (correct answer) Rehm attempts to integrate behavioral and cognitive theories by proposing that reinforcement can be self-generated rather than derived from external sources. Rehm views depression and its concomitant low rate of behavior (e.g., lack of involvement in activities) as a result of negative self-evaluations, lack of self-reinforcement and high rates of self-punishment.

According to Sue & Sue's Racial/Cultural Identity Development Model, a client who is more comfortable with his or her own race and less comfortable with others is likely to be in the stage of: • 1. dissonance. • 2. introspection. • 3. conformity. • 4. immersion.

4. immersion. (correct answer, your response) In the stage called resistance and immersion, the culturally different person experiences a strong sense of identification with, and commitment to, his or her minority group, and rejects the dominant values of society and culture. In the conformity (Response 3) stage, which is the first stage of this model, the minority person is distinguished by unequivocal preference for the dominant culture's values and the tendency to adopt negative stereotypes about his or her own minority culture as well as all other minority groups. In the dissonance stage (Response 1), the person experiences a growing awareness that not all values of the dominant group are beneficial, which leads to questioning and challenging of attitudes and beliefs. In the introspection stage (Response 2), the person begins to discover that the level of intensity of negative feelings directed toward the majority culture is draining, and also recognizes that many elements of the majority culture are highly functional and desirable.

To optimize discriminant validity you would want: • 1. moderate to high correlations with heteromethod, monotrait. • 2. moderate to high correlation with monomethod, heterotrait. • 3. low to moderate correlations with heteromethod, monotrait. • 4. low to moderate correlation with monomethod, heterotrait.

4. low to moderate correlation with monomethod, heterotrait. (correct answer) This question deals with the multitrait-multimethod matrix, which is used in general to establish construct validity for a test. Specifically, it establishes construct validity by establishing convergent validity (i.e., the test correlates with other instruments which purport to measure the same construct) and discriminant/divergent validity (i.e., the test does not correlate with instruments which purport to measure different constructs). Correlations of heteromethod, monotrait mean that the test correlates with other measurement instruments (heteromethod) that purport to measure the same trait (monotrait); thus, these correlations would point to convergent validity (Response 1). Discriminant/divergent validity would occur with low correlations of monomethod, heterotrait (Response 4). In other words, there are low correlations between the test and similar types of measurement instruments (monomethod) that purport to measure different traits (heterotrait).

To optimize discriminant validity you would want: • 1. moderate to high correlations with heteromethod, monotrait. • 2. moderate to high correlation with monomethod, heterotrait. • 3. low to moderate correlations with heteromethod, monotrait. • 4. low to moderate correlation with monomethod, heterotrait.

4. low to moderate correlation with monomethod, heterotrait. (correct answer) This question deals with the multitrait-multimethod matrix, which is used in general to establish construct validity for a test. Specifically, it establishes construct validity by establishing convergent validity (i.e., the test correlates with other instruments which purport to measure the same construct) and discriminant/divergent validity (i.e., the test does not correlate with instruments which purport to measure different constructs). Correlations of heteromethod, monotrait mean that the test correlates with other measurement instruments (heteromethod) that purport to measure the same trait (monotrait); thus, these correlations would point to convergent validity (Response 1). Discriminant/divergent validity would occur with low correlations of monomethod, heterotrait (Response 4). In other words, there are low correlations between the test and similar types of measurement instruments (monomethod) that purport to measure different traits (heterotrait).

A person states that she is depressed and anxious because of the current political and economic climate. According to Bronfenbrenner, the environmental influence on this woman is the: • 1. microsystem. • 2. mesosystem. • 3. exosystem. 4. macrosystem.

4. macrosystem. (correct answer)

According to classical test theory, test scores usually differ from true scores. The most likely explanation for this phenomenon is: • 1. instruments that are unreliable. • 2. a combination of nonsystematic and systematic error. • 3. differences in ability levels of test takers. • 4. nonsystematic error.

4. nonsystematic error. (correct answer) According to classical test theory, total variance in scores is equal to true variance (reliability) and error variance. In equation form, this is expressed as X = T + E. Systematic error (Response 2) refers to a type of experimental error that affects all scores in an identical manner. In terms of reliability, anything that is "systematic" would not decrease the consistency of the results and would therefore not be of concern. Unreliability of instrumentation (Response 1) is only one of many factors that could account for the difference between true and obtained test scores. Differences in ability of test-takers (Response 3) are not a hindrance to reliability in any way. In fact, the wider the range of difference in ability, the higher the level reliability will be (all other things held constant). This is true because the reliability coefficient, like all coefficients, tends to be larger when the range of values is less restricted.

Developed by Marsha Linehan, Dialectical Behavior Therapy requires patients to agree to all of the following conditions except: • 1. to work on reducing self-injurious and parasuicidal behavior. • 2. to work on reducing therapy-interfering behaviors. • 3. to attend all sessions within reason and remain in therapy for a specified period of time (e.g., one year). • 4. not to call the therapist outside of the regularly scheduled psychotherapy session.

4. not to call the therapist outside of the regularly scheduled psychotherapy session. (correct answer) Telephone contact is actually one of the primary modes of Dialectical Behavior Therapy, along with individual therapy, group skills training, and therapist consultation. All of the other responses describe conditions that patients must agree to at the onset of therapy. In addition, patients must agree to attend skills training classes, usually focusing on the four skills of core mindfulness, interpersonal effectiveness, emotion modulation, and distress tolerance.

Some children as young as six months old enjoy the game of hide and seek, while newborns do not. This can be attributed to the development of: • 1. assimilation. • 2. accommodation. • 3. object constancy. • 4. object permanence.

4. object permanence. (correct answer) With the development of object permanence, the child understands that objects or people continue to exist even when the child cannot see them. With this knowledge, the child can play hide and seek, or look for something or someone who cannot be seen. Assimilation (Response 1) is the process of taking in a new experience through the person's already established mental structure. Accommodation (Response 2) involves adjusting to reality demands by reorganizing or modifying the existing cognitive structure or scheme. Object constancy (Response 3), a term associated with Margaret Mahler, refers not only to the ability to maintain the image of the mother when she is absent (object permanence), but also to the ability to unify the good and bad aspects of the mother into a whole representation.

To test for a relationship between two variables while controlling for a third, the test of choice would be: • 1. ANCOVA. • 2. LISREL. • 3. factor analysis. • 4. partial correlation.

4. partial correlation. (correct answer, your response) This question actually gives the definition of partial correlation: a statistical procedure that investigates the relationship between two variables while controlling ("partialing out") the relationship of a third variable. LISREL (Response 2), also a test of relationship, is a sophisticated type of structural equation modeling that can determine if a given model of relationships among variables is correct. Factor analysis (Response 3), a test of structure, is conducted when there are a priori hypotheses about the underlying structure for a given set of variables. ANCOVA (Response 1) is somewhat similar to partial correlation, except that ANCOVA is used exclusively when partialing out the effect of a confound from research that is looking for differences between groups, not relationships among variables.

A test of math abilities was administered to most sixth graders in the United States. The results indicated statistically significant differences (p < .001) in math abilities for males and females, yet the difference between the means was actually very small. The best explanation for this finding would be: • 1. sampling error. • 2. a test with low reliability. • 3. systematic error. • 4. sample size.

4. sample size. (correct answer) Questions like this one crop up occasionally on the EPPP so it is worth understanding. Whenever sample size is large, there is a greater likelihood of finding statistical significance. As sample size grows larger, statistical significance can be found for even very small differences between group means. Keep in mind that there is a difference between statistical significance (i.e., results are not due to chance) and clinical significance (i.e., results are of a meaningful and relevant magnitude).

A test of math abilities was administered to most sixth graders in the United States. The results indicated statistically significant differences (p < .001) in math abilities for males and females, yet the difference between the means was actually very small. The best explanation for this finding would be: • 1. sampling error. • 2. a test with low reliability. • 3. systematic error. • 4. sample size.

4. sample size. (correct answer) Questions like this one crop up occasionally on the EPPP so it is worth understanding. Whenever sample size is large, there is a greater likelihood of finding statistical significance. As sample size grows larger, statistical significance can be found for even very small differences between group means. Keep in mind that there is a difference between statistical significance (i.e., results are not due to chance) and clinical significance (i.e., results are of a meaningful and relevant magnitude).

A main characteristic of Mahler's practicing subphase is: • 1. vacillation to and fro. • 2. stranger anxiety. • 3. symbiosis. • 4. separation anxiety.

4. separation anxiety. (correct answer) Practicing (10-16 months) is the second of the four subphases into which Mahler divides the major phase of separation-individuation. It is marked by walking and separation anxiety. Vacillation to and fro (Response 1) is associated with the third subphase of rapprochement (16-24 months). Stranger anxiety (Response 2) is associated with the first subphase of differentiation (5-10 months). Symbiosis (Response 3) occurs at from 3-4 weeks to 4-5 months and precedes separation-individuation.

In a multiple baseline across subjects design, treatment is applied: • 1. concurrently to the subjects. • 2. to half the subjects, while the other half serve as wait-list controls initially. • 3. in alternating sequence with baselines. • 4. sequentially to the subjects.

4. sequentially to the subjects. (correct answer) Multiple baseline is a single subject design in which treatment is applied sequentially (consecutively, successively) across subjects, situations, and behaviors. Wait-list controls (Response 2) are only an issue when implementing group designs. Treatment applied in alternating sequences with baseline (Response 3) describes as ABAB design.

A researcher is interested in whether there are differences between four different teaching styles for students of three different SES levels. The F ratio for teaching style (A) = 4.7*. The F ratio for SES (B) = 1.7. The F ratio for the Interaction (AxB) = .9. *= p < .05. It is safe to conclude that: • 1. significant differences for teaching style and SES were found. • 2. significant differences for SES and a significant interaction effect were found. • 3. a significant interaction effect was found. • 4. significant differences for teaching style were found.

4. significant differences for teaching style were found. (correct answer) While this data may look complicated, it is actually not that difficult to interpret. In any two way ANOVA, three F ratios are calculated, one for each main effect (e.g., teaching style or SES), and one for the interaction. To interpret an ANOVA for significance you need to look at the F ratios; the F ratio(s) that is/are starred are significant, and the significance level is described (e.g., p<.05). So in this example the only significant F ratio is for teaching style.

A researcher is studying differences between men and women in their ability to reduce cigarette use. Number of cigarettes smoked daily is measured pre-treatment, at the end of treatment and at a six-month follow-up. To analyze the results the researcher should use a: • 1. t-test • 2. one-way ANOVA • 3. factorial ANOVA • 4. split-plot ANOVA

4. split-plot ANOVA (correct answer) This research is looking for differences between men and women, and over time in terms of cigarette consumption (the DV). There are two independent variables (IVs), gender and time. Gender has two levels or groups (males and females) and the data are independent. Time has three levels or groups (pre, end, six-month follow-up), and the data are correlated. A t-test (Response 1) can only be used when there is one IV, with no more than two groups. A one-way ANOVA (Response 2) can also only be used when there is one IV, with two or more groups. A factorial ANOVA (Response 3) is an ANOVA that has two or more IVs, with the data on each IV being independent. The split-plot ANOVA is used when there are two (or more) IVs, and at least one has correlated data, and one has independent data.

Two programs designed to improve academic achievement are compared by measuring students at the beginning, middle and end of the school year. The statistic of choice is a: • 1. t-test. • 2. one-way ANOVA. • 3. factorial ANOVA. • 4. split-plot ANOVA.

4. split-plot ANOVA. (correct answer, your response) This experiment is a mixed design, consisting of two IVs. One variable is the repeated measures variable of time because the same subjects are being measured more than once (beginning, middle, and end of school year). The data collected on this variable are correlated. The other variable is a between-groups variable of program (e.g., Program 1 versus Program 2) where the data collected are independent. The split-plot ANOVA is the appropriate statistic in this situation. A t-test (Response 1) is only appropriate when there is only one IV, and only two groups are being compared. A one-way ANOVA (Response 2) can't be used when there are two IVs. A factorial ANOVA (Response 3) means that there is more than one IV, all the IVs are between-groups variables, and the data are independent.

A multivariate analysis of variance would be used to analyze collected data when: a. the researcher wants to analyze the effects of an extraneous variable b. the researcher wants to remove the effects of an extraneous variable c. the study includes two or more independent variables d. the study includes two or more dependent variables

40. D-- The multivariate analysis of variance (MANOVA) is a type of ANOVA used when two or more dependent variables are included in a study. Rather than using separate ANOVAs to evaluate the effects of each of the dependent variables, a researcher could use the MANOVA when all the dependent variables are measured on a ratio or interval scale. This also helps to control the experiment-wise error rate.

Working Memory is an expansion of the WISC-III Freedom from Distractibility factor. It is comprised of which of the following scales? a. Coding, symbol search, digit span b. Arithmetic and digit-span c. Perceptual organization, comprehension, similarities d. Digit-span, picture arrangement, coding

41. B-- The Working Memory scale is comprised of the WISC-III subtests of Arithmetic and Digit-Span and measures numerical ability, attention, and concentration

When undertaking token economies with seriously disturbed individuals in mental institutions, one of the major problems with the program's efficacy has to do with a. generalization of behaviors. b. choice of reinforcers. c. exchange ratio. d. reinforcement value.

42. A-- Use of a token economy involves administering secondary reinforcers such as a token each time the person engages in a desired behavior, or taking away a reinforcer when a person engages in an undesired behavior. The tokens can then be exchanged for primary reinforcers such as food or desired activities. Token economies are commonly used in institutional settings. A problem with them is that behaviors learned often fail to generalize to the real world, since tokens are not available in the real world every time we do something right.

When looking at an item characteristic curve (ICC), which of the following provides information about how well the item discriminates between high and low achievers? a. the Y-intercept b. the slope of the curve c. the position of the curve (left versus right) d. the position of the curve (top versus bottom)

47. B-- An item response curve provides one to three pieces of information about a test item - its difficulty (answer C); its ability to discriminate between high and low scorers (answer B); and the probability of answering the item correctly just by guessing (answer A).

An intervention targeting Bronfenbrenner's microsystem will focus on a. family members. b. family members and/or classmates. c. school and church. d. parents' work and friends.

48. B-- Bronfenbrenner's ecological model distinguishes between four interacting environmental levels that range from the most proximal to the most global. The microsystem is the individual's immediate environment and includes parents, siblings, caregivers, classmates, and teachers.

According to Lenore Walker, which of the following best describes the dynamic that keeps battered women "hooked" into their relationship with the batterer? a. Due to having grown up in an abusive family, the victim believes that battering is a normal part of relationships. b. The costs of the abuse and the benefits of remaining in the relationship are about equal. c. The woman fears that the abuse will become even more severe if she tries to leave the relationship. d. The woman lacks knowledge about the resources available to help her leave the relationship.

5. B-- Lenore Walker describes a cycle of violence that involves three stags: tension building, acute battering incident, and loving contrition. According to Walker, most of the benefits of the relationship occur in the third stage, when the batterer offers apologies, assurances that the attacks will never happen again, and declarations of love. The relationship tends to remain stable when the balance between the costs of the abuse and the benefits of the relationship are fairly similar. As violence escalates, the relationship becomes more unstable, and the man escalates his charming behavior in an attempt to restore stability.

Your managed care company denies to pay for further sessions for a client and requests that you do not tell the client of its decision. In this situation, you should a. terminate with the client as requested. b. continue to provide treatment to the client if you believe the client could benefit from treatment. c. inform the client of all aspects of this decision that are relevant to treatment. d. seek the advice of an attorney regarding a possible lawsuit against the managed care company.

50. C-- This question is not so much about managed care as it is about informed consent procedures. Whether or not a patient's treatment is being financed by a managed care company, a psychologist should inform clients of relevant information regarding treatment at the outset of the professional relationship and continue to provide such information as it arises throughout treatment. Due to the nature of managed care companies, there may be more concerns that need to be discussed with managed care patients than with other clients. But the general informed consent requirement applies to this situation as well as many others. The other choices don't really address the issue raised by the question. Both choices "A" and "B" describe two possibly acceptable responses to this situation, but neither are requirements -- whether or not you would continue to see the patient would depend on a number of factors, including the patient's needs and your policies. Choice "D" implies that the managed care company's actions are illegal and have caused you harm, but the question does not provide enough information for you to reasonably conclude that this is true.

According to Self Verification Theory, a person who believes that he dances poorly would prefer to be told by friends: a. "You are a pretty bad dancer" b. "You are not a bad dancer" c. "You should become a professional dancer" d. nothing about dancing

51. A-- Self Verification theory proposes that people need and seek confirmation of their self-concept, regardless of whether their self-concept is positive or negative. Thus, people prefer to be right rather than happy. According to this theory, a person who dances poorly would prefer to be told so (assuming the other's evaluation matches the person's self evaluation).

According to current research, the best predictor(s) for alcoholism would be a. family history of alcoholism. b. environmental stresses and opportunities for observational learning. c. interpersonal pressure and identifications. d. age and SES.

52. A-- If you wanted to find the best single predictor, you'd find out about alcoholism in the natural relatives of the patient. Even if the person is adopted away from the natural parents, the genetic connection is still the strongest one we have. So, when doing an initial assessment, you could ask about alcohol/drug abuse among family members. By the way, a good guess in answering any question structured as "the best predictor of (some disorder)" is "family history of that disorder." You won't be right 100% of the time, but you will be the majority of the time.

All of the following statements regarding item response theory are true, except a. it cannot be applied in the attempt to develop culture-fair tests. b. it's a useful theory in the development of computer programs designed to create tests tailored to the individual's level of ability. c. one of its assumptions is that test items measure a "latent trait." d. it usually has little practical significance unless one is working with very large samples.

53. A-- Item response theory is a highly technical mathematical approach to item analysis. Use of item analysis is based on a number of complex mathematical assumptions. One of these assumptions, known as invariance of item parameters, holds that the characteristics of items should be the same for all theoretically equivalent groups of subjects chosen from the same population. Thus, any culture-free test should demonstrate such invariance; i.e., a set of items shouldn't have a different set of characteristics for minority and non-minority subgroups. For this reason, item response theory has been applied to the development of culture-free tests, and choice A is not a true statement. The other choices are all true statements about item response theory, and therefore incorrect answers to this question. Consistent with choice B, item response theory is the theoretical basis of computer adaptive assessment, in which tests tailored to the examinee's ability level are computer generated. As stated by choice C, an assumption of item response theory is that items measure a latent trait, such as intelligence or general ability. And, finally, research supports the notion that the assumptions of item response theory only hold true for very large samples (choice D).

Which of the following is most consistent with the research regarding childhood sexual abuse? a. the effects are less severe for younger children compared to older children b. the effects are less severe the older the perpetrator is at the time of the abuse c. the effects are less severe when the perpetrator is a family member d. the effects are less severe when the perpetrator is a stranger

54. D-- Childhood sexual abuse often results in long-term consequences for the victims; however, certain factors have been found to moderate the severity of consequences. For example, the effects are generally less severe when the perpetrator is a stranger rather than a family member. However, approximately 70% of all incidents of childhood sexual abuse are committed by a family member.

56. According to Horn and Cattell (1966) a. crystallized and fluid intelligence become more integrated in later adulthood. b. crystallized and fluid intelligence are uncorrelated. c. crystallized intelligence develops through the use of fluid intelligence. d. fluid (but not crystallized) intelligence depends on exposure to particular cultural and educational experiences.

56. C-- Cattell and Horn distinguish between fluid and crystallized intelligence. The former is independent of specific instruction and is relatively culture free, while the latter depends on exposure to education and is affected by cultural experiences. Cattell and Horn believed that these two aspects of intelligence are highly correlated and that crystallized intelligence develops through the use of fluid intelligence.

57. Curriculum-based assessment is a. a form of norm-referenced assessment designed to compare the performance of students to other students who are receiving the same type of instruction. b. used to evaluate a curriculum in order to identify how it should be improved or changed. c. used to compare the curriculum at an educational institution to that of other institutions with similar students and goals. d. used to evaluate student performance in light of the current curriculum and identify possible modifications in instructional style.

57. D-- Curriculum-based assessment provides information about a student in the context of the existing curriculum. The results of such an assessment not only indicate the student's performance level -- they also provide feedback about the instruction itself, so that necessary changes can be made to better fit the student's ability and current knowledge. In contrast to choice A, curriculum-based assessment is criterion-referenced, since it involves evaluating the student's performance in light of an external standard (the curriculum). Norm-referenced assessment would involve evaluating a student's performance by way of a comparison to other students. In contrast to choice B, the assessment is not designed to bring about changes in the curriculum itself -- rather, its purpose is to help identify progress in terms of the existing curriculum and any change in instruction (e.g., pace, sequencing of topics) that would aid the student's progress in completing the curriculum.

58. If you want to keep a borderline patient involved with group therapy, what would be the best technique? a. You have them consult with a psychiatrist for a medication evaluation. b. You help the patient with the defense mechanism of splitting. c. You invite a client with a histrionic personality disorder to join the group. d. Offer individual therapy in addition to group therapy.

58. D-- Marsha Linehan (1993) has been achieving success with borderline patients with her use of dialectical behavioral therapy (DBT) which involves a combination of groups skills training and individual outpatient therapy. This combination has been successful at decreasing premature dropout rates in group therapy, as well as reducing suicide attempts and inpatient hospitalization rates.

59. The difference between professional ethics and professional values is best stated by which of the following? a. If a psychologist is ethical, there should be no difference between ethics and values. b. Ethics are standards for practice set by the profession, while values refer to judgments of right and wrong. c. Ethics can be stated more specifically than values. d. Values specifically define appropriate professional conduct, while ethics are a more general code of proper professional conduct.

59. B-- Response B distinguishes between ethics and values. Values are concerned with what is good and desirable and ethics refer to correct or appropriate practice. Values and ethics are related in that the latter are usually derived from the former; for example, privacy is a value that is reflected in the ethical standard requiring psychologists to obtain clients' informed consent before releasing information about therapy. Responses A and C are not necessarily true, and answer D is not true since values do not specifically define appropriate professional conduct.

60. There is evidence that the effectiveness of systematic desensitization for reducing anxiety is actually due to repeated exposure to the feared stimulus, which leads to extinction of the anxiety response. However, systematic desensitization was originally developed as an application of a. negative reinforcement. b. counterconditioning. c. stimulus discrimination. d. avoidance conditioning.

60. B-- In systematic desensitization, anxiety-arousing stimuli are paired with stimuli that produce an incompatible response (often relaxation). In other words, it was designed to use counterconditioning in order to eliminate an anxiety response. Some research suggests, however, that it is actually just the exposure to anxiety-arousing stimuli, without aversive consequences, that explains the effectiveness of this technique.

61. The basic requirements of a token economy are: a. stimulus sensitization, choice of tokens, rate of exchange. b. target behaviors, choice of reinforcers, rate of exchange. c. goal setting, staff cooperation, choice of reinforcers. d. target behaviors, choice of tokens, primary reinforcers.

61. B-- To institute a token economy program, you need to know the behaviors you want to change (the target behaviors). You also need to know what is reinforcing for the client (choice of reinforcer). For a hospitalized schizophrenic, it might be walking around; for a child, it might be a candy treat. You also need to know the relationship between token and the reinforcer (rate of exchange); that is, how many tokens will purchase the reinforcer.

62. When a multiple regression analysis is employed to predict outcome, there should be a. low intercorrelations among the predictors and high correlation of each predictor with the criterion. b. high intercorrelations among the predictors and high correlation of each predictor with the criterion. c. low intercorrelations among the predictors and low correlation of each predictor with the criterion. d. high intercorrelations among the predictors and low correlation of each predictor with the criterion.

62. A-- This question has come up in other examples throughout the tests. Simply stated, we need to have a high correlation between the predictor and the criterion we're making predictions about (this eliminates two of the four alternatives). Also, we need to have the predictors themselves be more or less independent of each other. That is, they shouldn't intercorrelate. If they do, then there's no point in using all of them -- if they all measure the same thing, why not use just one? So, you don't want the predictors to intercorrelate.

64. According to Kohlberg's theory, which of the following would be the best example of conventional morality? a. maximizing individual gains b. support of social standards c. behaving solely in line with one's own conscience d. behaving in such a way as to avoid risk and cost to the individual

64. B-- Conventional morality is the second level of Kohlberg's three levels of moral reasoning. It is between pre-conventional morality, where the behaviors we view as moral are those that result in a reward or avoidance of punishment; and post-conventional morality, where we function according to our own conscience, no matter what the conventional wisdom is. The person at the conventional level supports the social order. He or she believes in that which is approved of by others or by societal standards is moral.

66. According to social learning theory, job training would be most effective when using: a. behavioral modeling b. vestibule training c. reinforcement on a variable interval schedule d. reinforcers tailored to each employee's needs

66. A-- According to Bandura's social learning theory (or theory of observational learning), it is possible to learn a given behavior merely by watching a model perform it. Behavioral practice and reinforcement, though they influence the probability that a behavior will be learned, are not absolutely necessary for the behavior to be imitated. Therefore, Choices C and D, which involve reinforcement procedures, can be eliminated. Vestibule training (B) involves a simulation of work conditions, although it does not necessarily involve modeling.

67. Wernicke's, Broca's and conduction aphasia share which of the following difficulties? a. repeating what is said b. word prosody c. reception d. expression

67. A-- Difficulty repeating words just spoken and recalling the name of familiar objects are characteristic of all three disorders. Conduction aphasia is due to damage to the nerve fibers that connect Broca's to Wernicke's area and the most typical result is difficulty repeating what one has heard. Wernicke's, or receptive aphasia, primarily affects comprehension which results in impairment in spoken and written language, and anomia or problems recalling words. Broca's aphasia is characterized by difficulty expressing language, including difficulty repeating what is said.

69. A psychologist who obtained a Ph.D. in experimental psychology wants to change her specialty to clinical psychology. To meet the requirements set forth by the General Guidelines for Providers, the psychologist must a. complete an internship in clinical psychology under the supervision of a professional clinical psychologist. b. complete appropriate doctoral-level classes and supervised post-doctoral training. c. obtain a Ph.D. in clinical psychology from an accredited college or university. d. meet her state's requirements for licensure in clinical psychology.

69. B-- The licensing exam often contains questions such as these, and in approaching them, you should remember these buzzwords: training AND experience. Specifically, the Specialty Guidelines state that "professional psychologists who wish to qualify as clinical psychologists meet the same requirements with respect to subject matter and professional skills that apply to doctoral and postdoctoral education and training in clinical psychology." Choice "B" is the only one that includes an element of both training and experience, and is therefore the best answer. Moreover, unlike as stated by choice "C", a second Ph.D. would not be necessary -- doctoral level coursework in clinical psychology would be sufficient to meet the academic training aspect of this requirement.

70. The drug most often abused by teenagers is a. alcohol. b. tobacco. c. marijuana. d. cocaine.

70. A-- According to the latest survey completed by the U.S. Department of Health and Human Services (who keep statistics on these types of things) of teenagers aged 12-17, alcohol was their drug of preference. Asked about their drug use in the previous month, 21% had consumed at least one alcoholic drink, 18.2% had smoked cigarettes and 8.3% had used marijuana.

71. An elevated F score on the MMPI-2 (i.e., the F is greater than 70 and the K is very low) indicates that the: a. person is answering in a socially desirable manner. b. score should be considered in relation to the total profile. c. person is likely being careless or deliberately malingering. d. total profile can be considered valid.

71. C-- The validity scales on the MMPI-2, of which the F is one, are checks on response styles. Specifically, the F scale indicates if the person is answering in a deviant way, or is perhaps actually deviant. The higher the F, the more the answers suggest that the person is attempting to appear odd, disturbed, etc.

72. Premature termination would be most likely to occur when a. an African-American client has a Caucasian therapist. b. an African-American client has an African-American therapist. c. a Caucasian client has an African-American therapist. d. a Caucasian client has a Caucasian therapist.

72. A-- Research with African-American clients on the relationship between therapist-client racial similarity and therapy effectiveness has yielded contradictory results and suggests that this relationship is mitigated by a variety of factors. A clearer relationship exists between similarity and premature termination, with the probability of premature dropout increasing when an African-American client works with a racially dissimilar therapist.

73. In the context of expectancy theory, valence refers to: a. the willingness of a worker to exert effort. b. the strength of the worker's needs. c. the desirability of the job itself. d. the desirability of the consequences of performance.

73. D-- Expectancy theory predicts that motivation is related to three phenomena: beliefs about the relationship between effort and performance; beliefs about the relationship between performance and outcomes; and the desirability of those outcomes. The latter is referred to as valence.

74. The validity coefficient of a new job selection test is 0.25. This test would most likely be useful when: a. the percentage of correct hiring decisions without the new test is very low b. the percentage of correct hiring decisions without the new test is very high c. there are many applicants for few job openings d. there are few applicants for many job openings

74. C-- This question is referring to a test's incremental validity. Incremental validity is the increase in decision-making accuracy resulting from the use of a particular predictor. Three factors influence the incremental validity of a test: 1) the base rate, or the percentage of correct hiring decisions made when the test is not used; 2) the test's validity coefficient; and 3) the selection ratio, or the ratio of job openings to total applicants (for instance, if 100 people are applying for 5 positions, the selection ratio is 5/100, or .05). Incremental validity is greatest when the base rate is moderate, the validity coefficient is high, and the selection ratio is low. The Taylor-Russell tables can be used to determine a test's incremental validity, given specific values for the base rate, validity coefficient, and selection ratio. We can eliminate Choices A and B because a moderate base rate maximizes the usefulness of a selection test more than a low or high base rate does. Contrary to Choice D, a low selection ratio (many applicants for few job openings) is better than a high selection ratio.

75. You are a psychologist in private practice and, due to a heavy case load, you decide to work with another psychologist. The second psychologist will see many of the clients and share office space with you. You agree to give this psychologist 50% of all fees received. This arrangement is a. unethical, because the ethical standards prohibit fees for referrals. b. unethical, because it threatens the best interests of the clients. c. unethical, because the clients will be misled about their fees. d. ethical.

75. D-- There is nothing inherently unethical about this arrangement, and it is not barred by APA's ethical standards. The standards do prohibit payment for referrals that are based on the referral itself and not based on services rendered. However, both psychologists in this case will apparently be rendering services, so the arrangement appears acceptable.

76. A man is able to achieve erections during sleep, but, has difficulty achieving or maintaining erections during sexual activities. The most appropriate diagnosis would be: a. Hypoactive Sexual Desire Disorder b. Male Erectile Disorder c. Sexual Aversion Disorder d. Sexual Dysfunction Not Otherwise Specified

76. B-- Male Erectile Disorder is characterized by a persistent or recurrent inability to attain or maintain an erection until completion of sexual activity. This best fits the description in this question. Hypoactive Sexual Desire Disorder (A) is diagnosed when a person has persistently deficient or absent sexual fantasies and desire for sexual activity. Since the question did not indicate that the man in this question lacked sexual desire, we should not assume that to be the case. Although, if a lack of desire was also present, both diagnoses would be given. There also is no evidence of extreme aversion to, or avoidance of, all genital contact with a sexual partner - which eliminates Sexual Aversion Disorder (C). And Sexual Dysfunction NOS (D) is reserved for sexual dysfunctions that do not meet the criteria for any specific sexual dysfunction.

77. Severe maternal malnutrition during the third trimester is correlated with a number of intellectual, social, and motor deficits in children. Which of the following statements regarding the physiological correlates of these deficits is most true? a. Maternal malnutrition in the third trimester is most likely to result in incomplete development of the autonomic nervous system. b. Maternal malnutrition in the third trimester seems to have its most severe negative impact on the developing brain. c. Maternal malnutrition in the third trimester is most likely to cause physical disabilities that prevent the child from developing at a normal rate. d. Maternal malnutrition in the third trimester is not likely to have a significant physiological effect on the developing fetus; observed deficits in these children are probably due to social and environmental causes.

77. B-- Severe prenatal malnutrition is likely to have differential effects, depending on when in pregnancy it occurs. In the first trimester, it can result in congenital malformations and spontaneous abortion. In the third trimester (as well as in the first 3-6 months after birth), it is most likely to have a negative effect on the central nervous system -- specifically, the brain. Studies have suggested that these children often have an abnormally low number of brain cells and brain weight. The specific behavioral consequences may include apathy, unresponsiveness to environmental stimulation, irritability, an abnormally high-pitched cry, intellectual deficits, and lags in motor development.

78. One of the most consistent findings of the research on client factors in the therapy process is that a. more seriously disturbed clients have better outcomes in therapy. b. higher levels of intelligence predict better therapy outcomes. c. age is significantly related to therapy outcome. d. men are more likely to seek therapy.

78. B-- One of the most consistent findings of the research on client factors is that higher levels of intelligence predict better therapy outcome (Luborsky, 1989). Answer A is incorrect because seriously disturbed individuals actually have poorer outcomes (Garfield, 1994). In regard to answer C, age appears to be unrelated to therapy outcome and answer D would have to state that women are more likely to seek therapy to be correct (Luborsky et al., 1989).

79. When a psychologist believes that her patient's records will be used in a legal proceeding, she should: a. rewrite them to meet the higher standards of a forensic setting b. avoid the use of any psychological terminology in favor of legal terminology c. maintain them in the same kind and quality as all of her patients' records d. maintain them in the kind and quality consistent with reasonable scrutiny in an adjudicative forum

79. D-- According to the Specialty Guidelines for Forensic Psychologists (1991), "When forensic psychologists conduct an evaluation or engage in the treatment of a party to a legal proceeding, with foreknowledge that their professional services will be used in an adjudicative forum, they incur a special responsibility to provide the best documentation possible under the circumstances."

According to M. Seligman's theory of learned optimism, a student with an optimistic attribution style who fails an exam in a class which he usually does well in is most likely to say: a. "I was unlucky" b. "I didn't study enough" c. "the teacher is always a tough grader" d. "the test was hard this time"

8. D-- In Seligman's theory of learned optimism, attributions of optimistic people are believed to be the opposite of attributions of depressed people. Since depressed people make internal, stable, and global attributions to negative events, optimistic people would tend to make external, unstable, and specific attributions in response to negative events. Therefore, we can readily eliminate "B" ("I didn't study enough") since that's an internal attribution. Choice "C" ("the teacher is always a tough grader") is a stable attribution. That leaves Choices "A" ("I was unlucky") and Choice "D" ("the test was hard this time") - which are both external and unstable attributions. Of the two, however, Choice "D" is better since being unlucky would imply that success is a matter of luck.

80. The realization that gender is unaffected by superficial changes in appearance or activity is referred to as: a. gender constancy b. gender stability c. gender identity d. ego-dystonic transvestism

80. A-- Gender concept develops in a predictable sequence of stages during childhood. The first stage is known as "gender identity," which is the ability to categorize self and others as male or female. There is evidence that gender identity develops as early as 9 months or as late as 3 years, depending on how the researchers define it. The next stage, which develops by 4 years of age, is "gender stability," which is an understanding that one's gender does not change over time. The final stage is known as "gender constancy," which is characterized by an understanding that gender stays the same despite changes in appearance. Gender constancy is achieved by age 5 or 6.

81. The friend of a psychologist is the owner of a small publishing firm. The friend offers the psychologist substantial compensation if she would lend her name to endorsements for his publications, and at times advise and consult on book acquisitions and marketing plans. The psychologist: a. could accept this as long as APA affiliation is not mentioned in the endorsements. b. could accept this only if the publications she would be dealing with are in her area of competency. c. could accept this only if the publications she would be dealing with are in her area of competency, and the endorsements will be based on the psychologist's honest opinion of the publications. d. should consult the local ethics committee before making her decision.

81. C-- The ethical standards say that psychologists cannot make false, deceptive, fraudulent, or misleading statements. Unless the psychologist's endorsements were based on sound expertise and her honest opinion of the publications in question, these standards would be violated.

82. Overall, research on the compressed work week suggests that its effects are: a. increased job satisfaction, with no effect on job performance. b. increased job performance, with no effect on job satisfaction. c. decreased job performance, with no effect on satisfaction. d. increased job performance and increased job satisfaction.

82. A-- In the compressed work week (CWW), the number of days worked is shortened but the work hours per day are lengthened. For example, the work week might be compressed into four days a week, ten hours a day. Research results regarding the effects of the CWW are mixed, but most studies suggest that job satisfaction tends to increase, while job performance is not affected and (depending on the job) may even decrease. The CWW also tends to result in decreased absenteeism.

83. Of the following ethnic groups which group is not considered a race? a. Hispanic b. Native American c. African-American d. Asian-American

83. A-- Hispanics are an ethnic group like African-Americans, Asian-Americans, and Native Americans. Unlike these groups, Hispanics are not a specific race. According to the 1998 Census Bureau, the total Hispanic population of the US accounts for 11% of the total population. Sixty-three percent are of Mexican origin, 14.4% are of Central and South American origin, 10.6% of mainland Puerto Rican origin, 4.2% of Cuban origin, and 7.4% other (Alfredo Ardilis, Testing Hispanic Populations, Texas Psychologist, Winter 2000).

85. The California Tarasoff Statute: a. Changed the Ethics Code in Canada and the United States b. Extended the duty to warn clause c. Was necessary as issues implicated by the Tarasoff case account for a majority of forensic cases d. Applies to everyone working in a public setting

85. B-- The California Tarasoff statute gives clinicians explicit guidance about when a duty to act arises and tells clinicians what actions fulfill their duty. The first Tarasoff decision established a "duty to warn," however, the case was reheard several years later and the Tarasoff II decision modified the duty to warn to a "duty to protect." Choice C contradicts the conclusion by Behnke, Preis and Bates in "The Essentials of California Mental Health Law" Norton and Company, 1998, pg. 9 in which they state, "issues implicated by the Tarasoff case and its legal progeny account for a small percentage of forensic cases."

86. In children, positive outcome following a traumatic event is most likely if the child: a. has adequate social support and a high level of skill in at least one activity. b. has a high level of intelligence. c. is male. d. has experienced a traumatic stressor.

86. A-- In both children and adults, high levels of social support is one of the best predictors of positive psychological outcome following a traumatic event. Having skill in at least one activity is associated with self-efficacy beliefs, which are also associated with positive outcome following a trauma in both children and adults.

87. From the perspective of psychoanalytic theory, mania represents: a. a biological illness that psychoanalysis can do nothing about. b. acting-out of libidinous impulses. c. a regression to an infantile state. d. a defense against depression.

87. D-- According to classical psychoanalytic theory, mania occurs as a defense against depression, due to an inability of the person to tolerate or admit to being depressed.

88. In terms of self-concept and social relationships, early physical maturity has been found to have which of the following effects for adolescents? a. positive effects for both males and females b. positive effects for males but not for females c. positive effects for females but not for males d. negative effects for both males and females

88. B-- The effects of the timing of physical maturation are different for males and females. Boys seem to benefit from early maturation (early maturers are more popular and do better in school), while early-maturing girls experience a number of negative outcome (e.g., shyness, low self-esteem).

89. A Hypomanic Episode would most likely result in: a. decreased productivity, decreased efficiency, and decreased creativity b. decreased productivity, decreased efficiency, and increased creativity c. increased productivity, decreased efficiency, and decreased creativity d. increased productivity, increased efficiency, and increased creativity

89. D-- According to DSM-IV-TR, a Hypomanic Episode is characterized by a period of persistently elevated, expansive, or irritable mood that lasts at least 4 days and is accompanied by three or more symptoms including: inflated self-esteem or grandiosity; decreased need for sleep; talkativeness; racing thoughts; distractibility; excessive involvement in high-risk pleasurable activities; and increase in goal-directed activity. The symptoms are not severe enough to cause marked impairment in functioning and there are no psychotic features. During a Hypomanic Episode many individuals exhibit increased productivity, efficiency, and creativity.

To determine whether or not the doctrine of comparable worth is lived up to in an organization, one would rely on a. the perceptions of employees. b. a job description. c. a job evaluation. d. a task analysis.

9. C-- The doctrine of comparable worth states that workers (in particular, men and women) should get equal pay for performing jobs that have equivalent worth. A job evaluation is a method of determining the worth or value of jobs in an organization. Thus, it is the best answer to this question, because it would be a way of comparing the value of different jobs. Note that many critics believe that current methods of job evaluation contain inherent gender biases. Statistics clearly show that women earn less than men, and some believe that one reason for this is that job evaluation methods undervalue the work that women perform.

90. Beck's cognitive therapy includes all of the following characteristics except: a. it is referred to as "collaborative empiricism." b. it is structured and goal-oriented. c. relapse prevention is a focus throughout. d. it is based on an elaboration likelihood model.

90. D-- Beck's model includes all of the characteristics except an elaboration likelihood model, which is a cognitive model of attitude change that predicts that persuasion can occur in one of two ways--through a central and peripheral route (Petty, 1994). Beck's model is referred to as collaborative empiricism (answer A) because it is founded on a collaborative relationship between the therapist and client, and involves gathering evidence and testing hypotheses about the client's beliefs.

91. A client who claims that the disk jockey on a local rock n'roll radio station is speaking directly to him most likely: a. is experiencing a systematized delusion. b. is experiencing a delusion of reference. c. is experiencing a persecutory delusion. d. has heard the same song over and over again on the local "classic rock" station one too many times.

91. B-- A delusion of reference is one in which the person believes that objects or events in the immediate environment have an unusual and particular significance to him or her. In a persecutory delusion, the person feels that he or she is being mistreated in some way or conspired against; this is not the case here. Systematized delusions are organized around a coherent theme; there is no evidence that this is the case. Hearing the same song over and over again can drive one crazy, but probably not to the extent described in the question.

93. When men and women are asked to judge photos of women's faces: a. attractive women are viewed as less intelligent. b. there is no relation between ratings of beauty and personality. c. unattractive women are rated as having less positive personalities. d. women and men disagree on standards of facial beauty.

93. C-- Researchers find a consistently moderate positive correlation between ratings of facial beauty and ratings of personality and intelligence, regardless of whether the faces are female or male. This is an example of the halo effect. In other words, attractive people are rated as having better personalities and unattractive people are rated less positively.

94. You are an organizational process consultant hired by a hospital. Several nurses tell you they feel they're being overworked and underpaid. In this case, your most likely intervention would be to a. explain the nurses' concerns to the hospital administrators. b. conduct a job analysis to determine how much the nurses deserve to be paid and present your results to the hospital administrators. c. act with the best interests of the hospital's patients in mind. d. help the nurses organize regular meetings with the physicians and administrators to discuss their concerns.

94. D-- A process consultant, rather than merely correcting problems for his or her client, focuses on an organization's underlying processes, with the goal of making changes in processes so that individuals within the organization can solve their own problems. Thus, choice D is the best answer, as it is the only one which offers a way in which the nurses and administrators can work together to solve the problem of dissatisfaction with work conditions. By comparison, choices A and B would have the consultant diagnose and "fix" the problem; these interventions would be consistent with the medical rather than the process model of consultation.

95. According to Margaret Mahler, individuation involves a. developing relationships with significant others. b. developing an interest in the outside world. c. coming to recognize the existence of self and others. d. coming to recognize that Mother does not cease to exist when she disappears from sight.

95. C-- According to Mahler, a child must separate and individuate so that he or she can move from being part of a mother-child unit to being a member of a family. Separation occurs when the child disengages and differentiates from the mother, while individuation occurs when the child develops an inner representation of the mother, an ability to test reality, a sense of time, and an awareness of the existence of others as distinct from him or herself. In other words, when the child individuates, he or she moves away from symbiosis with the mother and toward the recognition and experience of his or her individual existence.

96. You belong to a managed-care panel and a client with a 12-session limit was referred to you. Without calling to cancel, she does not show up for her fourth session. How would you bill for the missed session? a. bill the insurance company b. bill the client according to office policies c. bill the client for the co-payment and the insurance company for the no-show d. you cannot bill either the client or insurance company

96. B-- The best answer is that you should bill according to your office policies which, presumably, was explained and agreed to by the client at the beginning of treatment (Standard 6.04: Fees and Financial Arrangements). If charging the client for the missed session was not agreed upon in advance, this would be unethical (which response "D" suggests). Responses "A" and "D" are correct in stating that you cannot bill the insurance company, since you should only bill insurance for services that you actually provide (Standard 6.06: Accuracy in Reports to Payors and Funding Sources). While your financial agreement may or may not require the client to cover the co-payment for missed sessions (response "C"), Current Procedural Terminology (CPT) codes do not provide a code for no-shows, and billing for services not rendered is unethical

97. Complex Partial Seizure Disorder, formerly known as Temporal Lobe Epilepsy, is known to originate in the temporal lobe. However, absence seizures, formerly known as petit mal seizures, are believed to originate in the: a. cerebellum b. thalamus c. occipital lobe d. parietal lobe

97. B-- Absence or Petit Mal Seizures are very brief (30 seconds or less) and are characterized by minimal motor activity and a lack of awareness. Absence seizures usually begin in childhood and most outgrow the condition by adulthood. Researchers believe that absence seizures originate in the thalamus (the central relay station for sensory information going to the cortex).

98. According to McClelland's Need for Achievement theory, individuals high in Need for Achievement will prefer: a. difficult tasks which bring very high acclaim when achieved. b. moderately difficult tasks which are both attainable and difficult enough to satisfy the person's need to feel a sense of accomplishment. c. easy tasks which are very easy to achieve and therefore provide immediate gratification of the person's needs. d. any task which gives the person the opportunity for recognition and control over others.

98. B-- According to McClelland's theory, three needs underlie motivation: need for achievement, need for power, and need for affiliation. Individuals high in need for achievement tend to seek autonomy, personal responsibility, and recognition for their efforts. Since the need for achievement is balanced against a need to avoid failure, these individuals tend to prefer moderately difficult goals and tasks.

99. The part of the brain that is compromised in Parkinson's Disease is the a. basal ganglia. b. substantia nigra. c. hypothalamus. d. frontal lobe.

99. B-- The substantia nigra is a group of neurons that is part of the extrapyramidal motor system. Parkinson's disease, which is characterized by tremors and other disorders of voluntary movement, is caused by degeneration of substantia nigra neurons. When substantia nigra neurons degenerate, the amount of dopamine they provide to neurons in the basal ganglia (also part of the extrapyramidal motor system) is reduced. In other words, basal ganglia neurons are also involved in Parkinson's disease -- the disease occurs when they get a lower than normal quantity of dopamine. Thus, if substantia nigra were not a choice, the basal ganglia would be the best answer. Note, however, that, unlike substantia nigra neurons, basal ganglia neurons remain intact in Parkinson's disease.

A person with a temporal lobe tumor is most likely to exhibit which of the following? a. memory impairment, depression or anxiety, and hallucinations. b. poor judgment, impulsivity, and inappropriate jocularity. c. confabulation, irritability and suspiciousness, and prosopagnosia. d. dysgraphia, depression and apathy, and denial of his or her problems.

a

During your first session with a 38 year old man, he tells you that he is homosexual and that he found out, several weeks ago, that his partner of six years is having an affair and is planning on moving out of their house. The man says that he is very nervous and anxious, that he wishes he wasn't a homosexual, and that he lays in bed at night obsessing about his partner. His anxiety has affected his work: He is having trouble concentrating, is not getting along well with his co-workers, and has "called in sick" several times in the past two weeks. Based on these symptoms, the best diagnosis is. a. Adjustment Disorder with Anxious Mood. b. Bereavement. c. Ego-Dystonic Homosexuality. d. P.T.S.D..

a

Recent research suggests that people who watch a lot of television. a. fear their environment more than people who report being less frequent TV viewers. b. fear their environment somewhat less than people who report being less frequent TV viewers. c. fear their environment much less than people who report being less frequent TV viewers. d. are about equal in terms of fear of their environment to people who report being less frequent TV viewers.

a

Research suggests that, in contrast to normal-weight people, overweight people eat in response to. a. external cues. b.hunger pains, which are more frequent in overweight people. c. sugary types of foods. d. stressful events.

a

The primary purpose of the state licensing boards is to. a. protect the public from poorly trained and incompetent psychologists. b. supervise the training of psychologists. c. keep a limit on the number of licenses granted yearly. d. uphold APA's ethical practices guidelines.

a

When conducting an experiment, a researcher sets alpha at .01. On the basis of the results of the t-test she uses to analyze her data, the researcher concludes that her results are "significant at the .01 level." This means that. a. there is a 1% probability that she has falsely rejected the null hypothesis. b. there is a 1% probability that she has falsely retained the null hypothesis. c. there is a 99% probability that she has correctly rejected the null hypothesis. d. there is a 99% probability that she has correctly retained the null hypothesis.

a

When using the Halstead-Reitan, an examinee's performance on seven measures (e.g., the Category Test, the Seashore Rhythm Test, and the Finger Oscillation Test) is used to derive a or an. a. Impairment Index, which can be used to determine if an examinee has brain damage. b. Impairment Index, which can be used to determine if an examinee is suffering from a functional disorder. c. Pathology Profile, which allows the examiner to compare the examinee's score profile to those of people with and without brain damage. d. Pathology Profile, which helps the examiner determine the appropriate DSM-IV diagnosis.

a

When working with a Japanese client in therapy, during the first session, you decide that your primary goals are to normalize the client's problems and instill a sense of hope. According to Sue and Zane (1987), these goals are. a. examples of "giving". b. ways of establishing ascribed credibility. c. ways of fostering "amae". d. expressions of cultural knowledge.

a

You notice that a friend of yours gets upset every time someone is standing less than 12 feet away from him or her. Which of the following characteristics is your friend most likely to possess. a. low self-esteem. b. female gender. c. a disdain of violence. d. low levels of authoritarianism.

a

118. One of the most influential theories in the field of attitude change is Leon Festinger's "cognitive dissonance theory." Based on the principles of this theory, it can be predicted that. a. the less an individual suffers to obtain something, the more he/she will like it once it has been obtained. b. to relieve dissonance after choosing one of two equally attractive alternatives, an individual will attribute greater attractiveness to the chosen alternative. c. to relieve dissonance after choosing one of two equally attractive alternatives, an individual will attribute his or her decision to dispositional factors. d. voluntary participation in an undesirable activity will result in greater dislike for the activity.

b

A Jungian "archetype" is best described as. a. an organized constellation of feelings, thoughts, and perceptions. b. a structural component of the collective unconscious. c. the "mask" adopted by a person in response to social demands. d. a significant historical event.

b

A child living in the United States is exposed to both English and a second language between the ages of six months and three years and, as result, becomes fluent in both languages. If the child had not been exposed to the second language until after the age of 4 or 5, she would have had more trouble acquiring the second language. This provides evidence for. a. the concept of critical periods. b. the concept of sensitive periods. c. the Whorfian hypothesis. d. the notion of imprinting.

b

A primary distinction between Freud and those psychologists who are collectively identified as "neo-Freudians" (e.g., Fromm, Homey, and Sullivan) is that the latter. a. emphasized the "innate wisdom" of the individual. b. placed greater emphasis on the ego functions and the impact of social influences. c. extended the number of and placed greater emphasis on instinctual (unconscious) forces. d. adopted a teleological approach that viewed behavior as being "pulled" rather than "pushed".

b

A psychologist who obtained a Ph.D. in industrial psychology wants to change her specialty to clinical psychology. To meet the requirements of APA's guidelines regarding a change in specialty, the psychologist must. a. complete an internship in clinical psychology under the supervision of a licensed clinical psychologist. b. complete appropriate doctoral-level classes and supervised post-doctoral training. c. obtain a second Ph.D. in clinical psychology from an accredited university or professional school. d. obtain appropriate supervision of her clinical practice.

b

A test developer designs an arithmetic achievement test to be used with junior high school students. When assessing the test's reliability, the test developer is likely to get an overestimate of its reliability if. a. the test contains items that are primarily of a moderate difficulty level. b. the examinees in the sample are more heterogeneous in terms of arithmetic achievement than the examinees the test was designed for. c. the test is a speeded test and the test developer uses the alternate forms method to assess its reliability. d. all of the above.

b

A woman is told that she has a serious illness for which there is no cure and that it is likely that she will die within six months. From the perspective of Kubler-Ross' stages that dying people pass through, you would expect the woman's first reaction to finding out about her prognosis to be. a. "Why me?" b."The test results must be inaccurate." c. "I must be being punished for something I did." d. "I hate my life anyway, so what difference does it make?"

b

An effective intervention for hyperactive children would probably include which of the following. a. the child will decide on appropriate tasks and rewards for completing tasks. b. the parents will set-up a behavioral routine and enforce it consistently. c. both parents and child will determine appropriate punishments for misbehaviors. d. the parents will determine a behavioral routine but be flexible in its enforcement.

b

Based on your knowledge of the statistics on suicide, you would predict that a client would be at the highest risk for suicide if. a. he is between the ages of 17 and 27. b. he has expressed a desire to kill himself. c. he has a personality disorder. d. his depressive symptoms have recently worsened.

b

Because they share a number of symptoms, it is often difficult to distinguish between Dementia and pseudodementia of Major Depressive Disorder in elderly patients. The presence of which of the following symptoms, however, is more suggestive of Major Depressive Disorder. a. patient gives confabulated answers to questions. b. patient complains about and is distressed by memory loss. c. patient exhibits apathy and loss of interest in usual activities. d. patient exhibits frequent incontinence.

b

Conduct Disorder frequently co-occurs with ADHD. Which of the following is not true about children with both disorders? a. They have higher rates of social maladjustment than children with ADHD only. b. They have more extensive cognitive difficulties and poorer school achievement than children with ADHD only. c. They have poorer long-term outcomes and a higher risk of adult psychopathology than children with ADHD only. d. They often have a less favorable response to stimulant medication than children with ADHD only.

b

Dr. Prach, a licensed psychologist, is consulting on staff at a teaching hospital. Several psychology interns come to her to complain that they are being excluded from doing the duties they were promised at the beginning of their internships. Dr. Prach would be most helpful if she. a. reports the administrators of the internship program to the ethics committee. b. explores options the interns have in making their needs known to the administrators. c. suggests that she "go to bat" for the interns and try to convince the administrators to change their minds. d. tells the interns that she should not get involved in their dispute.

b

From the perspective of Gestalt therapy, an optimal "contact boundary" is semipermeable and coterminous with the actual self. Introjection is one type of boundary disturbance. It occurs when. a. the contact boundary is located far in the environment. b. the contact boundary is located deep within the self. c. the contact boundary is turned back against the self. d. the contact boundary is extremely rigid.

b

In a research study in which it is necessary to use deception, subjects are not told the exact nature of the study prior to their participation. According to APA's Ethics Code, in this situation, the investigator is obligated to. a. explain why subjects were not informed in the presentation of his research findings. b. explain the true nature of the study to all subjects as soon as possible. c. file a waiver of informed consent with the Ethics Committee prior to beginning the study. d. do none of the above as long as the study is determined to be of significant scientific value.

b

In discussing a depressed client, a therapist says the client's problems are due to the fact that she did not experience normal grief following the death of her mother. He says that the focus of treatment will be on helping the client go through the grieving process and restore her interests and friendships so that she can overcome her loss. Apparently, this therapist is a practitioner of. a. rational-emotive therapy. b. interpersonal therapy. c. object-relations therapy. d. reality therapy.

b

In family therapy, the adolescent daughter complains that she doesn't think she can change and she feels that doing what the therapist has requested is pointless. The therapist responds by saying, "Well, in that case, I think you shouldn't even try." The therapist's response is an example of. a. reactance. b. restraining. c. positioning. d. prescription.

b

Rehm's self-control therapy begins with self-monitoring, which involves maintaining a daily log of. a. positive activities. b. automatic thoughts. c. distressful thoughts. d. antecedent events.

b

Research on Maslow's need hierarchy theory has demonstrated all of the following except. a. line managers perceive greater fulfillment of needs than staff managers, especially esteem and self-actualization needs. b. older workers have greater deficiencies in fulfilling esteem and self-actualization needs than younger workers. c. race seems to be related to fulfillment of needs with minority managers experiencing less fulfillment than non-minority managers. d. managers who work for small companies are less deficient in meeting their needs than managers at large companies, especially at lower levels of management.

b

Which of the following would be least likely to increase the power of a statistical test. a. increasing the reliability of the dependent variable measure by decreasing its susceptibility to random error. b. changing the level of significance (alpha) from .05 to .01. c. using a one-tailed test (assuming it is appropriate to do so). d. increasing experimental variance by increasing the magnitude of the independent variable effect.

b

A client has missed the last three appointments and has not paid for the past five. You receive a letter from a state in-patient facility requesting information about the client who is now a patient there. The request includes a valid release of information. According to the Ethics Code, you are acting ethically in this situation if you. a. notify the client that you will release the information only after an acceptable arrangement has been made for his unpaid bill. b. agree to release only a summary of your records until an acceptable arrangement has been made for his unpaid bill. c. provide the hospital with the information it has requested. d. use your own discretion as to the best course of action.

c

A psychologist is consulted by a medical doctor who is concerned about a patient in a pain clinic. The patient constantly asks for his pain medication and the nurses, who get tired of his barrage of requests, give him too much medication. To get the man to cut down on his requests, the psychologist is most likely to recommend. a. giving him a sedative to keep him quiet. b. giving him his medication after every twenty requests. c. giving him his medication once every four hours. d. giving him higher but less frequent doses of medication.

c

A researcher investigating the chemical correlates of memory would be most likely to find that administration of which of the following would improve the memory of elderly adults. a. L.G.N. b. T.H.E. c. R.N.A. d. G.A.B.A.

c

A test is developed to evaluate students' comprehension of information presented in a high school history class. When the test is administered to three classes of students at the end of the semester, all of the students perform very poorly on the test. This suggests that the test may have poor. a. concurrent validity. b. incremental validity. c. content validity. d. discriminant validity.

c

According to Sue and Zane (1987), in terms of ensuring good treatment outcomes, the most important factor in culturally-sensitive therapy is. a. demonstrating cultural knowledge. b. using culture-specific techniques. c. using techniques that establish credibility. d. being authentic and empathic.

c

Al A., a lonely, single, middle-aged psychologist and college professor, is invited to a play by a graduate student, Betty B., who Al finds sexually attractive. Betty has returned to school after a 15-year absence and has been divorced for three years. If Al accepts Betty's invitation, he will. a. be acting unethically because the Ethics Code prohibits professors from dating students. b. not be acting unethically as long as he doesn't become sexually involved with Betty. c. not be acting unethically as long as Betty is a graduate student in a department other than the one Al teaches in. d. not be acting unethically because this situation is not covered by the Ethics Code.

c

An OD change intervention is most likely to be successful when. a. an internal (rather than an external) change agent is used. b. an external change agent is used but he/she communicates directly only with the company's managers. c. an external change agent is used and he/she communicates directly with both managers and subordinates. d. an external change agent is hired to act as a representative of subordinates only.

c

An adolescent is brought to therapy by her mother who says the girl has recently become "another person." She says her daughter used to be easy-going and easy to get along with and was well-liked at school by her peers and teachers. However, now her daughter is constantly irritable, is smoking and drinking, and has started having trouble at school. When the therapist interviews the girl, he learns that she is engaging in high-risk sexual behavior, but the girl feels that nothing bad can ever happen to her. Based on these symptoms, the best diagnosis is. a. ADHD. b. Conduct Disorder. c. Bipolar 1 Disorder. d. Bipolar II Disorder.

c

An elderly client displays paranoid ideation. In addition to providing a safe, stable environment, treatment for this client would most likely include which of the following. a. ECT. b. peer counseling. c. a phenothiazine. d. methylphenidate.

c

Bill B., a 34-year old small business owner, has trouble completing tasks at home and work, frequently "blows his fuse" at family members and employees, and has his wife do all the paperwork at the office because he doesn't have the patience for it. His wife says Bill never listens to her and often does things without thinking about the consequences. When asked about his past, Bill says that he often got in trouble while he was in school. He frequently got into fights and, in high school, was arrested several times for drugs and drunk driving. All through school, his teachers said he wasn't living up to his potential. Although Bill no longer uses illegal drugs, he often drinks heavily on weekends. Based on these symptoms, the most likely diagnosis for Bill is. a. Antisocial Personality Disorder. b. Borderline Personality Disorder. c. ADHD. d. Bipolar Disorder.

c

From the perspective of Rational Emotive Therapy. a. irrational beliefs are acquired primarily through social learning processes. b. irrational beliefs are acquired largely through the process of selective reinforcement. c. people are biologically prone to the acquisition of irrational beliefs. d. people adopt irrational beliefs as the result of early traumatic events that are still unresolved.

c

One problem associated with subjective measures of performance is that they are susceptible to certain biases or errors. For example, the leniency bias occurs when an evaluator consistently overrates all of the employees being appraised. Which of the following is the most effective method for reducing such appraisal errors. a. graphic checklists. b. Likert scales. c. forced-choice scales. d. Guttman scales.

c

To ensure that a consent to treatment form is truly an "informed" consent, a therapist should. a. make sure that the client reads the entire form before signing it. b. warn the client of the potential risks involved in signing the form. c. make sure the client understands the information contained in the form before he or she signs it. d. warn the client of the potential limits of confidentiality.

c

When deciding on an intervention plan for a client suffering from agoraphobia, it is important to keep in mind that which of the following seems to be the key element in treating this disorder. a. counterconditioning. b. progressive relaxation. c. in vivo exposure. d. cognitive restructuring.

c

When faced with an "undifferentiated family ego mass," a family therapist who has adopted a Bowenian approach would be most likely to do which of the following. a. help differentiate family members by becoming emotionally triangulated into family dyads. b. help differentiate family members by using behavioral and cognitive-behavioral techniques that promote individuation. c. work first with the most differentiated family member to help him or her achieve greater individuation. d. work first with the "identified patient" to help him or her become more differentiated.

c

When using the technique known as the Premack Principle, the reinforcer is. a. applied intermittently. b. a generalized conditioned reinforcer. c. a behavior that occurs frequently. d. a stimulus that naturally elicits the desired behavior.

c

Which of the following is likely to be one of a child's first words. a. ball. b. give. c. cold. d. doggy.

c

Which of the following medical conditions is least likely to be a cause of an anxiety disorder due to a general medical condition. a. thyroid dysfunction. b. epilepsy. c. diabetes mellitus. d. malignancy.

c

You have just started seeing a couple in conjoint therapy and believe that the husband is physically abusing his wife. Most likely you will. a. wait for the husband or wife to bring up the issue of abuse. b. say that you want to see the wife in individual therapy. c. ask the couple how they deal with conflict and disagreement. d. ask the couple to sign a "no-violence" contract.

c

You've been seeing a client for eight months. During that time, he has complained several times about the course of therapy and, each time, you have discussed this matter with him and modified treatment goals. During the last three sessions, he has again mentioned that he is not happy with therapy. As an ethical psychologist, your best course of action would be to. a. realize that his behavior is a manifestation of resistance and ignore it. b. make a clinical decision as to whether or not the client is benefitting from therapy and, if you determine that he is, encourage him to continue in therapy. c. discuss his dissatisfaction with him and the possibility of a referral to another therapist. d. tell him that, as an ethical therapist, you must terminate treatment since he does not feel he is benefitting.

c

A feminist therapist is least likely to view which of the following as an important aspect or goal of therapy. a. identifying the sociopolitical and interpersonal forces that underlie the woman's "psychopathology". b. reconstructing the therapeutic process so that the traditional pattern of dependence is not recreated in the therapeutic relationship. c. using the therapeutic relationship as an opportunity to model appropriate behavior. d. identifying and integrating the masculine and feminine aspects of the woman's personality.

d

A worker whose job consists primarily of handling customer complaints will be happier if she blames the customer's problems on. a. external, stable, and global factors. b. internal, stable, and specific factors. c. external, unstable, and global factors. d. external, unstable, and specific factors.

d

According to Erikson, at age two, the primary task of psychosocial development is to develop BLANK, and failure to successfully accomplish this task leads to BLANK. a. an ego identity; identity diffusion. b. a capacity for industry; a sense of inferiority. c. a sense of basic trust; suspicion and mistrust. d. a sense of autonomy; doubt and shame.

d

According to Freud's psycho dynamic theory, the origin of anxiety is. a. an inability to free oneself from symbiotic relationships. b. reactions to strangers during infancy. c. an inability to repress id impulses. d. an inability to resolve conflicts between the id, the superego, and reality.

d

An industrial psychologist is hired by a company to assist in the development of a selection program for assembly-line workers. Based on her knowledge of the literature in this area, the psychologist is most likely to advise the company to use which of the following selection techniques. a. interviews and personal recommendations. b. interviews and selection tests. c. selection tests and assessment centers. d. biographical information and work samples.

d

In avoidance conditioning, the initial fear that causes the avoidance response is the result of which of the following. a. negative reinforcement. b. punishment. c. stimulus generalization. d. classical conditioning.

d

In which of the following situations would an exception to the requirement for an informed consent to treatment not be an issue. a. medical or psychological emergency. b. incompetency. c. client waiver. d. court-ordered treatment.

d

When attempting to expand your private practice, it is important to keep in mind that. a. in-person solicitations are always unethical. b. solicitations of individuals receiving similar services from another professional are unethical. c. solicitations are unethical only when they include deceptive or misleading information. d. solicitations are unethical when they include deceptive information or when the person solicited is susceptible to undue influence.

d

Which of the following is most true about pregnancy prevention programs for adolescents? a. They are most effective when they are school-based and they contribute to reduced pregnancy by reducing sexual activity. b. They are most effective when they are school-based and they contribute to reduced pregnancy by increasing contraceptive use. c. They are most effective when they are community-based and they contribute to reduced pregnancy by reducing sexual activity. d. They are most effective when they are community-based and they contribute to reduced pregnancy by increasing contraceptive use.

d

You receive a subpoena duces tecum from the court requesting that you provide it with information about one of your clients. Your best course of action would be to. a. immediately provide the information requested to the court. b. obtain the client's consent and then provide the information requested to the court. c. assert the privilege not to reveal confidential information and then provide the information requested to the court. d. assert the privilege not to reveal confidential information and provide the information only after obtaining the client's consent or being ordered to do so by the court.

d

Melanie Klein

* splitting as a major defense mechanism - used when the infant has hostile feelings towards a loved object * splitting prevents object constancy

Central Route

* thinking about whatever relevant info is available and elaborating on the message's arguments

Object Relations Family Therapy

* transferences and projections between couples and family members * projection * members express dissatisfaction and try to change one another

Parametric Tests

* used when data is interval or ratio.

Non-Parametric tests

* used when data is nominal or ordinal. Less powerful b/c they use less data in the calculation

Which of the following is most relevant to equity theory. a. Rosenthal effect. b. social comparison theory. c. drive theory. d. the buffering hypothesis.

b

Balance Theory

* explains attitude change when two people have attitudes toward the same object or activity * when there is imbalance, the people involved will move toward changing their feelings

MRI vs fMRI

* MRI can visualize the structure *fMRI (functional) can visualize functions

Five Factor Theory of Personality

* OCEAN * openness * conscientiousness * extraversion * agreeblness * neuroticism

Symmetry Theory

* extends balance theory by considering the intensity of the relationship * the stronger the bond, the more intense any imbalance

AAMR

* American Association on Mental Retardation

Eyseneck's Three Dimensions of Personality

*1. Introversion/Extraversion *2. Neuroticism/Emotional Stability *3. Psychoticism

Evolutionary Theories of Development

1. All evolutionarily-influenced characteristics develop, and this requires examining not only the functioning of these characteristics in adults but also their ontogeny. 2. All evolved characteristics develop via continuous and bidirectional gene-environment interactions that emerge dynamically over time. 3. Development is constrained by genetic, environmental, and cultural factors. 4. An extended childhood is needed in which to learn the complexities of human social communities and economies. 5. Many aspects of childhood serve as preparations for adulthood and were selected over the course of evolution (deferred adaptations). 6. Some characteristics of infants and children were selected to serve an adaptive function at specific times in development and not as preparations for adulthood (ontogenetic adaptations). 7. Children show a high degree of plasticity, or flexibility, and the ability to adapt to different contexts.

Stages of Gender Development

1. Gender Roles - societal expectations for appropriate male or female behavior 2. Gender Identity - individual's perception of him/herself as male or female. 3. Gender Constancy - realization that gender does not change with dress or behavior

A patient who experiences agraphia, acalculia, right-left disorientation, and finger agnosia is most likely suffering from: • 1. Gerstmann's syndrome. • 2. Ganser's syndrome. • 3. Kluver-Bucy syndrome. • 4. Munchausen's syndrome.

1. Gerstmann's syndrome. (correct answer)

Industrial/Organizational psychologists use identical elements to: • 1. attain transfer of training. • 2. optimize programmed instruction. • 3. minimize biases in employee ratings. • 4. enhance job rotation.

1. attain transfer of training. (correct answer)

The substance most implicated in Seasonal Affective Disorder is: • 1. melatonin. • 2. serotonin. • 3. norepinephrine. • 4. dopamine.

1. melatonin. (correct answer)

You are working on developing a new test. To optimize your reliability you should: • 1. utilize a heterogeneous sample. • 2. utilize a homogeneous sample. • 3. measure subjects on the same test at two different points in time. • 4. measure subjects on two similar versions of the test at two different points in time.

1. utilize a heterogeneous sample. (correct answer)

100. The tendency to rate all using the low end of the rating scale is referred to as: a. floor effect b. contrast effect c. central tendency bias d. severity error

100. D-- Another term for the strictness bias or error is severity error. This rater bias refers to the tendency of raters to rate all ratees using the low end of the rating scale. A rater rating all individuals using the middle range of a rating scale is exhibiting the central tendency bias (c.). If the ratings of one individual is affected by the ratings given to another, then the rater bias is a contrast effect (b.). A floor effect is not a rater bias. Characteristic of a measuring instrument, floor effect refers to the inability to distinguish between individuals who have low levels of what is being measured.

126. Holland's approach to vocational guidance is based on the assumption that: a. interests motivate occupational choice b. behavior is a function of personality-environment congruence c. job satisfaction and performance are related to a match between aptitude and job requirements d. interests change in a predictable way over the course of the lifespan

126. B-- Holland's six occupational themes represent both personality and environment types. According to Holland, job satisfaction, productivity, and other behaviors are affected by the congruence or fit between a person's personality and the nature of the work environment

An advantage of using a MANOVA over multiple one-way ANOVAs is that a. the use of a MANOVA reduces the experiment-wise error rate. b. a MANOVA can be used when the study involves more than one dependent variable. c. a MANOVA is the more appropriate test when the researcher has an a priori hypotheses about the nature of the relationship between the independent and dependent variables. d. a MANOVA involves simpler mathematical calculations.

17. A-- When a study involves two or more dependent variables, data can be analyzed with either multiple (one for each dependent variable) statistical tests (e.g., multiple one-way ANOVAs) or one MANOVA. An advantage of the latter technique is that it reduces the probability that at least one Type I error (incorrect rejection of the null hypothesis) will be made. This is because the fewer statistical tests one conducts, the less likely it is that a Type I error will occur. In an experiment that involves more than one comparison, the probability of at least one Type I error is referred to as the experiment-wise error rate.

When helping his younger sister with her homework, an older brother "thinks aloud" while solving problems and gives his sister suggestions and support. This is best described as a. mentoring. b. scaffolding. c. social facilitation. d. social referencing.

21. B-- The older brother is using techniques associated with "scaffolding," which was originally described by Vygotsky and refers to the temporary support that parents, teachers, and more experienced children give to a child to help him/her do a task or acquire a skill

Children with Generalized Anxiety Disorder most frequently worry about: a. being away from home or close relatives b. being embarrassed in public c. performance in school or sporting events d. having an illness

28. C-- According to DSM-IV-TR, children and adolescents with Generalized Anxiety Disorder frequently worry about their performance or competence at school and in sporting events. They may also worry about catastrophic events like earthquakes or nuclear war. Choice A is more characteristic of Separation Anxiety Disorder. Choice B indicates Social Phobia. And Choice D is characteristic of Hypochondriasis.

According to the central limit theorem, a. as sample size increases, the shape of a sample distribution becomes more normal. b. as the size of a sampling distribution of means increases, its distribution becomes more normal. c. as sample size increases, the shape of a sampling distribution of means becomes more normal. d. as sample size increases, the shape of a sampling distribution of means approximates the shape of the population distribution.

29. C-- According to the central limit theorem, the shape of a sampling distribution of means approaches normality as sample size increases. The central limit theorem is covered in the Advanced Statistics section of your materials, and you should study it after you have a reasonably solid grasp of the material presented in the rest of the section.

Group decision-making is most likely to be adversely affected by: • 1. social loafing and social inhibition. • 2. social comparison and deindividuation. • 3. response polarization and groupthink. • 4. risky shift and diffusion of responsibility.

3. response polarization and groupthink. (correct answer)

The most effective behavioral technique to reduce a school-aged child's temper tantrums is: a. modeling b. extinction c. positive reinforcement d. negative reinforcement

30. B-- Extinction is the elimination or reduction in the frequency of a response achieved by the removal of the reinforcement maintaining the response. The extinction of temper tantrums would entail ignoring the child's behavior or not reinforcing it with attention. This technique has been found to be effective for reducing temper tantrums. Positive (c.) and negative (d.) reinforcement are used to strengthen behaviors. Modeling (a.) is used to teach new behaviors.

Thinning refers to the process of: a. switching from a fixed interval to a fixed ratio schedule b. switching from a variable interval to a variable ratio schedule c. switching from an intermittent to a continuous schedule d. switching from a continuous to an intermittent schedule

31. D-- Continuous schedules, or reinforcing every response, are associated with quick learning, satiation and extinction. The process of thinning, or switching from a continuous to an intermittent schedule, is used to increase the resistance to extinction once a behavior is established.

When you look up to a person, are influenced by that person, identify with that person and hold them in high regard, that person has: a. Expert power b. Coercive power c. Referent power d. Reward power

37. C-- Referent power is based on a person's attraction to or desire to be like the holder of power. "A" Expert power is based on the belief that the power holder has special knowledge or expertise. "B" Coercive power results from the holder's ability to punish others. And finally, "D" reward power, results from the holder's ability to reward others.

Postpartum blues occur in about _____ of new mothers. • 1. 0 - 3%. • 2. 10 - 15%. • 3. 30 - 50%. • 4. 50 - 80%.

4. 50 - 80%. (correct answer)

Family therapy based in part on social learning theory is most likely to focus on: • 1. hierarchy and boundaries. • 2. circular questioning and hypothesis formation. • 3. differentiation and lack of differentiation in the family structure. • 4. communication problems and deficient reward exchanges.

4. communication problems and deficient reward exchanges. (correct answer)

A man believes that his hands are misshapen. He is convinced that his fingers are becoming crooked, and that he must be developing arthritis. He holds this belief even though he has seen several physicians who have told him that his hands are not crooked and that he does not have arthritis. He checks his hands several times a day, and is preoccupied with the fear that his hands are becoming arthritic. These symptoms are most suggestive of: • 1. hypochondriasis. • 2. factitious disorder. • 3. obsessive-compulsive disorder. • 4. delusional disorder.

4. delusional disorder. (correct answer)

A man feels uncomfortable when he is viewed as an African-American male. According to Cross, this man's discomfort suggests he is most likely in the identity stage of: • 1. immersion-emersion • 2. internalization • 3. encounter • 4. preencounter

4. preencounter (correct answer)

In comparing the relationship between satisfaction and performance, and satisfaction and length of employment: • 1. both correlations are weak and positive. • 2. both correlations are moderate and positive. • 3. the first correlation is weak and positive, the second moderate and negative. • 4. the first correlation is weak and positive, the second moderate and positive.

4. the first correlation is weak and positive, the second moderate and positive. (correct answer)

You have been seeing a client for several months and believe that treatment has been a success. When you suggest termination to the client, he expresses a desire to continue therapy. Both you and the client are unable to come up with additional treatment goals; however, the client states that he really looks forward to coming to therapy just to talk. In this situation, you should: a. terminate treatment immediately. b. terminate treatment after a period of pretermination counseling, in which the client's reasons for not wanting to terminate are explored. c. continue seeing the client until he is ready to end therapy. d. seek consultation.

44. B-- Ethical standard 10.10 requires that a psychologist terminate a professional relationship when it becomes "reasonably clear that the client/patient no longer needs the service, is not likely to benefit, or is being harmed by continued service." Prior to termination, the psychologist must "provide pretermination counseling and suggest alternative service providers as appropriate."

One of the criteria for the diagnosis of autism is: a. Difficulty with phonemes b. Difficulty with morphemes c. Reversal of pronouns d. Reversal of sentences

45. C-- People with autism rarely speak (about half don't speak at all) and when they do, speech usually involves abnormalities such as echolalia and reversal of pronouns, for example, saying "you" instead of "I".

What is the approximate probability that a person with a Schizophrenic brother (not an identical or fraternal twin) would also have Schizophrenia? a. 60% b. 45% c. 10% d. 0.5%

46. C-- The concordance rate for Schizophrenia among siblings is about 10%. If you didn't know this, you might have been able to guess the correct answer. A rate of 60% or 45% probably should have seemed too high. And 0.5% is at the low end of estimates of the approximate lifetime prevalence rate of Schizophrenia and therefore should have seemed too low to be the concordance rate among related individuals.

A child is reinforced for cleaning up her room and for doing homework. Reinforcement for the homework is stopped. One could predict that cleaning up will: a. increase and doing homework will decrease. b. decrease and doing homework will also decrease. c. increase and doing homework will increase. d. decrease and doing homework will increase.

6. A-- This question has to do with the behavioral contrast effect. If we are reinforced for performing two different operants, and reinforcement for one of these behaviors stops, we tend to increase the rate of the remaining reinforced behavior. That is probably because the reinforcement that remains seems to become more valuable.

63. A primary reinforcer is the same as: a. A conditioned reinforcer b. The first reinforcer used c. Pseudoconditioning d. An unconditioned reinforcer

63. D-- A primary reinforcer is the same as an unconditioned reinforcer. These are items that acquire their reinforcing value without special training. Food and water are examples of primary reinforcers.

68. Which of the following models of leadership provides a "decision tree" to help a leader determine whether an autocratic, consultative, or consensual decision-making approach is best given the nature of the work situation? a. Fiedler's contingency model b. Hersey and Blanchard's situational model c. Vroom and Yetton's normative model d. House's path-goal model

68. C-- You may have been able to answer this one through the process of elimination if you knew that the models described by Fiedler, Hersey and Blanchard, and House don't include a decision-tree, which is provided by the Vroom and Yetton model.

Research investigating the relationship between sexual abuse in childhood and the development of Bulimia Nervosa in adulthood has found: a. the same rate as in the general population. b. a higher rate for children who were sexually abused. c. no significant relationship. d. a significant relationship for men but not for women.

B-- Although some research suggests that there is no relationship between sexual abuse and Bulimia and other Eating Disorders, we have reviewed more recent research that suggests that children who are sexually abused are more at risk.

The purpose of rotation in factor analysis is to facilitate interpretation of the factors. Rotation: a. alters the factor loadings for each variable but not the eigenvalue for each factor b. alters the eigenvalue for each factor but not the factor loadings for the variables c. alters the factor loadings for each variable and the eigenvalue for each factor d. does not alter the eigenvalue for each factor nor the factor loadings for the variables

C-- In factor analysis, rotating the factors changes the factor loadings for the variables and eigenvalue for each factor although the total of the eigenvalues remains the same.

Brad and Dave decide to go to a play, which Brad is required to see for his English literature class. The play turns out to be poorly written and performed. According to the predictions of cognitive dissonance theory, which of the following statements is most likely to be true? • 1. Dave will report liking the play more than Brad. • 2. Brad will report liking the play more than Dave. • 3. Both Brad and Dave will report liking the play to the same extent. • 4. The extent to which Brad will report liking the play will depend on how much of his grade is based upon seeing the play.

Dave will report liking the play more than Brad. (correct answer)

According to Troiden's model of gay and lesbian identity development, during which stage is an individual most likely to have negative attitudes about being homosexual while nevertheless acknowledging membership in this group? • 1. Sensitization. • 2. Identity Confusion. • 3. Identity Assumption. • 4. Commitment.

Identity Assumption. (correct answer)

False Belief Task

In this experiment you are trying to determine if Betsy has developed a sense of the independence of others' minds to the point that she can imagine that people can have other (false) beliefs about the world than what she has. In order to correctly guess that Ann will look for the ball in the blue box (where she last left the ball), Betsy would have to have an understanding of how beliefs are formed and that people's beliefs can differ from reality (the reality that the ball is now in the red box). Research has found that the ability to correctly answer the false-belief experiment normally develops around the age of four. Therefore, two-year-old Betsy would answer that Ann would look in the red box (Response 2) because Betsy does not yet grasp the fact that just because she knows where the ball is, Ann does not.

Meichenbaum: Cognitive Behavior Modification (CBM)

SELF-INSTRUCTION THERAPY * 1. therapist modeling, 2. therapist verbalization, 3. patient verbalization, 4. patient silently talks thru the task, 5. independent task performance STRESS INOCULATION TRAINING * 1. education and cognitive preparation, 2. coping skills acquisition, 3. application of skills in imagination and in vivo

An advocate of a Japanese management philosophy is most likely to advocate which of the following. a. encouraging employees to try out more than one job within the company. b. encouraging each employee to come up with ways for increasing opportunities for individual responsibility and autonomy. c. closely supervising employees . d. rapidly promoting employees who demonstrate superiority.

a

An industrial psychologist suggests that a factory manager play background music for lineworkers. The manager decides to try this out and begins to play the radio in the factory. The music is most likely to have which of the following effects. a. it will improve performance on routine tasks. b. it will have positive long-term effects on satisfaction and motivation. c. it will decrease worker irritability and frustration. d. it will increase on-the-job accidents.

a

Approximately 40% of women report experiencing some degree of distress just prior to menstruation. The symptoms of this "premenstrual syndrome" include temporary weight gain, irritability, restlessness, and headache, and appear to be due to which of the following. a. decreased hormonal levels. b. increased blood flow. c. elevated blood sugar levels. d. excessive fluid loss.

a

Freud argued that the "work of the mental apparatus is directed toward keeping the quantity of excitation low." If the mental apparatus is unsuccessful in doing so, the result is likely to be. a. anxiety. b. ego decompensation. c. psychosis. d. object-cathexis.

a

In their theories of development, both Piaget and Kohlberg emphasize which of the following. a. cognitive maturation. b. individual differences. c. imprinting. d. unconscious conflicts.

a

Margaret Mahler, who is associated with object relations tlieory, views separation-individuation as a reflection of. a. development of a permanent sense of self and a permanent mental representation of the other. b. the formation of strong bonds with additional "significant others". c. splitting of the ego and internalization of the "good mother" as the ideal object. d. development of the conflict-free ego sphere.

a

Most interpretations of performance on the Bender-Gestalt are directed toward. a. screening for brain damage. b. assessing personality. c. estimating intelligence. d. identifying learning disabilities.

a

Parasuicide (i.e., attempted, but not completed, suicide) is least associated with. a. middle to upper SES. b. female gender. c. younger age (under 30). d. divorce.

a

The owner of several fast-food restaurants would be best advised to use which of the following intervention strategies if his goal is to help his employees stop smoking cigarettes. a. supplement a behaviorally-oriented program with inter-restaurant competition. b. give employees individual bonuses for reduced cigarette smoking. c. provide employees with information about the negative side-effects of cigarette smoking that is designed to arouse a moderately high level of fear. d. institute a self-control program that is monitored by a physician and a behavioral psychologist.

a

Which of the following individuals is least likely to view maladaptive behavior as the inhibition of the natural capacity for growth. a. Rogers. b. Wolpe. c. Goldstein. d. Maslow.

b

During the first few sessions of family therapy, a structural family therapist makes use of "tracking" and "mimesis" in order to. a. keep from becoming emotionally triangulated into the family system. b. join with the family system. c. make initial changes in the family structure. d. reduce stress in the family system.

b

Individuals with which of the following disorders cross-dress for the purposes of sexual arousal. a. transsexualism. b. transvestism. c. gender identity disorder. d. all of the above.

b

Mental health professionals are sometimes called upon to assist the court in assessing a person's potential for violent behavior. In general, the predictions made by psychologists and psychiatrists yield an abundance of. a. false negatives. b. false positives. c. true negatives. d. true positives.

b

Parkinson's Disease is associated primarily with a loss of cells in the. a. reticular formation. b. basal ganglia. c. amygdala. d. temporal and occipital lobes.

b

Which of the following would help confirm a diagnosis of Alzheimer's Dementia while ruling out a diagnosis of Dementia due to Head Trauma? a. anterograde and retrograde amnesia appear at about the same time. b. anterograde amnesia appears first; after a period of time, retrograde amnesia appears. c. retrograde amnesia appears first and gradually worsens; after a period of time, anterograde amnesia appears. d. anterograde and retrograde amnesia appear at about the same time but the retrograde amnesia begins with impairments in recent memory and then progresses to impairments in remote memory.

b

Which of the following would probably be least useful when a therapist is working with an Asian-American client. a. identifying therapy goals during the initial session. b. establishing an egalitarian relationship with the client early in therapy. c. making use of "relationship questions" that elicit information about the client's interactions with others. d. asking the client to try to identify exceptions to the presenting problem.

b

You are hired by a community mental health center as an outside consultant to help one of the clinicians develop a treatment plan for a difficult client. This type of consultation is referred to as. a. consultee-centered case consultation. b. client-centered case consultation. c. client-administrative case consultation. d. consultee-administrative case consultation.

b

You conduct a study to compare the effects of treatment type and treatment duration on cigarette smoking. Following administration of the treatments to subjects for the appropriate length of time, you determine the average number of cigarettes smoked each day by subjects in each group and obtain the following data. After two weeks, treatment one subject smoked 10 cigarettes per day, treatment subject two smoked ten per day and treatment three subject smoked 15. After four weeks, subject one smoked 6 per day, subject two smoked 12 and subject under treatment three smoked 9. After eight weeks subject one smoked 3 per day, subject two smoked 6 and subject three smoked three per day. Based on the above information, you can conclude that there are. a. main effects of treatment type and duration only. b. main effects of treatment type and duration and interaction effects. c. main effects of treatment type only. d. interaction effects only.

b

Zelda obtains a score of 41 on a test that has a mean of 50 and a standard deviation of 6. If the raw scores are changed so that the test now has a mean of 100 and a standard deviation of 12, Zelda's raw score would be equal to. a. 91. b. 82. c. 41. d. none of the above.

b

According to the interference hypothesis, normal aging is associated with a diminishing ability to screen out irrelevant distractions. Assuming that this hypothesis is true, this has the greatest implications for which aspect of attention? a. divided. b. sustained. c. selective. d. episodic.

c

"Privilege" refers to a client's legal right to have confidential communications protected from disclosure. The situations that legally allow a waiver of privilege vary from state to state. However, generally speaking, which of the following is true about privilege. 1. Privilege is waived when the information is requested at a proceeding initiated by the client in a criminal action to determine his or her sanity. 2. Privilege is waived when the therapist suspects that a minor (e.g., the client or child of the client) has been the victim of child abuse. 3. Privilege is waived when the therapist has reasonable suspicion to believe that the client is a danger to another person. a. 3 only. b. 2 and 3 only. c 1,2 and 3. d. none of the above.

c

A 19-year old male who grew up in the inner-city receives a WAIS-R Verbal IQ score that is 20 points higher than his Performance IQ score. This discrepancy suggests. a. a learning disability. b. an educational deficit. c. right hemisphere damage. d. an invalid test.

c

After several sessions with her therapist, a client starts acting toward the therapist as though he were the client's father. Assuming that the therapist is a practitioner of Gestalt therapy, he is most likely to respond to this transference by. a. ignoring it. b. temporarily assuming the role of the client's father. c. helping the client see the difference between her transference and reality. d. helping the client understand how her past relationship with her father is affecting her current relationships with men.

c

If you are most concerned about the reliability of an achievement test you will be administering to high school freshmen, which one of the following kinds of tests would you be least likely to use. a. fill-in-the-blank. b. four-alternative multiple-choice. c. true-false. d. essay.

c

In contrast to traditional psychodynamic psychotherapy, the brief psychotherapies. a. focus on current phenomenon only. b. discourage the development of transference. c. focus on specific conflicts. d. all of the above.

c

In the assessment of cognitive abilities with a standardized test, you would "test the limits". a. before administering the test. b. before administering each subtest. c. after administering the test. d. whenever it seems appropriate to do so.

c

In vivo flooding has been found to be an effective treatment for Agoraphobia. Studies investigating the effects of in vivo flooding for this disorder suggest that. a. frequent, brief exposures are more effective than less frequent, longer exposures to the feared stimuli. b. people with high arousability are more responsive to in vivo treatments than people with low arousability. c. high anxiety provocation is not the key factor in the effectiveness of in vivo flooding. d. courtterconditioning is a necessary component of in vivo flooding as a treatment for agoraphobia.

c

Increasing age is least likely to have which of the following effects on a man's sexual response cycle. a. the time to achieve an erection will increase. b. complete penile erection may not occur until just prior to orgasm. c. the resolution phase and refractory period will both increase. d. all of the above.

c

Kohlberg's theory of moral development views it as being. a. the result of parental expectations. b. biologically innate and universal. c. promoted by peer interactions. d. the result of school and church teachings.

c

Which of the following statements is least true of individuals who have been diagnosed with Autistic Disorder. a. only a small percentage of them go on as adults to live and work independently. b. signs of the disorder may be present in early infancy. c. a subset of individuals with the disorder, "those who do have the cognitive ability to eventually function independently", "grow out" of the disorder in adulthood. d. they are sometimes excellent in tasks involving long-term memory.

c

Withdrawal from which of the following substances is least likely to involve hallucinations. a. a sedative. b. an anxiolytic. c. cocaine. d. alcohol.

c

You are conducting a study to investigate the phenomenon known as "blocking." Consequently, you will. a. first present the CS prior to the US; then, once the CR is established, present the US prior to the CS. b. first present the CS prior to the US; then, once the CR is established, present the CS prior to a sec and US. c. first present the CS prior to the US; then, once the CR is established, present the original CS simultaneously with a second CS prior to the US. d. first present the CS prior to the US; then, once the CR is established, present a second CS prior to the original CS.

c

You are using in vivo aversive conditioning to reduce alcohol consumption in chronic alcoholics. To maximize the effectiveness of the treatment, the aversive stimulus (electric shock) should be applied. a. while the client is thinking about taking a drink. b. right before the client takes a drink. c. immediately after the client begins drinking. d. after the client has finished a drink

c

You are using neuroleptics to treat a schizophrenic patient. If you were to rely on research into their use in the treatment of schizophrenia, you would discover which of the following. a. thorazine tends to be the least effective drug for treating schizophrenia. b. the thioxanthenes tend to be the most effective drugs for treating schizophrenia. c. the selection of an appropriate neuroleptic depends largely on the patient's responsivity. d. no neuroleptic has been found to be particularly effective for the positive symptoms of this disorder.

c

Dr. Dither is hired by the court to conduct an evaluation of a family as part of a child protection matter. Based on Dr. Dither's evaluation and other evidence, the court determines that the child can remain at home with his family who will be provided with supervision and counseling. Six months after the case is heard in court. Dr. Dither is contacted by the mother who says she would like to begin therapy with him for issues unrelated to the child protection matter. According to APA's Guidelines for Psychological Evaluations in Child Protection Matters, Dr. Dither should. a. agree to see the woman in therapy since the child protection matter has been resolved. b. agree to see the woman in therapy only if her problem is, in fact, unrelated to the child protection matter. c. agree to see the women in therapy since his familiarity with her situation will be an advantage. d. refer the woman to a colleague.

d

Dr. Lane, a psychologist, is conducting a research study on peer pressure. Her subjects will be 120 fifth grade students. The study will involve a 30-minute interview with each student and observation of the students during their lunch hour. Before beginning the study, the psychologist should. a. obtain the consent of the students. b. obtain the consent of the students' parents. c. obtain the consent of appropriate school personnel. d. obtain the assent of the students and the consent of their parents.

d

The mother of a 35-year-old man says that, for the past five weeks, he has been acting very uncharacteristically. He seems perplexed and nervous much of the time, his tone of voice varies between flat and shrill, and his speech is sometimes like "word salad." The mother says that sometimes it seems like he is listening to voices or sounds that she cannot hear. Based on this information, the best diagnosis is. a. schizophrenia, disorganized type. b. schizoid personality disorder. c. schizoaffective disorder. d. schizophreniform disorder.

d

You administer the MMPI-2 to several clients. One client, a 27-year old male, obtains a T score of 100 on the F scales. This means that. a. the client has attempted to "fake good". b. the client is insecure, intolerant of unconventional behavior, and has a lack of insight. c. you should use the F scale score as a correction factor for other scales. d. assuming no scoring errors, you should consider the test results invalid.

d

You are working as an industrial psychologist and are hired as a consultant by the owner of a medium-sized company who says she is having problems with employee productivity. As an advocate of "goal-setting theory," you are most likely to tell the owner that, to maximize the productivity of her workers, she should. a. let the workers set their own goals. b. let the workers set their own goals when the task is simple and structured, but assign goals when the task is complex and unstructured. c. assign the workers moderate goals that are within the capabilities of the least proficient workers. d. assign the workers relatively difficult goals and give them feedback about their attainment of those goals.

d

The overjustification hypothesis is most closely related to: • 1. internal and external locus of control. • 2. intrinsic and extrinsic motivation. • 3. ratio of inputs and outputs. • 4. Gain-Loss Theory.

intrinsic and extrinsic motivation. (correct answer)

According to Peter Lewinsohn, depression results from: • 1. low rates of behavior. • 2. too much punishment. • 3. cognitive distortions. • 4. deficits in serotonin.

low rates of behavior. (correct answer, your response) Lewinsohn is a behaviorist who argues that depression is essentially caused by a low rate of behavior. All other symptoms of depression are thought to result from this initial low rate of behavior which in itself is the result of a lack of reinforcement from the environment. Lewinsohn's therapy is based on the principle of teaching depressed patients how to elicit higher rates of reinforcement from the environment, which thereby increases rates of behavior, and reduces depression.

Which of the following best describes the cause of family dysfunction according to structural family therapy? • 1. Deficits in the ability of family members to communicate their needs and an inappropriate system of rewards and exchanges. • 2. Malfunctions in the hierarchy of the family with the symptom serving as a message. • 3. Formation of subsystems where closeness of two members excludes appropriate involvement of a third member. • 4. Malfunctioning arrangements in the family hierarchy with problematic patterns of intimacy.

• 4. Malfunctioning arrangements in the family hierarchy with problematic patterns of intimacy. (correct answer)

What is the most common anticholinergic side effect? • 1. Dry mouth. • 2. Blurred vision. • 3. Akathisia. • 4. Urinary retention.

1. Dry mouth. (correct answer, your response) The most common anticholinergic side effect is dry mouth (Response 1). Blurred vision (Response 2) and urinary retention (Response 4) are also anticholinergic side effects, but they are less common than dry mouth. Akathisia (Response 3) is a common side effect of traditional antipsychotic medications and involves a feeling of intense restlessness accompanied by anxiety and dysphoria.

Which of the following statements most accurately describes the Cattell-Horn-Carroll theory of intelligence? • 1. It is a three strata model, with general intelligence at the top level, broad cognitive abilities in the middle, and narrow cognitive abilities at the bottom. • 2. It is a triarchic theory that focuses on the process of intelligence, not the product. • 3. It proposes that intelligence is a single factor. • 4. It posits four basic underlying factors to the construct of intelligence.

1. It is a three strata model, with general intelligence at the top level, broad cognitive abilities in the middle, and narrow cognitive abilities at the bottom. (correct answer, your response) The Cattell-Horn-Carroll theory is considered by many to be the most empirically validated theory of human intelligence. It is the foundation for the Woodcock-Johnson III (WJ-III) battery, a popular test of general aptitude and school achievement, and has been used to guide the development of the Wechsler Intelligence Scale for Children-IV (WISC-IV), the most established test of children's intelligence. According to this theory, intelligence can be understood in three levels: general intelligence or "g" (stratum III); about ten broad cognitive abilities, such as fluid reasoning, short-term memory, visual processing, reading and writing (stratum II); and finally, about 70 narrow cognitive abilities (stratum I). This theory, based on the original work of Cattell, has gone through much development and there is some variance in the way it can be presented. For example, Horn and Carroll disagree over the validity of the general intelligence factor. Sternberg's Triarchic Theory of intelligence focuses on process rather than product (Response 2). Spearman proposed that intelligence is a single factor "g" (Response 3). The WISC-IV has eliminated Verbal IQ (VIQ) and Performance IQ (PIQ), and instead Full Scale IQ (FSIQ) is comprised of four indices or factors: Verbal Comprehension Index (VCI), Perceptual Reasoning Index (PCI), Working Memory Index (WMI) and Processing Speed Index (PSI) (Response 4).

A psychologist devises a test to detect eating disorders among adolescents. She is concerned that the test is not able to detect less severe cases of the disorder. What can the psychologist do to increase the sensitivity of this test? • 1. Lower the predictor cutoff. • 2. Raise the predictor cutoff. • 3. Raise the criterion cutoff. • 4. Lower the predictor cutoff and raise the criterion cutoff.

1. Lower the predictor cutoff. (correct answer)

Which substance mimics the effects of morphine in the brain? • 1. Opioid peptides. • 2. Opioid antagonists. • 3. Cathecholamines. • 4. Adrenaline.

1. Opioid peptides. (correct answer, your response) Opioid peptides are amino acids that mimic the effects of opiates (e.g., morphine) in the brain, resulting in feelings of euphoria and decreased sensations of pain. Opioid antagonists (Response 2) act on opiate receptors to block the effects of opiates on the brain. Opioid antagonists (e.g., naloxone, naltrexone) are used at times to treat opiate addiction. The catecholamines (Response 3) include dopamine, norepinephine/noradrenaline, and epinephrine/adrenaline (Response 4). The catecholamines cause general physiological changes that prepare the body for physical activity (e.g., fight or flight). Dysregulation of dopamine and norepinephrine levels have also been implicated in a number of disorders (e.g., Depression, Parkinsons, Schizophrenia).

According to nigrescence models, the deracinated person is most likely to be in which of the following stages? • 1. Preencounter. • 2. Encounter. • 3. Immersion-emersion. • 4. Internalization.

1. Preencounter. (correct answer, your response) A person is thought to be deracinated (without race) when he views being Black as an obstacle, and strongly prefers the dominant culture's values. These characteristics are most closely associated with the first stage of Cross's model, preencounter (called conformity in other models). Encounter (also known as dissonance) is the second stage and involves conflict between old self-deprecating attitudes and newly emerging appreciating attitudes (Response 2). In immersion-emersion, (also called resistance and immersion), there is generally a strong identification with the minority group, and rejection of the values of the dominant culture (Response 3). In the final stage of internalization (also called integrative awareness), there is an acceptance and valuing of the self as well as the dominant culture (Response 4).

Which of the following medications is least likely to cause problems in concentration, attention, and memory? • 1. Prozac (fluoxetine). • 2. Elavil (amitryptyline). • 3. Chlorpromazine (thorazine). • 4. Xanax (alprazolam).

1. Prozac (fluoxetine). (correct answer)

Which of the following medications is least likely to cause problems in concentration, attention, and memory? • 1. Prozac (fluoxetine). • 2. Elavil (amitryptyline). • 3. Chlorpromazine (thorazine). • 4. Xanax (alprazolam).

1. Prozac (fluoxetine). (correct answer, your response) Of all the medications listed here, Prozac (fluoxetine), a Selective-Serotonin Reuptake Inhibitor (SSRI) antidepressant, is least likely to affect concentration, attention, and memory. Elavil (amitryptyline), a tricyclic antidepressant (Response 2), and Chlorpromazine (thorazine), an antipsychotic (Response 3), both have high levels of anticholinergic side effects, including dry mouth, blurry vision, confusion, and decreased memory. Xanax (alprazolam) is a benzodiazepine (Response 4). It commonly causes drowsiness, and occasionally causes mild cognitive deficits and some amnesia.

Which part of the brain is responsible for interpreting the expression of positive and negative emotions? • 1. The frontal lobes. • 2. The temporal lobes. • 3. The right hemisphere. • 4. The left hemisphere.

1. The frontal lobes. (correct answer)

Irving Yalom determined several factors that are potentially curative in group psychotherapy. These factors include: • 1. family reenactment, catharsis, group cohesiveness. • 2. family reenactment, interpersonal learning, leader style. • 3. group cohesiveness, interpersonal learning, confrontation. • 4. catharsis, universality, interpretation.

1. family reenactment, catharsis, group cohesiveness. (correct answer)

A teaching assistant is asked to observe a hyperactive student's on-task behavior. She would most likely be using: • 1. interval recording. • 2. content sampling. • 3. event recording. • 4. purposive sampling.

1. interval recording. (correct answer, your response) Interval recording is used when a behavior is not discrete and has no clear beginning or end, because in such instances it is not possible to measure the number of times a child is on-task. Instead, the time period of observation is divided into smaller intervals, for example an hour is divided into twelve five-minute intervals. There are two methods of interval sampling: either the observer notes whether the behavior was present at the moment the interval ends (momentary time sampling), or the observer notes whether the behavior was present for the entire duration of the interval (whole-interval sampling). Event recording (Response 3), also called frequency recording, is used for discrete, easily measured behaviors such as a child making her bed. In event recording, the observer simply records the number of times the target behavior occurs. Content sampling (Response 2) is a source of error when assessing the reliability of a test, in particular, alternate forms reliability and split-half reliability. Purposive sampling (Response 4) is a sampling procedure for selecting subjects. It involves selecting a particular sample because it is believed to be representative of the population (e.g., test marketing a new product in Los Angeles because it is believed that how Angelinos view the product will be similar to how people in the rest of the country will view the product).

In extreme cases, abrupt cessation of benzodiazepines (e.g., Xanax, Valium) can be life threatening. More commonly, signs of acute withdrawal include: • 1. rebound anxiety. • 2. slurred speech and incoordination. • 3. fever and diarrhea. • 4. dysphoric mood.

1. rebound anxiety. (correct answer, your response) Abrupt cessation of a benzodiazepine can lead to some of the characteristic symptoms of sedative, hypnotic and anxiolytic withdrawal. One of these symptoms is anxiety. Additional symptoms include autonomic hyperactivity, hand tremor, insomnia, nausea or vomiting, psychomotor agitation, and grand mal seizures. Slurred speech and incoordination (Response 2) is included among the diagnostic criteria for sedative, hypnotic and anxiolytic intoxication. Fever and diarrhea can occur with opioid withdrawal (Response 3). Dysphoric mood is a hallmark of cocaine and/or amphetamine withdrawal, although it can occur with other syndromes, such as opioid withdrawal (Response 4).

A patient with a lesion in the left inferior frontal area is most likely to exhibit impairments in: • 1. speech fluency. • 2. speech comprehension. • 3. proprioception. • 4. memory.

1. speech fluency Broca's area is located in the left frontal lobe, which controls the muscles that produce speech. Broca's aphasia, which results from damage to this area, includes difficulties with speech production and fluency. Difficulties in speech comprehension (Response 2) would result from damage to Wernicke's area, located in the temporal lobes. Impairment in proprioception (Response 3) would result from damage to the parietal lobes. Memory problems (Response 4) would result from damage to several areas, including the frontal lobes, as well as the hippocampus, and the temporal lobes.

A researcher would use LISREL, a form of structural equation modeling, in an attempt to: • 1. test a causal model of relationships among variables. • 2. derive a causal model of relationships among variables. • 3. determine whether underlying variables are latent or manifest. • 4. determine the non-linear relationship between multiple IVs and multiple DVs.

1. test a causal model of relationships among variables. (correct answer) LISREL, which stands for LInear Structural RELations, is a computer program that can determine whether a given causal model of relationships among variables is correct. It does not generate a causal model (Response 2). Although LISREL does make distinctions between latent (unobserved) and manifest (observed) variables, it is not used to determine whether a specific variable is latent or manifest. The experimenter must make this determination as part of the causal model. As the name implies, LISREL only analyzes linear relationships (Response 4). It is able, however, to look at direct and indirect effects, as well as both unidirectional and bi-directional paths.

Children with autism show relative strength on: • 1. the Embedded Figures Test. • 2. the TONI-3 test. • 3. Raven's Progressive Matrices. • 4. the Stanford-Binet.

1. the Embedded Figures Test. (correct answer) A significant body of research has found that children with autism perform better in relation to their general mental age on the Embedded Figures Test (EFT). The EFT measures cognitive functioning by having the examinee try to locate simple geometrical shapes that are hidden in more complex diagrams. The TONI-3 (Response 2) or Test of Nonverbal Intelligence measures intelligence, aptitude, abstract reasoning, and problem solving completely free of the use of language. It is particularly well suited for individuals who are known or believed to have disorders of communication or thinking that may result from mental retardation, deafness, developmental disabilities, autism, cerebral palsy, stroke, disease, head injury, or other neurological impairment. Raven's Progressive Matrices (Response 3) is generally thought to be one of the best non-verbal tests of intelligence. It is also suited for individuals with severe motor impairments and speech limitations. Spearman himself considered Raven's Progressive Matrices to be the best non-verbal measure of the "g factor." The Stanford-Binet (Response 4) is a widely used test of intelligence (requiring verbal skills) with a low floor (e.g., IQ scores can be obtained in the teens) and high ceiling (e.g., IQ score of 180+ can be obtained), making it the test of choice for assessing both giftedness and mental retardation.

What is the optimal treatment for Nicotine Dependence? • 1. A single strategy (i.e., counseling or pharmacotherapy) because it enhances compliance. • 2. A combination of counseling and pharmacotherapy. • 3. Pharmacotherapy alone for first-time quitters, and pharmacotherapy and behavioral intervention for those that have previously attempted to quit. • 4. An individually-tailored program based on the smoker's history.

2. A combination of counseling and pharmacotherapy. (correct answer)

Which of the following findings supports the Dopamine Hypothesis? • 1. Akathisia is the most common side effect of the traditional antipsychotics. • 2. Cocaine Intoxication may result in auditory, visual, or tactile hallucinations. • 3. The novel antipsychotic drugs do not exert their primary effect on dopamine. • 4. Patients with Parkinson's Disease show a deficiency in dopamine.

2. Cocaine Intoxication may result in auditory, visual, or tactile hallucinations.

A researcher is studying the effects of restrictive dieting on binge eating. Subjects are divided into three groups: a control group, a sensible diet group, and a highly restrictive diet group. Binge eating is measured on a numerical scale of 0 to 75. The researcher finds, contrary to expectations, that restrictive dieters do not differ from sensible dieters in terms of severity of binge eating. However, both types of dieters do differ from the control group in terms of binge eating at a statistically significant level. What type of statistic must have been used to arrive at these results? • 1. Analysis of covariance (ANCOVA). • 2. One-way ANOVA. • 3. Factorial ANOVA. • 4. Multiple analysis of variance (MANOVA).

2. One-way ANOVA. (correct answer)

In the above question, the child's response is characteristic of which stage in Piaget's theory? • 1. Concrete operational. • 2. Preoperational. • 3. Sensorimotor. • 4. Formal operational.

2. Preoperational. (correct answer, your response) Animism is characteristic of the preoperational stage of development (ages 2 to 7), in which the child thinks and reasons intuitively and employs magical thinking. In the concrete operational stage (Response 1), the child uses logical thought processes and operates on the real and perceivable world of objects (ages 7 to 11). The sensorimotor stage (Response 3) involves sensory observation, as well as the exploration and manipulation of the environment (birth to 2). The formal operational stage (Response 4) is characterized by the ability to think abstractly and reason deductively (11 years and beyond).

In which of the following situations would a researcher most likely decide to use an ANCOVA (Analysis of Covariance)? • 1. When in designing a study an extraneous variable is anticipated. • 2. When unexpected differences are uncovered among treatment groups with regard to an extraneous variable. • 3. To remove the effects of an extraneous variable in the correlation of two continuous variables. • 4. To transform a moderator variable into another IV.

2. When unexpected differences are uncovered among treatment groups with regard to an extraneous variable. (correct answer, your response) The ANCOVA is similar to the analysis of variance (ANOVA), with an additional element of co-varying out an extraneous variable, a variable that correlates with the outcome measure. An ANCOVA is typically most appropriate only when the extraneous variable is unanticipated. If the variable is anticipated (Response 1), it should be accounted for directly in the research design, usually through a randomized block ANOVA. A randomized block ANOVA transforms an extraneous variable, one that moderates the relationship between the IV and DV, into another IV to be studied (Response 4). In this way, interaction effects can also be studied and more information is yielded than in an ANCOVA. You might have been able to eliminate Response 3 by recognizing that the correlation of two continuous variables deals with correlation coefficients, not variations of the ANOVA, which are tests of difference. Removing the effects of an extraneous variable in a correlation (Response 3) is the definition of partial correlation.

A traditional neuroleptic would best treat: • 1. avolition. • 2. bizarre delusions. • 3. thought disorder. • 4. poverty of speech.

2. bizarre delusions. (correct answer) Traditional neuroleptics, such as Haldol and Thorazine, are good at treating the positive symptoms of Schizophrenia, such as delusions, hallucinations, and disorganized thinking. They are not very good at treating the negative symptoms, such as flat affect, avolition (Response 1), also known as lack of motivation, and poverty of speech (Response 4), also known as alogia. The atypical antipsychotics, such as Risperdal, were originally viewed as a breakthrough in the treatment of negative symptoms; however, recent large-scale research studies indicate this is not the case. Thought disorder (Response 3) is a term sometimes applied to psychotic disorders, and is too broad to be a good answer for this question. One way to approach this question is to recognize that Responses 1 and 4 are both types of negative symptoms; it's unlikely that a drug would be good at treating one but not the other. You could therefore eliminate both responses.

Systemic family therapy involves: • 1. the use of genograms to illustrate the nature of relationships among family members. • 2. circular questions to clarify family members' attributions of the presenting problem. • 3. an exploration of transferences and projections that occur between the family members. • 4. an examination of boundaries and subsystems.

2. circular questions to clarify family members' attributions of the presenting problem. (correct answer)

BBThe presence of daytime sleepiness, episodes of loss of muscle tone associated with intense emotion, and hypnogogic hallucinations suggest a diagnosis of: • 1. sleep apnea. • 2. narcolepsy. • 3. primary insomnia. • 4. sleep terror disorder.

2. narcolepsy. (correct answer, your response) The hallmarks of narcolepsy include irresistible attacks of refreshing sleep, along with cataplexy (sudden bilateral loss of muscle tone) frequently triggered by intense emotion, and/or recurrent intrusions of REM sleep into the transition between sleep and wakefulness resulting in hypnogogic or hypnopompic hallucinations or sleep paralysis. Daytime sleepiness is typically one of the first symptoms of narcolepsy. Both sleep apnea (breathing related sleep disorder) and primary insomnia result in sleep disruption that leads to excessive sleepiness, however cataplexy and hallucinations are absent (Responses 1 & 3). Characteristics of sleep terror disorder include abrupt awakening from sleep, intense fear and autonomic arousal, relative unresponsiveness to comfort by others, and amnesia for the episode (Response 4).

The Solomon Four-Group design is used: • 1. to control for carryover effects when repeated measures are used. • 2. to control for the effects of testing. • 3. to control for the effects of history and failure to return to baseline in single-subject designs. • 4. to control for the effects of experimenter expectancies and demand characteristics.

2. to control for the effects of testing. (correct answer, your response) The Solomon Four-Group is a type of research design that controls for the effects of testing (also called practice effects). Subjects are divided into four groups: the first group is tested before and after treatment (pretest/intervention/posttest); the second group is tested twice without any treatment (pretest/no intervention/posttest); the third group is treated and measured afterwards (intervention/posttest); and the fourth is only tested once (posttest). The design accounts for all possible effects of testing. The Latin Square is the most sophisticated counterbalancing design and controls for carryover effects when repeated measures are used (Response 1). Multiple baseline designs eliminate the threat of history and failure to return to baselines (as well as ethical concerns) in single-subject designs (Response 3). Experiment expectances and demand characteristics (Response 4) are controlled by making experiments double-blind.

Adverse impact generally constitutes discrimination, but a test that adversely affects hiring or promotion may still be used: • 1. when the percentage of minorities hired or promoted is at least 4/5 of the percentage of non-minorities hired or promoted. • 2. when the test is valid, job-related, and no alternatives are available. • 3. during times of economic hardship, such as periods of high rates of inflation and unemployment. • 4. when the test has an extremely high rate of accurately predicting success on the criterion.

2. when the test is valid, job-related, and no alternatives are available. (correct answer)

The earliest age of operations is: • 1. 2 years. • 2. 4 years. • 3. 7 years. • 4. 11 years.

3. 7 years. (correct answer, your response Piaget described four stages leading up to the capacity for adult thought: sensorimotor, preoperational, concrete operational and formal operational. The concrete operational stage involves the beginning of organized and logical thought. The child acquires the ability to comprehend the principle of conservation and is able to order events in a logical sequence. However, the child can only think about actual physical (concrete) objects; they have not yet developed the capacity for abstract reasoning. The concrete operational stage typically occurs between the ages of 7 and 11 years (Response 3). The preoperational stage begins at about age 2 (Response 1), and the formal operational stage may begin at age 11 (Response 4).

Dr. Spook hypothesizes that "fear" involves three components: physical arousal, cognitive appraisal of danger, and suggestibility. He designs a self-report questionnaire that yields a quantitative measure of degree of fear. He includes questions addressing physical arousal, cognitive appraisal of danger, and suggestibility. To confirm his hypothesis, he should use which of the following statistics? • 1. Discriminant analysis. • 2. Path analysis. • 3. Factor analysis. • 4. Analysis of variance.

3. Factor analysis. (correct answer)

A man exhibits shallow emotionality, dysphoria when not the focus of attention, dramatic speech, and eccentric appearance. In addition, he has many acquaintances but few close friends. These symptoms are most suggestive of: • 1. Borderline Personality Disorder. • 2. Narcissistic Personality Disorder. • 3. Histrionic Personality Disorder. • 4. Schizotypal Personality Disorder.

3. Histrionic Personality Disorder. (correct answer) These are all symptoms of someone with Histrionic Personality Disorder. Borderline Personality Disorder (Response 1) is characterized by instability in the areas of mood, interpersonal relationships, and self-image, as well as significant impulsivity. Although this disorder can also involve attention seeking, its hallmarks are instability and impulsivity, neither of which are prominent in this scenario. Narcissistic Personality Disorder (Response 2) is characterized by a pattern of grandiosity, need for admiration, and lack of empathy. Persons with Narcissistic Personality Disorder seek admiration, whereas persons with Histrionic Personality Disorder seek attention, even attention based on their weaknesses. Schizotypal Personality Disorder (Response 4) involves deficits in interpersonal functioning marked by discomfort with and reduced capacity for close relationships, as well as by peculiarities in cognition, ideation, appearance, and behavior.

Which of the following disorders is autosomal dominant? • 1. Multiple Sclerosis. • 2. Parkinson's. • 3. Huntington's. • 4. Tourette's.

3. Huntington's. (correct answer) Huntington's Disease (Response 3) is inherited as an autosomal dominant condition. Huntington's results from a single, abnormal gene on one of the autosomal chromosomes (one of the first 22 "non-sex" chromosomes). When one parent has Huntington's Disease, there is a 50% chance that each of the offspring will inherit the disorder. There is some evidence that both Tourette's Syndrome (Response 4) and Parkinson's Disease (Response 2) have a genetic component, but further research needs to be done to determine the specific mechanisms of heredity. Multiple Sclerosis (MS) is not a hereditary disease (ruling out Response 1), however, increasing scientific evidence suggests that genetics may play a role in determining a person's susceptibility to MS.

Which of the following statements best represents George Kelly's Personal Construct Theory? • 1. People experience the world as unified meaningful wholes, not as affective or cognitive fragments. • 2. People tend to make dispositional explanations for the behavior of others, while they make situational explanations for their own behavior. • 3. People experience the world based on their anticipation of future events, which in turn is based on their past experience. • 4. People are motivated to change their thinking when they experience awareness of inconsistency among their constructs.

3. People experience the world based on their anticipation of future events, which in turn is based on their past experience. (correct answer, your response) George Kelly is known for describing ordinary people as scientists, who are constantly forming, testing, and revising hypotheses about the world around them. His Personal Construct Theory is based on the fundamental postulate that "a person's processes are psychologically channelized by the ways in which he anticipates events," meaning that we perceive the world according to what we expect to see. According to Kelly, these expectations are based on our past experiences. As our experiences change, we revise our expectations. Gestalt theory posited that people experience the world in whole, not affective or cognitive fragments (Response 1). The tendency to attribute negative outcomes to dispositional factors in others, while attributing these same negative outcomes to situational factors in ourselves, defines the actor-observer bias (Response 2). Cognitive Dissonance theory posits that people change their attitudes to match their behavior because they experience dissonance when they become aware of inconsistencies among their cognitions (Response 4).

In which of the following circumstances would it be problematic to use a chi square? • 1. When looking for differences between groups. • 2. When the data are ordinal. • 3. When repeated observations are made. • 4. When there is more than one independent variable.

3. When repeated observations are made. (correct answer)

The standard error of the estimate has: • 1. a direct relationship with the SD of the predictor and an indirect relationship with validity. • 2. an indirect relationship with the SD of the predictor and a direct relationship with validity. • 3. a direct relationship with the SD of the criterion and an indirect relationship with validity. • 4. an indirect relationship with the SD of the criterion and a direct relationship with validity.

3. a direct relationship with the SD of the criterion and an indirect relationship with validity. (correct answer)

A researcher looks at several variables (study time, graduate school GPA, anxiety level) in an attempt to predict who is likely to pass the EPPP. The researcher will analyze her results using a: • 1. multiple regression. • 2. multiple cutoff. • 3. discriminant analysis. • 4. path analysis.

3. discriminant analysis. (correct answer) A discriminant analysis is required when several independent variables are used to predict group membership. Group membership is another way of saying discrete categories. In this case, three continuous variables are being used to predict whether subjects belong in the "pass exam" or "fail exam" category. Multiple regression (Response 1) is appropriate when several independent variables are used to predict one continuous dependent variable (e.g., EPPP score). In a test with multiple cutoff (Response 2), a subject can only pass the overall test if he meets the cutoff on each subtest. Path analysis (Response 4) applies multiple regression techniques to test a theoretical model that specifies causal links among variables.

Theories of gender role development assume that certain processes occur at different times in development. Most theories propose that the sequence of acquisition is: • 1. gender identity, gender roles, gender constancy. • 2. gender identity, gender constancy, gender roles. • 3. gender roles, gender identity, gender constancy. • 4. gender roles, gender constancy, gender identity.

3. gender roles, gender identity, gender constancy. (correct answer)

Frame-of-reference (FOR) training is used to: • 1. reduce ratee-based sources of error. • 2. reduce the halo effect. • 3. improve the accuracy of performance ratings. • 4. eliminate unfair discrimination.

3. improve the accuracy of performance ratings. (correct answer, your response) Frame-of-reference (FOR) training is used to improve the accuracy of performance ratings. FOR training provides raters with common performance standards (references) to help raters become clear on what constitutes "good" and "bad" behavior. This type of training also tends to improve agreement among raters. Such training may in fact reduce ratee-based sources of error (Response 1), such as personal biases and the halo effect (Response 2); it may also decrease the effect of biases that lead to unfair discrimination (Response 4), but the goal of FOR training is broader than the elimination of any one specific source of error.

In a research study, low power is most directly associated with: • 1. low alpha. • 2. low beta. • 3. less likelihood of significance. • 4. less reliability.

3. less likelihood of significance. (correct answer)

In weighting variables in a multiple regression equation, the researcher uses: • 1. factor analysis. • 2. covariance. • 3. line of best fit. • 4. multiple cutoff.

3. line of best fit. (correct answer)

In weighting variables in a multiple regression equation, the researcher uses: • 1. factor analysis. • 2. covariance. • 3. line of best fit. • 4. multiple cutoff.

3. line of best fit. (correct answer, your response) Regression or prediction equations are calculated based on the line of best fit. A scatter plot is graphed and then a straight line is made which best matches or fits the data, using a process called the least squares criterion. The equation for this line is then the regression equation. Factor analysis (Response 1) is not involved in multiple regression; it is the statistical analysis of the structure of a construct or test. Covariance (Response 2) means that two variables are correlated: they "covary" or vary together. Multiple cutoff (Response 4) describes a testing situation in which a subject must meet the cutoff on each subtest in order to pass the overall test.

Solution-focused family therapy emphasizes: • 1. ratio of costs and benefits. • 2. re-storying and unique outcomes. • 3. the miracle question, the exception question, and the scaling question. • 4. positive connotation and circular questioning.

3. the miracle question, the exception question, and the scaling question. (correct answer, your response) Solution-focused family therapy is a "post-modern" family therapy developed by Steve de Shazer and his colleagues. A strength-based approach, it encourages patients to recognize solutions that have worked in the past or are likely to work. It is noted for its use of the miracle question ("Suppose one night, while you were asleep, there was a miracle and this problem was solved. How would you know? What would be different?"), the exception question (i.e., focusing attention on times the patient did not have the problem), and the scaling question ("On a scale of zero to ten, with zero being how depressed you felt when you called me, and ten being how you will feel the day after the miracle, how do you feel right now?"). Richard Stuart, one of the pioneers of marital behavior therapy, developed an approach that combines operant learning principles with social exchange theory. According to the social exchange model, behavior in relationships is maintained by its ratio of costs and benefits (Response 1). Narrative therapy, developed by Michael White, makes use of re-storying and unique outcomes (Response 2). Positive connotation and circular questioning (Response 4) are most characteristic of the Milan group.

Indifference and euphoria suggest damage to: • 1. the frontal lobes. • 2. the temporal lobes. • 3. the right hemisphere. • 4. the left hemisphere.

3. the right hemisphere. (correct answer)

Indifference and euphoria suggest damage to: • 1. the frontal lobes. • 2. the temporal lobes. • 3. the right hemisphere. • 4. the left hemisphere.

3. the right hemisphere. (correct answer, your response) Indifference and euphoria suggest right hemisphere damage. Other affective abnormalities that may result from right hemisphere damage include depression, hysteria, florid manic excitement, and impulsivity. The right hemisphere is also involved with perceptual, visuospatial, artistic, musical, and intuitive activities. The left hemisphere (Response 4) is responsible for language in most people. Damage to the left hemisphere typically results in difficulty with speech (comprehension or expression). The left hemisphere is also responsible for rational, logical, analytical and abstract thinking. The frontal lobes (Response 1) play a critical role in inhibition, planning and initiative, judgment, and higher mental functions (e.g., cognitive flexibility). The temporal lobes (Response 2) contain the primary auditory cortex. Along with several other attached brain structures (e.g., amygdala and hippocampus), the temporal lobes are involved in emotional behavior and memory.

Partial complex seizures typically originate in: • 1. the frontal lobes. • 2. the brain stem. • 3. the temporal lobes. • 4. the limbic system.

3. the temporal lobes. (correct answer)

After conducting a two-way ANOVA, you determine significant differences have been found. You are most likely to conclude: • 1. there are main effects and there may or may not be interaction effects. • 2. there are interaction effects and there may or may not be main effects. • 3. there can be any combination of main effects and interactions. • 4. there are probably neither main effects nor interaction effects because the results may be due to chance.

3. there can be any combination of main effects and interactions. (correct answer)

The latent trait model or item response theory is an alternative to classical test theory. Item response theory is based on the premise that: • 1. total variability in test takers' scores can be explained by a combination of the test's reliability and error variability. • 2. differences between two persons in terms of percentage correct obtained on the same exam reflect differences in underlying latent traits. • 3. when test content is of varying difficulty, uniform scales of measurement can be applied to persons of different ability levels. • 4. the test taker's response to a given item will indicate the extent of the latent trait possessed.

3. when test content is of varying difficulty, uniform scales of measurement can be applied to persons of different ability levels. (correct answer)

The latent trait model or item response theory is an alternative to classical test theory. Item response theory is based on the premise that: • 1. total variability in test takers' scores can be explained by a combination of the test's reliability and error variability. • 2. differences between two persons in terms of percentage correct obtained on the same exam reflect differences in underlying latent traits. • 3. when test content is of varying difficulty, uniform scales of measurement can be applied to persons of different ability levels. • 4. the test taker's response to a given item will indicate the extent of the latent trait possessed.

3. when test content is of varying difficulty, uniform scales of measurement can be applied to persons of different ability levels. (correct answer) According to item response theory (latent trait model), it is assumed that item performance is related to the amount of the respondent's latent trait, e.g., statistics ability. Latent trait models are used to establish a uniform scale of measurement that can be applied to individuals of varying ability and to test content of varying difficulty. Response 1 accurately describes the premise of classical test theory. Response 2 is more in line with classical test theory than item response theory. The determination as to how much of a latent trait is possessed requires analyzing the responses to all items, not just one (Response 4).

Which of the following tests is appropriate for examining experimental outcome that is non-linear? • 1. eta. • 2. Multiple regression. • 3. Pearson r. • 4. Trend analysis.

4. Trend analysis. (correct answer)

Taylor-Russell tables demonstrate that to optimize your incremental validity you would want: • 1. both a base rate and a selection ratio of about .5. • 2. both a base rate and a selection ratio of about .1. • 3. a base rate of .1 and a selection ratio of .5. • 4. a base rate of .5 and a selection ratio of .1.

4. a base rate of .5 and a selection ratio of .1. (correct answer, your response) Incremental validity is the proportion of improvement in the success rate achieved by adding a predictor test (over and above the starting base rate). The base rate is the rate of selecting successful employees without a predictor test. The selection ratio is the ratio of the number of openings to the number of applicants (e.g., one opening for every 10 applicants is 1:10 or .1). So, incremental validity refers to how much using a predictor test improves our ability to select successful employees. According to the Taylor-Russell tables, incremental validity is optimized when the base rate is moderate (about .5) and the selection rate is low (close to .1). In other words, a good predictor test will make the biggest difference when there has been moderate success at choosing successful employees without the predictor test, and there is a large pool of applicants with relatively few openings.

A hypothetical research study has demonstrated that smoking behavior is correlated with socioeconomic status, lower SES being linked to higher levels of smoking. However, this relationship depends on age: the link between SES and smoking is highest among older persons and much less strong among young persons. When education is factored in, the correlation between smoking and SES goes down to close to 0.0. In this hypothetical example, education acts as: • 1. an independent variable. • 2. a dependent variable. • 3. a moderator variable. • 4. a mediator variable.

4. a mediator variable. (correct answer)

A hypothetical research study has demonstrated that smoking behavior is correlated with socioeconomic status, lower SES being linked to higher levels of smoking. However, this relationship depends on age: the link between SES and smoking is highest among older persons and much less strong among young persons. When education is factored in, the correlation between smoking and SES goes down to close to 0.0. In this hypothetical example, education acts as: • 1. an independent variable. • 2. a dependent variable. • 3. a moderator variable. • 4. a mediator variable.

4. a mediator variable. (correct answer, your response) A basic correlation is between a predictor variable and a criterion variable (this type of correlation is termed zero-order correlation). Here, the predictor is SES (sometimes termed independent variable, Response 1). The criterion is smoking (sometimes termed dependent variable, Response 2). Now it gets tricky. If this correlation between SES and smoking is weak when people are young and stronger when people are older, clearly "age" is a variable that affects the strength of the correlation. This type of variable is termed "moderator" (Response 3) and is defined as a variable that influences the strength of the relationship between two other variables (here, SES and smoking). A mediator variable (Response 4) explains why there is a relationship between the predictor and the criterion. For example, education explains the link between SES and smoking. The way to identify a mediator is that when you control for the effect of the mediator (in this case education) the relationship between the IV and the DV (in this case SES and smoking) goes down to close to

A Jacksonian Seizure is: • 1. a petit mal seizure. • 2. a complex partial seizure. • 3. a tonic/clonic seizure. • 4. a simple partial seizure.

4. a simple partial seizure. (correct answer, your response) A simple partial seizure begins focally in one hemisphere and does not impair the level of consciousness. This type of seizure may involve virtually any brain function (e.g., jerking of an extremity, abnormal sensation in one part of the body, a feeling of deja-vu, etc.). A Jacksonian Seizure is a type of simple partial seizure (Response 4) where there is a brief alteration in movement, sensation or nerve function that starts in one part of the body, then spreads to another, oftentimes beginning in an extremity and "marching" up the limb. A simple partial seizure is differentiated from a complex partial seizure (Response 2) in that the latter involves either a pronounced change in consciousness or outright loss of consciousness in addition to the other disruptions described. A tonic/clonic or grand mal seizure (Response 3), a type of generalized seizure, involves a tonic stage, which is characterized by continuous tension or contraction, followed by a clonic stage of rapid, involuntary, alternating muscular contractions, and relaxation. A petit mal seizure (Response 1), also a generalized seizure, is characterized by a brief change in the level of consciousness, a transitional period of twitching or eye-rolling and a return to normal functioning.

The most common extrapyramidal side effect of neuroleptics, which involves a sense of restlessness and jitters, is called: • 1. dystonia. • 2. parkinsonism. • 3. akinesia. • 4. akathisia.

4. akathisia. (correct answer, your response) Akathisia is the most prevalent side effect of neuroleptics, and involves restlessness and dysphoria. Akathisia is a common reason for medication non-compliance. Dystonia (Response 1) refers to acute contractions of the tongue, face, and neck. Parkinsonism (Response 2) involves stiffness, a mask-like face, shuffling gait, drooling, and tremors. Symptoms of akinesia (Response 3) include apathy, and a decrease in spontaneous gestures and voluntary movements. These difficult terms may be best remembered through recognizing their etymologies. Dystonia comes from "dys" (impairment or dysfunction) and "tone" (muscle tone). Akinesia comes from "a" (without) and "kinesia" (movements). Parkinsonism is a term used for symptoms that mimic those of Parkinson's Disease but have a different cause.

Family therapy based in part on social learning theory is most likely to focus on: • 1. hierarchy and boundaries. • 2. circular questioning and hypothesis formation. • 3. differentiation and lack of differentiation in the family structure. • 4. communication problems and deficient reward exchanges.

4. communication problems and deficient reward exchanges. (correct answer, your response) This question is challenging because it requires knowledge that behavioral marital therapy is rooted in both social learning theory and behavioral analysis. This theory contends that normal family functioning results when adaptive behavior is rewarded, maladaptive behavior is not reinforced, and benefits of being a member of the family outweigh costs. Pathology results from maladaptive behavior that is reinforced by family attention and reward, from deficient reward exchanges, and from communication deficits. If you weren't familiar with this information, you might have been able to arrive at the correct answer through process of elimination. Structural family therapists, such as Minuchin, focus on hierarchies, boundaries, and subsystems (Response 1). Circular questioning and hypothesis formation (Response 2) are techniques in Systemic Family Therapy (Milan group). Differentiation issues are the critical focus of Bowen's Family Systems therapy (Response 3).

In conducting an ABAB design, the researcher would be concerned with: • 1. history and maturation. • 2. regression and diffusion. • 3. failure of IV to return to baseline. • 4. failure of DV to return to baseline.

4. failure of DV to return to baseline. (correct answer)

In a chi-square test, measuring subjects twice (pre-intervention and post-intervention) results in a violation of: • 1. homoscedasticity. • 2. normality. • 3. random assignment. • 4. independence.

4. independence. (correct answer, your response) The chi-square test requires independence of observations, thus you cannot measure subjects twice or use any kind of repeated measures. Because the chi square test is non-parametric, it does not require normality (Response 2) or homoscedasticity (Response 1). Random assignment (Response 3) is not a concern here. Random assignment is the hallmark of true experimental research; the lack of random assignment in a quasi-experimental design can pose a threat to internal validity.

The key elements of Self-Instructional Therapy are: • 1. graded practice, coping statements, reinforcement. • 2. coping statements, skill development, self-reinforcement. • 3. problem-solving, graded practice, cognitive restructuring. • 4. repetition, graded practice, cognitive restructuring.

4. repetition, graded practice, cognitive restructuring. (correct answer)

The key elements of Self-Instructional Therapy are: • 1. graded practice, coping statements, reinforcement. • 2. coping statements, skill development, self-reinforcement. • 3. problem-solving, graded practice, cognitive restructuring. • 4. repetition, graded practice, cognitive restructuring.

4. repetition, graded practice, cognitive restructuring. (correct answer) Meichenbaum's Self-Instructional Therapy combines graduated practice with elements of Rational-Emotive Theory, and involves repeated practice of a targeted task. The five-step procedure begins with the therapist modeling a task and verbalizing the steps out loud, and ends with the client performing the task while thinking the task through. Self-Instructional Therapy helps a person who lacks adaptive cognitions in performing tasks to develop such cognitions, hence, it is a form of cognitive restructuring. Self-Instructional Training involves helping the client learn to think about how to perform a task, rather than problem-solving (Response 3). Coping statements and skill development are key elements of Meichenbaum's Stress Inoculation Training (Responses 1 and 2).

Which of the following theories is an expansion of Maslow's work? • 1. ERG theory. • 2. Two-Factor theory. • 3. Acquired needs theory. • 4. VIE theory.

Herzberg's Two-Factor Theory is an extension of Maslow's work, and divides Maslow's five needs into upper level needs (motivators) and lower level needs (hygiene factors). Although ERG theory (Response 1) is also based on Maslow's theory, it is not really an expansion, but rather a modification. For example, ERG theory includes three needs (existence, relatedness and growth), which are not arranged hierarchically. The theory includes the notion of frustration-regression, and also posits that satisfying a need can make the need even stronger. Acquired needs theory (Response 3) is based on McClelland's experiments with the TAT, and includes three work-related needs: nAch, nAff, and nPow. Vroom's VIE theory (Response 4) asserts that people will behave in ways that are based on their perceived expectancy of success and rewards.

Test-retest reliability would be the most appropriate type of reliability for: • 1. a test measuring an enduring trait. • 2. a test measuring a state. • 3. a test of intelligence. • 4. an instrument measuring autonomic arousal.

a test measuring an enduring trait. (correct answer)

In order to gain admittance to Elite Prep, an applicant must score high on all five sections of the entrance exam. This type of scoring is termed: • 1. additive. • 2. compensatory. • 3. nomothetic. • 4. conjunctive.

conjunctive. (correct answer)

In Donald Kirkpatrick's training evaluation model there are four levels of evaluation. Which of the following is not one of these levels? • 1. Reaction of the student. • 2. Instructor effectiveness. • 3. Behavior. • 4. Results.

• 2. Instructor effectiveness. (correct answer, your response) Donald Kirkpatrick has developed a four-level model to assess effectiveness of training. The first level of evaluation measures participant reactions (Response 1) in terms of what the student thought and felt about the training received. The second level measures learning, or the increase in skills and knowledge. The third level measures behavior/transfer (Response 3), the extent to which the skills and knowledge are being used in the everyday environment. Results (Response 4) is the final level, evaluated based on the effects on the business (e.g., business income) as a result of the training. Instructor effectiveness (Response 2) is not one of the levels of Kirkpatrick's training evaluation model.

A husband and wife have decided to use mediation to help end their marriage. At the end of the mediation process, both people are satisfied with the terms of the divorce settlement. This is an example of: • 1. informational justice. • 2. distributive justice. • 3. procedural justice. • 4. interpersonal justice.

• 2. distributive justice. (correct answer, your response) Distributive or outcome justice has its roots in social equity theory and refers to the satisfaction of participants in a process (e.g., mediation) when they believe that the outcome of the process is both fair and favorable. In this question the outcome (terms of the divorce settlement) determined the participants' satisfaction. Procedural justice (Response 3) refers to participants' satisfaction based on the perceived fairness of the process used to come to a decision. For example, if the couple was satisfied with the mediation because they felt the mediator was successfully impartial or that they had equal opportunity to be heard, this would be an example of procedural justice. A more recent development of research is the theory of interactional justice. Interactional justice is defined as the quality of interpersonal treatment received during the process, and reflects concerns about the fairness of non-procedural aspects of interaction. The two components of interactional justice are informational justice and interpersonal justice. Informational justice (Response 1) refers to the appropriateness of the explanation given for the procedures used, while interpersonal justice (Response 4) refers to the manner in which people are treated by authority figures in the decision making process (e.g., whether they are treated with politeness, dignity, and respect).

Your supervisor assigns you a project in which you have very little interest. Cognitive Dissonance Theory predicts that you would be most likely to change your opinion of this project if: • 1. you are given a bonus in exchange for doing this project. • 2. the project greatly benefits your colleagues. • 3. you complete the project on your own time at home. • 4. you have expertise with this type of project.

• 3. you complete the project on your own time at home. (correct answer, your response) According to Cognitive Dissonance Theory, if a person holds a belief that is inconsistent with his or her other actions or other views, the person will experience dissonance, or discomfort. The person will then attempt to resolve the dissonance by changing the problematic belief. In this scenario, you believe that you disinterested in the project. You are most likely to change this belief if you do something that is inconsistent with the belief. Out of the choices given, doing the project on your own time would be most inconsistent with disinterest in the project.

Which of the following accurately describes the difference between Bipolar Disorder with Psychotic Features and Schizoaffective Disorder? • 1. In Bipolar Disorder, mood and psychotic symptoms occur simultaneously, whereas in Schizoaffective Disorder, mood symptoms alternate with psychotic symptoms. • 2. Bipolar Disorder involves manic episodes with psychotic symptoms, while Schizoaffective Disorder involves either manic or depressive episodes with psychotic symptoms. • 3. Bipolar Disorder involves psychotic symptoms superimposed on mood symptoms, while Schizoaffective Disorder involves a distinct period of mood symptoms without psychotic symptoms. • 4. Bipolar Disorder involves psychotic symptoms superimposed on mood symptoms, while Schizoaffective Disorder involves a distinct period of psychotic symptoms without mood symptoms.

• 4. Bipolar Disorder involves psychotic symptoms superimposed on mood symptoms, while Schizoaffective Disorder involves a distinct period of psychotic symptoms without mood symptoms. (correct answer, your response) In Bipolar Disorder with Psychotic Features, psychotic symptoms occur in the presence of a Manic, Depressed, or Mixed Episode. In other words, when the mood symptoms remit, the person does not experience any psychotic symptoms. Schizoaffective Disorder is diagnosed in the presence of a Major Depressive Episode, Manic Episode, or Mixed Episode, concurrent with the symptoms of Schizophrenia. During the course of this disorder, delusions or hallucinations occur for at least 2 weeks without prominent mood symptoms. Therefore, Response 3 can be ruled out. In Schizoaffective Disorder, mood and psychotic symptoms do not alternate; rather, they are concurrent except for the two-week period when there are psychotic symptoms without mood symptoms (Response 1). Bipolar Disorder may involve not only Manic Episodes, but Depressive and Mixed Episodes as well (Response 2).


संबंधित स्टडी सेट्स

(Comm 89) reading: Diffusion of Innovation Theory (Rice)

View Set

Homework Chapter 12....Nano Server and Windows Containers

View Set

Chapter 8 - Treatment Procedures

View Set